Re: [FRIAM] [EXT] the Monty Hall problem

2023-08-11 Thread John Kennison
Hi Nick,

I think you are onto something with the "intuition trap". When I first heard 
the Monty Hall problem, I suspected the best strategy would be to stick to 
one's original choice. If Monty Hall is trying to get me to change my choice, 
he is probably trying to avoid having to give me an expensive car.

A mathematical proof requires nothing but cold logic. Finding a proof usually 
requires intuition.

--John


From: Friam  on behalf of Nicholas Thompson 

Sent: Wednesday, August 9, 2023 10:46 PM
To: The Friday Morning Applied Complexity Coffee Group 
Subject: [EXT] [FRIAM] the Monty Hall problem

In a  moment of supreme indolence [and no small amount of arrogance] I took on 
the rhetorical challenge of explaining the correct solution of the Monty Hall 
problem (switch).   I worked at it for several days and now I think it is 
perfect.


The Best Explanation of the Solution of the Monty Hall Problem

Here is the standard version of the Monty Hall Problem, as laid out in 
Wikipedia:

Suppose you're on a game show, and you're given the choice of three doors: 
Behind one door is a car; behind the others, goats. You pick a door, say No. 1, 
and the host, who knows what's behind the doors, opens another door, say No. 3, 
which has a goat. He then says to you, "Do you want to pick door No. 2?" Is it 
to your advantage to switch your choice?

This standard presentation of the problem contains some sly “intuition 
traps”,[1] so put aside goats and cars for a moment. Let’s talk about thimbles 
and peas.  I ask you to close your eyes, and then I put before you three 
thimbles, one of which hides a pea.  If you choose the one hiding a pea, you 
get all the gold in China.  Call the three thimbles, 1, 2, and 3.

1.I ask you to choose one of the thimbles.  You choose 1.  What is the 
probability that you choose the pea.   ANS: 1/3.

2.   Now, I group the thimbles as follows.  I slide thimble 2 a bit closer 
to thimble 3 (in a matter that would not dislodge a pea) and I declare that 
thimble 1 forms one group, A, and thimble 2 and 3 another group, B.

3.   I ask you to choose whether to choose from Group A or Group B: i.e, I 
am asking you to make your choice of thimble in two stages, first deciding on a 
group, and then deciding which member of the group to pick. Which group should 
you choose from?  ANS: It doesn’t matter.   If the pea is in Group A and you 
choose from it, you have only one option to choose, so the probability is 1 x 
1/3.  If the pea is in Group B and you choose from it, the pea has 2/3 chance 
of being in the group, but you must choose only one of the two members of the 
group, so your chance is again, 1/3:  2/3 x ½ = 1/3.

4.   Now, I offer to guarantee you that, if the pea is in group B, and you 
choose from group B, you will choose the thimble with the pea. (Perhaps I 
promise to slide the pea under whichever Group B thimble you choose, if you 
pick from Group B.)  Should you choose from Group A or Group B?   ANS:   Group 
B.  If you chose from Group A, and the pea is there, only one choice is 
possible, so the probability is still 1 x 1/3=1/3.   Now, however, if you chose 
from group B, and the pea is there, since you are guaranteed to make the right 
choice, the probability of getting the pea is 1 x 2/3=2/3.

5.   The effect of Monty Hall’s statement of the problem is to sort the 
doors into two groups, the Selected Group containing one door and the 
Unselected Group, containing two doors.   When he then shows you which door in 
the unselected group does not contain the car, your choice now boils down to 
choosing between Group A and Group B, which, as we have known all along, is a 
choice between a 1/3 and a 2/3 chance of choosing the group that contains the 
pea.



[1] The intuition trap has something to do with the fact that doors, goats, and 
cars are difficult to group.  So, it’s harder to see that by asking you to 
select one door at the beginning of the procedure, Monty has gotten you the 
group the doors and take the problem in two steps.  This doesn’t change the 
outcome, but it does require us to keep the conditional probabilities firmly in 
mind. “IF the car is in the unselected group, AND I choose from the unselected 
group, and I have been guaranteed to get the car if I choose from the 
unselected group, THEN, choosing from the unselected group is the better 
option.”
-. --- - / ...- .- .-.. .. -.. / -- --- .-. ... . / -.-. --- -.. .
FRIAM Applied Complexity Group listserv
Fridays 9a-12p Friday St. Johns Cafe   /   Thursdays 9a-12p Zoom 
https://bit.ly/virtualfriam
to (un)subscribe http://redfish.com/mailman/listinfo/friam_redfish.com
FRIAM-COMIC http://friam-comic.blogspot.com/
archives:  5/2017 thru present https://redfish.com/pipermail/friam_redfish.com/
  1/2003 thru 6/2021  http://friam.383.s1.nabble.com/


Re: [FRIAM] [EXT] Swirlies redux

2023-08-05 Thread John Kennison
Hi Nick,

My theory would be that in order to drain the upper bottle, the best route for 
the water is to move along a cyclical path.  And the best path follows a 
preferred rotation, say clockwise or counterclockwise.  If you start the 
rotation along the preferred rotation, then the bottle drains more quickly. But 
if you start the rotation in the opposite of the preferred rotation, the bottle 
takes more time to drain.


--John



From: Friam  on behalf of Nicholas Thompson 

Sent: Friday, August 4, 2023 10:05 PM
To: The Friday Morning Applied Complexity Coffee Group 
Subject: [EXT] [FRIAM] Swirlies redux


Ok, folks. I apologize to those of you who are fed up with my kitchen physics, 
but there has been a bit of a development in that saga that I want to share 
with those few of you who aren’t.   Years ago, I came home for the summer with 
my ears ringing with the notion that structures are formed to dissipate 
gradients.  Please set aside any teleological implications of this statement 
and ask the question in its most neutral form:  Do the structures that 
sometimes form as a gradient is dissipated dissipate it more quickly once the 
structure has been formed.   Or, as I came to interpret it, does facilitating 
the formation of such a structure speed the dissipation of the gradient.

I was the family dishwasher at the time.  I deplore washing dishes, but I love 
messing around with warm soapy water, and so I started to experiment with 
starting the vortex that forms after you pull the plug out of the sink before I 
pulled the plug.  Quickly, it became apparent that facilitating the vortex 
formation in that way GREATLY SLOWED the emptying of the sink.  Triumphally, I 
wrote Steve on Friam only to be greeted by a torrent of scatological raillery, 
so intense and so persistent from the fluid dynamicists on the list that I 
never heard from Steve. The burden of this raillery I have distilled into 
Roberts Rule of Order:  DEFROCKED ENGLISH MAJORS SHOULD NOT BE ALLOWED TO TALK 
about fluid dynamics.

More than a decade later, I am back in Massachusetts, washing dishes at the 
same sink, and the question occurred to me again. I raised it finally with 
Steve, and he generously sent me the little two-bottle toy, where you flip it 
over and the water drains from one bottle to the other.  As it drains, it forms 
a vortex in the draining bottle, and the occurrence of the vortex greatly 
increases the speed of the draining.  Finally, if one facilitates the formation 
of the vortex by rotating the bottle a bit, the bottle drains even more 
quickly.  Thus, the result is entirely different, especially if one substitutes 
two large pop bottles for the ones included in the kit.

At the risk of bringing another round of raillery down on my head, I opine that 
the difference has something to do with the fact that two bottle situation is 
more of a closed system than the sink situation.  The test would be to saw the 
bottom off both bottles and demonstrate that vortex-formation now slows 
drainage.

It will be a while, though, before I can get two extra bottles to destroy.

Does anybody care to make a prediction and offer an explanation why the results 
should be different in the two cases?

Nick
-. --- - / ...- .- .-.. .. -.. / -- --- .-. ... . / -.-. --- -.. .
FRIAM Applied Complexity Group listserv
Fridays 9a-12p Friday St. Johns Cafe   /   Thursdays 9a-12p Zoom 
https://bit.ly/virtualfriam
to (un)subscribe http://redfish.com/mailman/listinfo/friam_redfish.com
FRIAM-COMIC http://friam-comic.blogspot.com/
archives:  5/2017 thru present https://redfish.com/pipermail/friam_redfish.com/
  1/2003 thru 6/2021  http://friam.383.s1.nabble.com/


Re: [FRIAM] [EXT] Cumulus Nevis

2023-07-03 Thread John Kennison
Hi Nick,

We have been entertaining visitors for the week-end. I did notice that you seem 
to be back on the farm. Do you want to meet for chess or philosophy or both?

--John.

From: Friam  on behalf of Nicholas Thompson 

Sent: Saturday, July 1, 2023 3:40 PM
To: Frank Wimberly ; friam@redfish.com 
Subject: [EXT] [FRIAM] Cumulus Nevis

Frank, you can add this to your categories of cloud, cumulo-nimbus, cumulus 
congestus, cumulus castellatus, cumulo-arboris, and now cumulo-nevis

[IMG_1175.jpg] Actually, if I have my Latin right, it could be known as nix 
cumulus.



Nick.


Sent from my Dumb Phone
-. --- - / ...- .- .-.. .. -.. / -- --- .-. ... . / -.-. --- -.. .
FRIAM Applied Complexity Group listserv
Fridays 9a-12p Friday St. Johns Cafe   /   Thursdays 9a-12p Zoom 
https://bit.ly/virtualfriam
to (un)subscribe http://redfish.com/mailman/listinfo/friam_redfish.com
FRIAM-COMIC http://friam-comic.blogspot.com/
archives:  5/2017 thru present https://redfish.com/pipermail/friam_redfish.com/
  1/2003 thru 6/2021  http://friam.383.s1.nabble.com/


Re: [FRIAM] [EXT] New Mexico SNOTEL Santa Fe (922) (05P09S ) (PST) Daily series for wateryear=2023

2023-06-08 Thread John Kennison
Is that the Santa Fe Nonsoon or the Massachusetts Nonsoon

From: Friam  on behalf of Nicholas Thompson 

Sent: Wednesday, June 7, 2023 8:56 PM
To: The Friday Morning Applied Complexity Coffee Group 
Subject: [EXT] [FRIAM] New Mexico SNOTEL Santa Fe (922) (05P09S ) (PST) Daily 
series for wateryear=2023


https://wcc.sc.egov.usda.gov/nwcc/view?intervalType=+View+Current+=WYGRAPH=Daily=plot=922=WATERYEAR

Sent from Mail for Windows



The Joon Nonsoon continues.



N
-. --- - / ...- .- .-.. .. -.. / -- --- .-. ... . / -.-. --- -.. .
FRIAM Applied Complexity Group listserv
Fridays 9a-12p Friday St. Johns Cafe   /   Thursdays 9a-12p Zoom 
https://bit.ly/virtualfriam
to (un)subscribe http://redfish.com/mailman/listinfo/friam_redfish.com
FRIAM-COMIC http://friam-comic.blogspot.com/
archives:  5/2017 thru present https://redfish.com/pipermail/friam_redfish.com/
  1/2003 thru 6/2021  http://friam.383.s1.nabble.com/


Re: [FRIAM] [EXT] Fwd: [EXT] News Alert: Most young men are single. Most young women are not.

2023-02-22 Thread John Kennison
I would like to hear more about these strange numbers. They could arise from a 
huge number of lesbian attachments as compared to gay male attachments. 
Alternatively, there might be a much larger number of males as compared to 
females, despite the fact that men are more likely to commit suicide, or Nick's 
suggestion of women who feel attached to men who do not reciprocate that 
feeling (or to males to whom many women feel attached). . I don't see any other 
possibilities.

--John

From: Friam  on behalf of Nicholas Thompson 

Sent: Wednesday, February 22, 2023 12:02 PM
To: friam@redfish.com 
Subject: [EXT] [FRIAM] Fwd: [EXT] News Alert: Most young men are single. Most 
young women are not.

Last time I checked, the average number of attached males has to equal the 
average number of attached females, unless, of course, females, feel attached 
to men who don’t feel attached.

Sent from my Dumb Phone

Begin forwarded message:

From: The Hill 
Date: February 22, 2023 at 7:01:34 AM MST
To: nthomp...@clarku.edu
Subject: [EXT] News Alert: Most young men are single. Most young women are not.
Reply-To: emailt...@thehill.com


Click in for the latest news from The Hill.
[https://nxslink.thehill.com/img/6230d9d5b246d1049531e425i8dkh.26a0/4ea82909.gif]
 View 
Online
[I'm an image]
 
News Alert
News Alert
 
 
[Facebook]
   [Twitter] 

[LinkedIn] 

Re: [FRIAM] [EXT] Re: cognition all the way down

2020-10-17 Thread John Kennison

Hi Nick,

I read (once over lightly)  the piece by Levin and Dennett called "Cognition 
All the Way Down". It intrigues me but I also find myself saying "Yeah sure". 
Can we really say anything about the goals of microorganisms just by allowing 
ourselves the luxury of using language that would normally be reserved for 
objects that we believe are 'conscious' in some sense. This is the language 
that evolved as a convenient way of describing motives etc of the humans we 
encounter. Is that necessarily the best analytic language for examining what 
motive is? Or is it a language that is inherently based on dualism and 
therefore totally inappropriate for discussing the type of advances that the 
"Cognition All the Way Down" piece envisions?

If we find that having any kind of discussion of motive will inevitably get 
tangled up in inherently dualistic language, we need a different approach.

Here is a concrete proposal: Can we build a machine that functions effectively 
but can have a nervous breakdown? Maybe you, I and Jon could work on this.

--John

From: Friam  on behalf of thompnicks...@gmail.com 

Sent: Saturday, October 17, 2020 5:03 PM
To: 'The Friday Morning Applied Complexity Coffee Group' 
Subject: [EXT] Re: [FRIAM] cognition all the way down


Bah Dennett.  One of these days he is going to FINALLY read Peirce and realize 
that his entire career has been a footnote.  What a bounder!



Yeah.  I’m envious.



Nick



Nicholas Thompson

Emeritus Professor of Ethology and Psychology

Clark University

thompnicks...@gmail.com

https://wordpress.clarku.edu/nthompson/





From: Friam  On Behalf Of Roger Critchlow
Sent: Saturday, October 17, 2020 2:39 PM
To: The Friday Morning Applied Complexity Coffee Group 
Subject: [FRIAM] cognition all the way down



https://aeon.co/essays/how-to-understand-cells-tissues-and-organisms-as-agents-with-agendas



by Michael Levin and Daniel Dennett

via hackernews



-- rec --
-  . -..-. . -. -.. -..-. .. ... -..-.  . .-. .
FRIAM Applied Complexity Group listserv
Zoom Fridays 9:30a-12p Mtn GMT-6  bit.ly/virtualfriam
un/subscribe http://redfish.com/mailman/listinfo/friam_redfish.com
archives: http://friam.471366.n2.nabble.com/
FRIAM-COMIC http://friam-comic.blogspot.com/ 


Re: [FRIAM] [EXT] Re: [EXT] Re: A pluralistic model of the mind?

2019-12-12 Thread John Kennison
I'm pretty slow on the uptake in this conversation. I'm still thinking about 
there being no "out there". The language we use seems to be based on concepts 
such as "out there". So if "out there" makes no sense then our language is 
deeply flawed and, at best, an approximate instrument. It would hardly be 
surprising if there are things that our language cannot express. The same seems 
to be true of science which is based on experiments. The most fundamental kind 
of experiments seem to presume basic geometry which would, I think, involve a 
concept of "out there".

--John

From: Friam  on behalf of Prof David West 

Sent: Thursday, December 12, 2019 7:34 AM
To: friam@redfish.com 
Subject: [EXT] Re: [FRIAM] [EXT] Re: A pluralistic model of the mind?

Nick style larding follows:


On Thu, Dec 12, 2019, at 5:15 AM, Eric Charles wrote:
I think the effableness is a red herring. "Last night I ate spaghetti" doesn't 
fully and completely explain exactly what happened last night... but we all 
agree that I used words to describe a thing that is not "ineffable". So far, no 
argument has been offered to demonstrate that Dave's conversation with God is 
any more or less effable than my having eaten spaghetti

[DW-->in the case of speaking with God, I completely agree with you. In the 
second example, I was at least attempting to depict  an 'X' that was truly 
ineffable, in that even asserting a label like 'Experience' is falsehood. I 
will leave that for another time and consider your other comments. <---dw]


Absent an argument to that effect, we are begging the question by taking it as 
given that the two differ. I think the more interesting issue that Dave's 
example brings up is our original issue regarding monism, in its relation to 
the question "what is real?"

[DW -->May I restate as a question of what criteria are sufficient to assert 
that something is not real? In a previous post you asserted that something is 
not real if it has no "effects;" and you seem to reiterate that assertion in 
your remarks about a Deist god having no current effects. Are "effects" the 
criteria, and if so how do we utilize them to determine the reality of 'X'? <-- 
dw]

We all agree that that Dave could have been having a conversation with 
something real or something not real, right? We can call the other option 
"imaginary" or "illusory" or whatever else we want to call it, but we recognize 
that people sometimes have conversations with those types of conversational 
partners, so it is a live possibility.

I said in prior emails, we are in-particular talking about "monism" in contrast 
to mind-matter dualism (and all variants of that particular dualism), meaning 
that we reject that mental things and matter things are made of two different 
stuffs.

[DW -->This is a key point that I would really appreciate a good monist to 
explain to me. Using a metaphor of a computer, there seems to be one kind of 
"stuff," the ones and zeros (high and low voltages) flowing about a set of 
circuits. But, any given sequence of ones and zeros can 'effect' a given state 
of the collective circuitry, and any given sequence of states can effect more 
comprehensible constructs like the screensaver of Bears Ears that appears 
behind this email window. The graphic is, of course, but ones and zeros.  Ones 
and zeros is the "stuff" of all. What status have the "constructs" up to and 
including the images and icons? <--dw]

So Dave is talking to God. Whether he is talking to something that is "mental" 
or something "physical" is a post-hoc judgement. That is, we discover that 
based on future experience, not based on the initial experience, which is 
neutral with regards to that distinction. What later experiences will allow us 
to determine if the conversational partner is real? That is hard to specify 
when we are discussing a deity with ambiguous properties, but the method must 
be in principle very similar to how we would confirm or reject the reality of 
any other conversational partner. How do you determine when a child has an 
imaginary friend versus a real friend? You look for other consequences of the 
conversational partner. Ultimately we look for convergent agreement by anyone 
who honestly inquires into the existence of the conversational partner (i.e., 
the long term convergence / pattern-stability, referenced earlier in the 
conversation).

[DW -->I would argue that we have a body of precisely this kind of evidence, 
convergent agreement/pattern-stability, with regard "honest inquiries." I make 
this assertion with regard "goddness" but would make it emphatically with 
regard the "mystical otherness."  That evidence does not, however, seem to 
result in the assignation of "Real" status. So something else must be in play. 
What? <--dw]

The only thing we can't allow - because it is internally incoherent - is for 
there to exist a "real" thing with "no consequences" that we could investigate. 
So the 

Re: [FRIAM] [EXT] Re: A pluralistic model of the mind?

2019-12-08 Thread John Kennison
Eric,

Many thanks for your three versions of my question. I haven't decided whether 
any of them represent what my original question was but I appreciate all three. 
I guess I want to say that my original experience of "seeing" a bear in the 
woods is the same regardless of whether later experiences, by me or others, 
lead to the judgement that there was, or was not, an actual bear in the woods. 
Later judgements might affect how I regard my experience (or perhaps I should 
say influence  subsequent experience relating to my original experience.)

--John

From: Eric Charles 
Sent: Saturday, December 7, 2019 9:00 AM
To: John Kennison 
Cc: The Friday Morning Applied Complexity Coffee Group 
Subject: Re: [EXT] Re: [FRIAM] A pluralistic model of the mind?

John,
This is a wonderful question, and though it has already gone one way in the 
thread, I want to point out that there is another way it can go. "Are you 
really asserting," you ask, a bit rephrased, "that the bear I think is in the 
woods is somehow out there even when there is no bear?"

 We COULD understand your question as a philosophical/ontological/metaphysical 
one.  Along these lines, Glen channels Nick fairly well, and points out that 
the judgement of the in-woods-bear is illusory is a post-hoc judgement, which 
one can only arrive based on later experience. The judgement of "real" vs. 
"illusion" is after the fact, and the fact is that the initial experience is 
"bear in woods" without any such baggage. Nick brings in that this is a bit of 
a statistical issue, with expectations being based on past experience, and he 
points out that the "problem of induction" reminds us that the next time could 
always be different. Glen rightly chimes in with the observation that it is 
nigh impossible for us to see anything "fully prove out". That point is 
wonderful, because it brings us to Peirce's definition of "Truth". Recall that 
Peirce is the first combination History/Philosophy/Anthropology of Science guy. 
Sure, there is a lot written about science before Peirce, but Peirce has read 
the actual records of the scientists, and is a highly reputed scientist, and is 
interested in what Scientists are actually doing, not what they say they are 
doing, or what it might make abstract sense for them to be doing. Thus, when on 
good behavior, Peirce is explicitly articulating The Scientist's working 
definition of Truth: Truth is that upon which we would ultimately agree, when 
the dust of all the investigations settle. Truth is exactly that which will be 
fully proved out, should it take millennia for the proving. And until the dust 
settles, all assertions of Truth are provisional. Or, to phrase it differently, 
when a scientists asserts the truth of a conclusion within their field, they 
are exactly asserting that the conclusion will fully prove out in future 
investigation, and nothing more. If the conclusion doesn't prove out, then they 
were wrong. Any scientists trying to assert they are doing something else, 
something philosophically/ontologically/metaphysically deeper than that is, on 
Peirce's account, misrepresenting their actual activity and/or they have 
squarely stepped outside the role of Scientist.

We COULD understand your question as something bordering philosophy and 
psychology (at least as they were understood in the early 1900s). Returning to 
the start... That bear in the woods is initially experienced as out-there, and 
remains experienced as out-there, unless some later experience leads you to the 
conclusion that it is not out-there. Given John's initial question, we can 
surmise that the further investigation will lead you to not only conclude that 
there is no bear out there, but that there never was (the latter being a second 
conclusion, presumably distinct from the first). But when - via further 
investigation - we determine the bear was never-in-the-woods, what do we 
conclude? Is it possible to conclude "I was wrong that the bear was out there" 
without jumping immediately to "the bear was in-here the whole time"? Nick 
asserts that we can conclude our initial belief inaccurate without jumping 
immediately to the existence of "mental bears" in the mind/soul-theater/brain. 
He asserts that is possible, both because "in-here" creates a host of 
philosophical problems, and because we must not let the 20-steps-down-the-road 
conclusion color our view of the initial experience. The initial experience is 
unalterably of "a bear in the woods". That experience happened, past-tense, and 
it some sort of screwy post-hoc shenanigans to try use that conclusion to 
reinterpret the initial experience into something it wasn't. At this point, 
while we are clearly drawing upon what we laid out as Peircian in the first 
paragraph, we are actually in the middle of a Wi

Re: [FRIAM] [EXT] Re: A pluralistic model of the mind?

2019-12-06 Thread John Kennison
Hi Nick, and Eric,

I am grappling with Nick's ideas that mental states must be physical things and 
even are "out there" rather than "in here". What about delusions? If I think I 
see bear in the woods but I am mistaken, is this false perception "out there" 
even when the bear is not?

--John

From: Friam  on behalf of Eric Charles 

Sent: Thursday, December 5, 2019 8:41 PM
To: The Friday Morning Applied Complexity Coffee Group 
Subject: [EXT] Re: [FRIAM] A pluralistic model of the mind?

Nick,
Your need to complicate things is fascinating. You are a monist. You are a 
monist is the sense of not thinking that "mental" things and "physical" things 
are made of different stuffs. At that point, you can throw a new word in the 
mix (e.g., 'experience', 'neutral stuff'), or you can throw your hat in with 
one or the other side of the original division, e.g., "I am a materialist" or 
"I am an idealist". To that, you add the insight that that later discussion is 
all a bit weird, because once you have decided to be a monist it weirdly 
doesn't matter much what you call the stuff.That insight is in need of support, 
because the old dichotomy is so built in to our language and culture that the 
claim it doesn't matter which side you choose is very unintuitive. That is 
solid, and you should develop it further.

Instead, you bring up some sort of discussion about serial vs. parallel 
processing that has nothing to do with that topic at all, then you muddle the 
issues up. Whether you think of "consciousness" as "serial" or "parallel" has 
no bearing on the prior issue. Given that you are talking with a bunch of 
computationally minded people, and that you brought up Turing Machines, the 
first problem is that a serial system can simulate a parallel system, so while 
parallel buys you time savings (sometimes a little, sometimes a lot), it 
doesn't change what the system is capable of in any more fundamental way 
(assuming you are still limited to writing zeros and ones). But you don't even 
need that, because it just doesn't matter. Being a "monist" has nothing to do 
with the serial vs. parallel issue at all. There is no reason a body can't be 
doing many things at once. Or, you can change your level of analysis and 
somehow set up your definition so that there is only one thing the body is 
doing, but that one thing has parts. It is just a word game at that point. If I 
have a 5-berry pie, is it 5 different types of pie at once, or is it its own 1 
flavor of pie? We can talk about the pros and cons of labeling it different 
ways, but it is the same thing whichever way we label it and... it has 
nothing to do with monism vs. dualism

Admonishment over.

So... Say more about the monism part... That is a solid issue and you are 
getting somewhere with it...

It SEEMS so important a difference if one person claims that all we can never 
know is ideas ("You don't know 'the chair', just your idea of the chair!") and 
another person claims that knowing isn't ever a thing and that there is just 
material ("There is no 'idea' of the chair, there is only your physical body in 
relation to the physical world!"). It seems that they are making vastly 
different claims, and that they should disagree about almost everything. How is 
it that THAT doesn't matter?

Eric


---
Eric P. Charles, Ph.D.
Department of Justice - Personnel Psychologist
American University - Adjunct Instructor



On Thu, Dec 5, 2019 at 1:20 AM 
mailto:thompnicks...@gmail.com>> wrote:

Hi, everybody,



I have gotten all the communications off of nabble and concentrated them below. 
 If you read this message in plain text, a lot of useful formatting will go 
away, so I encourage you to enable HTML.  Or perhaps, I can fit it all up as a 
Word file, tomorrow.



.  I have not had time to dig into the contents much.  I am pleased that 
everybody took the issue straight on, and I look forward to grappling with your 
comments.



A recapitulation of the thread:



First, some text from the review which Roger sent:



This is exactly as radical as it sounds. Bishop Berkeley and other idealists 
argued that objects are dependent on mind; Manzotti argues the reverse of this: 
Mind exists in objects. In The Spread Mind, Manzotti contends that we are 
mistaken to believe that objects “do not depend on our presence. . . . Our 
bodies enable processes that change the ontology of the world. Our bodies bring 
into existence the physical objects with which our experience is identical. We 
are our experience. We are not our bodies.” And later: “We are the world and 
the world is us—everything is physical.” This includes dreams, hallucinations, 
memories—all are the imagined physical objects themselves, not neural firings 
or mental representations (we must at one time have perceived an object to 
hallucinate or dream it, although it can be an unreal combination of other 
objects, as in the case 

Re: [FRIAM] [EXT] FW: Mathematical Inquiry

2019-08-29 Thread John Kennison
Hi Nick,

I'm not a probabilist nor a statistician
but I think you could find a math web site that would give you what you want. 
The Wolfram Mathematica gives the normal distribution and I imagine that you 
could subtract one distribution function from another .

I'm not certain why you seem to think it would be normally distributed, which 
is symmetric in both directions. I vaguely think it would be some kind of beta 
function, in part because I remember pictures of beta distributions and they 
seen to be about right--but I have forgotten the hypotheses that would lead to 
such a function.

--John


From: Friam  on behalf of Nick Thompson 

Sent: Monday, August 26, 2019 5:41 AM
To: Friam 
Subject: [EXT] [FRIAM] FW: Mathematical Inquiry




Dear Mathematical Friammers,



What follows is a problem in mathematics, which, of course, has nothing to do 
with me.



Jones is a diabetic, and he has a glucose monitor that gives him his exact 
blood glucose level moment to moment.  Jones notices at that after breakfast, 
his blood sugars behave in in very different manners, even though he eats 
exactly the same food every day, doesn’t exercise at that time of day ever, and 
takes exactly the same amount of insulin.  Some mornings, his blood sugar rises 
steadily for several hours after a meal, sometimes it falls steadily.  Only 
rarely does it remain steady.  One variable seems left for Jones to control and 
that is the exact timing of the relation between when he take his insulin and 
the time he begins his meal.



So, Jones imagines a model as follows.  Because Jones always takes exactly the 
amount of insulin necessary to account for the amount of sugar he eats, he 
assumes that the curves of insulin activity and sugar activity are both normal 
curves, with the same median time and the same sd and, therefore, the same area 
under the curve.  However, one curve is offset from the other because sometimes 
Jones takes his insulin before he eats his sugar and sometimes he eats his 
sugar before he takes his insulin.  Bearing in mind that the Insulin curve 
SUBTRACTS from the sugar curve, Jones wonders about the shape of the difference 
curve that results from different offsets between eating his meal and taking 
his insulin.  He wonders if, perhaps, that this whole dramatic failure of 
control, could be due to the fact that on some days he takes his insulin a 
little too early and the sugar in the meal is slow to catch up and on other 
days, he takes it too late and the insulin is slow to  catch up.  Thus, the 
correct offset is a tipping point, an unstable equilibrium which is very 
difficult to achieve.



Jones is not a mathematician, but he hangs around with mathematicians, and he 
suspects that there is a software that is readily available on line for free 
that would allow him to display the different curves that result from the 
different offsets and, perhaps, even better, display the function that relates 
the integral of the difference function as a function of the offset.   This 
function might have some interesting properties that could be used to guide 
Jones’s injection behavior.



Does anybody have any thoughts on Jones’s predicament?



Not that I care, but still,



Nick



Nicholas S. Thompson

Emeritus Professor of Psychology and Biology

Clark University

http://home.earthlink.net/~nickthompson/naturaldesigns/



FRIAM Applied Complexity Group listserv
Meets Fridays 9a-11:30 at cafe at St. John's College
to unsubscribe http://redfish.com/mailman/listinfo/friam_redfish.com
archives back to 2003: http://friam.471366.n2.nabble.com/
FRIAM-COMIC http://friam-comic.blogspot.com/ by Dr. Strangelove


Re: [FRIAM] Formalizing the concept of design

2018-10-30 Thread John Kennison
Thanks Glen,


I will look into "Anticipatory Systems".--John


From: Friam  on behalf of ∄ uǝʃƃ 

Sent: Tuesday, October 30, 2018 9:38:44 AM
To: FriAM
Subject: Re: [FRIAM] Formalizing the concept of design

FWIW, "Life Itself" is inadequate for me to read Rosen with any sympathy.  I 
would also recommend a copy of Anticipatory Systems.  E.g. o, Rosen seems to 
cite [†] this from von Neumann [‡]:

>> All these are very crude steps in the direction of a systematic theory of 
>> automata.  They represent, in addition, only one particular direction.  This 
>> is, as I indicated before, the direction towards forming a rigorous concept 
>> of what constitutes "complication."  They illustrate that "complication" on 
>> its lower levels is probably degenerative, that is, that every automaton 
>> that can produce other automata will only be able to produce less 
>> complicated ones.  There is, however, a certain minimum level where this 
>> degenerative characteristic ceases to be universal.  At this point automata 
>> which can re produce themselves, or even construct higher entities, become 
>> possible. This fact, that complication, as well as organization, below a 
>> certain minimum level is degenerative, and beyond that level can become self 
>> - supporting and even increasing, will clearly play an important role in any 
>> future theory of the subject.

[†] Rosen, "Anticipatory Systems", pp 321-322. Pergamon Press, 1985.
[‡] von Neumann, "The General and Logical Theory of Automata", p 318.  In Taub, 
"Collected Works".


On 10/30/18 6:27 AM, John Kennison wrote:
> I am going to wait until I receive my copy of Rosen's "Life Itself" which I 
> ordered, and which is due art the end of this week. I want to take another 
> look at what Rosen promised (or seemed to promise) and what he delivered.

--
∄ uǝʃƃ


FRIAM Applied Complexity Group listserv
Meets Fridays 9a-11:30 at cafe at St. John's College
to unsubscribe http://redfish.com/mailman/listinfo/friam_redfish.com
FRIAM-COMIC http://friam-comic.blogspot.com/ by Dr. Strangelove

FRIAM Applied Complexity Group listserv
Meets Fridays 9a-11:30 at cafe at St. John's College
to unsubscribe http://redfish.com/mailman/listinfo/friam_redfish.com
FRIAM-COMIC http://friam-comic.blogspot.com/ by Dr. Strangelove


Re: [FRIAM] Formalizing the concept of design

2018-10-30 Thread John Kennison
Hi Nick, and others,


I am going to wait until I receive my copy of Rosen's "Life Itself" which I 
ordered, and which is due art the end of this week. I want to take another look 
at what Rosen promised (or seemed to promise) and what he delivered.


--John



From: Friam  on behalf of Nick Thompson 

Sent: Sunday, October 28, 2018 2:32 AM
To: 'The Friday Morning Applied Complexity Coffee Group'
Cc: 'Jon Zingale'
Subject: [FRIAM] Formalizing the concept of design


Dear Frank, Jon, Eric, and anybody else,



OK.  Let me be blunt.  I wish that the mathematically inclined among you would 
help me.  As I have told you all before, my brother was a mathematician and I 
got one of his two math genes, which was enough to give me vague mathematical 
intuitions but not enough that I could actually make good on more than a few of 
them.  I have long been carrying around in my head the notion of “design 
arrays" which I have offered as a kind of universal way of looking at such 
troublesome concepts as adaptation, motivation, communication, learning, 
development, etc., where our explanatory concepts seem to be fatally entangled 
with our descriptive ones.  A design co-array is a co-listing of circumstances 
and adaptive techniques, with all the pairings leading to a common outcome.



I guess I am wondering, Is this way of thinking about telic phenomena a 
mathematical way?  It seems to me to relate to the idea of mapping.  The 
motivated animal maps his behavior onto his circumstances and thence, 
convergently, onto outcomes.  But I am also wondering, aside from making my 
deceased big brother proud, is there any benefit to formalizing it.  It is my 
understanding of mathematics that the benefit of formalization is the capacity 
to be led, through the formalization to some unexpected prediction concerning 
the phenomenon.  It’s hard for me to see what benefits such a formalization 
would provide.



To make it as easy for you to think about this problem, I have ocr-ed its most 
lucid and concise description among my papers, cleaned it up, and attached it 
above.



I am eager for anybody’s thoughts.



Nick



Nicholas S. Thompson

Emeritus Professor of Psychology and Biology

Clark University

http://home.earthlink.net/~nickthompson/naturaldesigns/



From: Friam [mailto:friam-boun...@redfish.com] On Behalf Of Frank Wimberly

Sent: Saturday, October 27, 2018 5:24 PM

To: The Friday Morning Applied Complexity Coffee Group 

Subject: Re: [FRIAM] Open Letter, draft #2



Well, I'm not a sequential machine although my wife has her doubts.



Thanks for the algebraic geometry suggestions.  Jon Zingale and I will try to 
master the subject.  Others may join us on Saturday mornings if they wish.



Frank

---

Frank Wimberly



My memoir:

https://wacsequentisl mww.amazon.com/author/frankwimberly



My scientific publications:

https://www.researchgate.net/profile/Frank_Wimberly2



Phone (505) 670-9918



On Sat, Oct 27, 2018, 5:16 PM John Kennison  wrote:

Hi Frank,



I didn't realize it was supposed to be a joke --it seemed like a relevant 
example. I'm not an algebraic geometer but:



 . . . there is a historical survey in 
https://www.ime.usp.br/~pleite/pub/artigos/abhyankar/abhyankar.pdf

Historical Ramblings in Algebraic Geometry and Related Algebra

www.ime.usp.br

Historical Ramblings in Algebraic Geometry and Related Algebra Author(s): 
Shreeram S. Abhyankar Source: The American Mathematical Monthly, Vol. 83, No. 6 
(Jun. - Jul ...



If you read that you can tell if you like Ahbyankar's style. He wrote a more 
thorough survey in 295 pages called "Algebraic Geometry for Scientists and 
Engineers'' (including computer scientists.



--John





From: Friam  on behalf of Frank Wimberly 


Sent: Saturday, October 27, 2018 5:53:53 PM

To: The Friday Morning Applied Complexity Coffee Group

Subject: Re: [FRIAM] Open Letter, draft #2



Sorry, John.  It was a weak attempt to be humorous.



Also, I mistyped.  I meant "algebraic geometry" when I was asking for a book 
recommendation.



Frank

---

Frank Wimberly



My memoir:

https://www.amazon.com/author/frankwimberly



My scientific publications:

https://www.researchgate.net/profile/Frank_Wimberly2



Phone (505) 670-9918



On Sat, Oct 27, 2018, 12:56 PM Marcus Daniels  wrote:

John writes:



“Is there something that animals, or more particularly humans, can do which we 
can prove cannot be duplicated by a sequential machine?”



A sequential computer program could simply be a loop that sampled random 
numbers and indexed into the address space of the computer program itself (not 
its memory).   One could make a specialized computer using a FPGA that even had 
an instruction to do that random dispatching.   To counter the arguments of 
Penrose, one could do the same using quantum states.



ht

Re: [FRIAM] Formalizing the concept of design

2018-10-29 Thread John Kennison
Hi Nick,


I would like to grapple with your challenge to mathematicians, but I need a 
clearer idea of what the challenge is. Here are some possibilities:


(1) Recently, I have been trying to find some kind of mathematical entity that 
produces a "chronicle" (i.e. a sequence of outcomes in response to various 
stimuli) but is not equivalent to a sequential machine. Is this search for such 
an entity a reasonable response (or a partial response) to your challenge?


(2) Are you specifically interested in a mathematical model of motives?


(3) Or would you like a mathematical discussion of what Rosen was searching for 
and what he accomplished?


(4) Or are you looking for something else?


--John


From: Friam  on behalf of glen 
Sent: Monday, October 29, 2018 8:50:15 AM
To: The Friday Morning Applied Complexity Coffee Group
Subject: Re: [FRIAM] Formalizing the concept of design

My silence is due to my attempts to unplug on the weekends. I think examples 
abound. So my next question was to ask what type of example would you care 
about? I can't give an example in "natural design" because I don't know what 
that phrase means. But Marcus has already mentioned ADFs in genetic 
programming. We've just mentioned social (high order) and physiological (low 
order) influences on sexual behavior. Intercellular "communication" can happen 
via molecule diffusion and extracellular vesicles. Etc.

It seems similar to that aphorism: "Nonlinear systems is like non-elephant 
zoology" ... or somesuch, misquoted from somebody somewhere. 8^)

On October 28, 2018 11:20:52 PM PDT, Nick Thompson  
wrote:
>Hi, Glen,
>
>
>
>I am continuing to think about what you say below.  I guess, in my
>defense, I would say that this a statement about what natural designs
>means, not a claim that all animals behave in this highly schematized
>way.  After all, even in the course of orgiastic sex, the organism
>continues to breath (well mostly) so the different activities can be
>simultaneous as well as switched between.   For real examples, have a
>look at some of the diagrams in Tinbergen’s Study of Instinct.  See
>below
>
>
>
>I just reread Rosen's chapter on epistemology:  God what a hot mess!
>But exciting, still.  I wish somebody would sit down and read it with
>me.
>
>Nick
>
>
>
>Nicholas S. Thompson
>
>Emeritus Professor of Psychology and Biology
>
>Clark University
>
>http://home.earthlink.net/~nickthompson/naturaldesigns/
>
>
>
>
>
>
>
>
>
>-Original Message-
>From: Friam [mailto:friam-boun...@redfish.com] On Behalf Of ? u???
>Sent: Sunday, October 28, 2018 4:53 AM
>To: FriAM 
>Subject: Re: [FRIAM] Formalizing the concept of design
>
>
>
>This description suffers from the same criticism I made before: you're
>assuming a *strict* hierarchy, where the higher order can only operate
>over whole components from the lower order.  I.e. the gun's algorithm
>1st chooses the type/medium of target (ballistic, air, water), then
>uses that type to select the specific tracking sub-algorithm.
>
>
>
>And while this is mostly how it's done in artificial systems, I suspect
>biology does NOT use strict hierarchies.  A higher order function can
>operate over a mixture of operands, some complex wholes in that higher
>order and some from the lower orders.  E.g. if the gun's higher order
>selection is based not only on the 3 types (ballistic, air, water), but
>also on a lower order measure like *speed*, then it may well use he
>same sub-algorithm for both air and water.  So, it takes both high
>order constructs and low order constructs as its operands.
>
>
>
>You see your assumption of a strict hierarchy peeking through when you
>say sex is the only motive that is ESSENTIALLY social.  What do you
>mean by "essentially"?  Couldn't we say that *all* the behavior of all
>the social animals is, in part, social?  ... including following others
>to the water hole?  So, these functions would be mixed ... do not obey
>a strict hierarchy.
>
>
>
>On 10/27/18 11:32 PM, Nick Thompson wrote:
>
>> But the function that connects the two arrays will be different in
>the two kinds of gun because a surface target is capable of different
>sorts of motion from an aerial target.
>
>> [...]
>
>> So, the gun would display two levels of design, the lower level that
>relates trajectory to firing and the higher level that relates the
>lower level design to target type.
>
>> [...]
>
>> This conception of multiple hierarchical layers of design is a useful
>
>
>> way to describe many of the phenomena that ethologists and
>
>> socio-biologists are required to explain. …
>
>
>
>
>
>
>
>
>
>
>
>--
>
>∄ uǝʃƃ
>
>
>
>
>
>FRIAM Applied Complexity Group listserv
>
>Meets Fridays 9a-11:30 at cafe at St. John's College to unsubscribe
>
>http://redfish.com/mailman/listinfo/friam_redfish.com
>
>FRIAM-COMIC  

Re: [FRIAM] Open Letter, draft #2

2018-10-27 Thread John Kennison
Hi Frank,


I didn't realize it was supposed to be a joke --it seemed like a relevant 
example. I'm not an algebraic geometer but:


 . . . there is a historical survey in 
https://www.ime.usp.br/~pleite/pub/artigos/abhyankar/abhyankar.pdf

Historical Ramblings in Algebraic Geometry and Related 
Algebra
www.ime.usp.br
Historical Ramblings in Algebraic Geometry and Related Algebra Author(s): 
Shreeram S. Abhyankar Source: The American Mathematical Monthly, Vol. 83, No. 6 
(Jun. - Jul ...




If you read 
that you can tell if you like Ahbyankar's style. He wrote a more thorough 
survey in 295 pages called "Algebraic Geometry for Scientists and Engineers'' 
(including computer scientists.


--John




From: Friam  on behalf of Frank Wimberly 

Sent: Saturday, October 27, 2018 5:53:53 PM
To: The Friday Morning Applied Complexity Coffee Group
Subject: Re: [FRIAM] Open Letter, draft #2

Sorry, John.  It was a weak attempt to be humorous.

Also, I mistyped.  I meant "algebraic geometry" when I was asking for a book 
recommendation.

Frank

---
Frank Wimberly

My memoir:
https://www.amazon.com/author/frankwimberly

My scientific publications:
https://www.researchgate.net/profile/Frank_Wimberly2

Phone (505) 670-9918

On Sat, Oct 27, 2018, 12:56 PM Marcus Daniels 
mailto:mar...@snoutfarm.com>> wrote:

John writes:



“Is there something that animals, or more particularly humans, can do which we 
can prove cannot be duplicated by a sequential machine?”



A sequential computer program could simply be a loop that sampled random 
numbers and indexed into the address space of the computer program itself (not 
its memory).   One could make a specialized computer using a FPGA that even had 
an instruction to do that random dispatching.   To counter the arguments of 
Penrose, one could do the same using quantum states.



https://www.springer.com/us/book/9781402078941



There are all kinds of physical processes that are simulated on classical 
supercomputers, of course.



Marcus


FRIAM Applied Complexity Group listserv
Meets Fridays 9a-11:30 at cafe at St. John's College
to unsubscribe http://redfish.com/mailman/listinfo/friam_redfish.com
FRIAM-COMIC http://friam-comic.blogspot.com/ by Dr. Strangelove

FRIAM Applied Complexity Group listserv
Meets Fridays 9a-11:30 at cafe at St. John's College
to unsubscribe http://redfish.com/mailman/listinfo/friam_redfish.com
FRIAM-COMIC http://friam-comic.blogspot.com/ by Dr. Strangelove


Re: [FRIAM] Open Letter, draft #2

2018-10-27 Thread John Kennison
P.S. I just realized that when Frank said "Look no further than me" he was not 
referring to his considerable knowledge, but to himself. It's a nice point.

JK

____
From: John Kennison
Sent: Saturday, October 27, 2018 2:32:37 PM
To: The Friday Morning Applied Complexity Coffee Group
Subject: Re: [FRIAM] Open Letter, draft #2


Hi Marcus, Frank, et al,


I am a mathematician who knows category theory, which Rosen seems to have found 
exciting. My thinking about a machine that would learn how to reprogram itself 
is surely at a naive level, but it seems to me that it would feel more like a 
being than a machine. All I know about Genetic Programming is what I just read 
after Googling that term and it looks like great idea. Frank's credentials are 
impressive but I don't know enough to, as yet, formulate a reasonable question 
to him.


It looks that some very sophisticated programs can evolve and reprogram 
themselves which means, I think, that sequential machines can do amazing 
things. I guess my question would be:


Is there something that animals, or more particularly humans, can do which we 
can prove cannot be duplicated by a sequential machine?'


I have read Dennett's "Consciousness Explained" and Hofstadter's "I am a 
Strange Loop". There are parts of each book which still seem vague to me, but 
it seems likely that the answer to the above question is "No".  But then I 
would need a different approach to trying to figure out what Rosen might be 
driving at. (I once had s copy of Rosen's "Life Itself" but I can't find it now 
--so I ordered a used copy for about $9.).


On Geometric Algebra --my Googling of this term suggests that it has to do with 
what I would call tensors, which I saw briefly in an undergraduate Physics 
course, and very abstractly in a graduate Math course. I convinced myself they 
were about the same thing


--John


From: Friam  on behalf of Marcus Daniels 

Sent: Saturday, October 27, 2018 11:11:10 AM
To: The Friday Morning Applied Complexity Coffee Group
Subject: Re: [FRIAM] Open Letter, draft #2


How idoes genetic programming with automatic function definition not achieve 
this?



From: Friam  on behalf of John Kennison 

Reply-To: The Friday Morning Applied Complexity Coffee Group 
Date: Saturday, October 27, 2018 at 6:21 AM
To: "Friam@redfish. com" 
Subject: Re: [FRIAM] Open Letter, draft #2



I like the idea of a non-sequential machine, or perhaps, a being, whose 
operation is NOT determined by knowing how its component parts function. I 
don’t see how to go about constructing such a thing, unless I assume that there 
are laws of physics which remain undiscovered. So for now, I will settle on 
trying to describe a ``machine’’ which is not a sequential machine. I think 
that Rosen is right in saying that having a parallel machine (in which various 
operations happen simultaneously) will not do the trick because given any 
parallel machine one can define a sequential machine that functions in the same 
way. One might make a machine in which the outputs only happen with a certain 
specified probability, but I don’t think that is different enough. So I 
rephrase the problem as describing an entity that receives inputs and produces 
outputs that is cannot be duplicated (or reasonably modeled) by a probabilistic 
sequential machine (one in which the outputs happen with a specified 
probability). I thought of starting with a sequential machine, which has rules 
for how to react to inputs when in a given state. But now let’s suppose that 
the rules may change. The entity is capable of learning new rules. This seems 
more biological –the entity can reprogram itself. But I find that naive ways of 
reprograming can probably be duplicated by a sequential machine. For example, 
if the machine reprograms when dissatisfied with previous performance, then 
there is a new state, of being dissatisfied, and the reprograming activity can, 
it seems, be re-expressed as more sophisticated rules for producing an output.



--John



From: Friam  on behalf of Tom Johnson 

Sent: Wednesday, October 24, 2018 4:23:01 PM
To: Friam@redfish. com
Subject: Re: [FRIAM] Open Letter, draft #2



Yes, but not with multiple signatories.  Sorry.


Tom Johnson
Institute for Analytic Journalism   -- Santa Fe, NM USA
505.577.6482(c)505.473.9646(h)
NM Foundation for Open Government<http://nmfog.org>
Check out It's The People's 
Data<https://www.facebook.com/pages/Its-The-Peoples-Data/1599854626919671>

http://www.jtjohnson.com<http://www.jtjohnson.com/>   
t...@jtjohnson.com<mailto:t...@jtjohnson.com>






On Wed, Oct 24, 2018 at 11:45 AM Nick Thompson 
mailto:nickthomp...@earthlink.net>

Re: [FRIAM] Open Letter, draft #2

2018-10-27 Thread John Kennison
Hi Marcus, Frank, et al,


I am a mathematician who knows category theory, which Rosen seems to have found 
exciting. My thinking about a machine that would learn how to reprogram itself 
is surely at a naive level, but it seems to me that it would feel more like a 
being than a machine. All I know about Genetic Programming is what I just read 
after Googling that term and it looks like great idea. Frank's credentials are 
impressive but I don't know enough to, as yet, formulate a reasonable question 
to him.


It looks that some very sophisticated programs can evolve and reprogram 
themselves which means, I think, that sequential machines can do amazing 
things. I guess my question would be:


Is there something that animals, or more particularly humans, can do which we 
can prove cannot be duplicated by a sequential machine?'


I have read Dennett's "Consciousness Explained" and Hofstadter's "I am a 
Strange Loop". There are parts of each book which still seem vague to me, but 
it seems likely that the answer to the above question is "No".  But then I 
would need a different approach to trying to figure out what Rosen might be 
driving at. (I once had s copy of Rosen's "Life Itself" but I can't find it now 
--so I ordered a used copy for about $9.).


On Geometric Algebra --my Googling of this term suggests that it has to do with 
what I would call tensors, which I saw briefly in an undergraduate Physics 
course, and very abstractly in a graduate Math course. I convinced myself they 
were about the same thing


--John


From: Friam  on behalf of Marcus Daniels 

Sent: Saturday, October 27, 2018 11:11:10 AM
To: The Friday Morning Applied Complexity Coffee Group
Subject: Re: [FRIAM] Open Letter, draft #2


How idoes genetic programming with automatic function definition not achieve 
this?



From: Friam  on behalf of John Kennison 

Reply-To: The Friday Morning Applied Complexity Coffee Group 
Date: Saturday, October 27, 2018 at 6:21 AM
To: "Friam@redfish. com" 
Subject: Re: [FRIAM] Open Letter, draft #2



I like the idea of a non-sequential machine, or perhaps, a being, whose 
operation is NOT determined by knowing how its component parts function. I 
don’t see how to go about constructing such a thing, unless I assume that there 
are laws of physics which remain undiscovered. So for now, I will settle on 
trying to describe a ``machine’’ which is not a sequential machine. I think 
that Rosen is right in saying that having a parallel machine (in which various 
operations happen simultaneously) will not do the trick because given any 
parallel machine one can define a sequential machine that functions in the same 
way. One might make a machine in which the outputs only happen with a certain 
specified probability, but I don’t think that is different enough. So I 
rephrase the problem as describing an entity that receives inputs and produces 
outputs that is cannot be duplicated (or reasonably modeled) by a probabilistic 
sequential machine (one in which the outputs happen with a specified 
probability). I thought of starting with a sequential machine, which has rules 
for how to react to inputs when in a given state. But now let’s suppose that 
the rules may change. The entity is capable of learning new rules. This seems 
more biological –the entity can reprogram itself. But I find that naive ways of 
reprograming can probably be duplicated by a sequential machine. For example, 
if the machine reprograms when dissatisfied with previous performance, then 
there is a new state, of being dissatisfied, and the reprograming activity can, 
it seems, be re-expressed as more sophisticated rules for producing an output.



--John



From: Friam  on behalf of Tom Johnson 

Sent: Wednesday, October 24, 2018 4:23:01 PM
To: Friam@redfish. com
Subject: Re: [FRIAM] Open Letter, draft #2



Yes, but not with multiple signatories.  Sorry.


Tom Johnson
Institute for Analytic Journalism   -- Santa Fe, NM USA
505.577.6482(c)505.473.9646(h)
NM Foundation for Open Government<http://nmfog.org>
Check out It's The People's 
Data<https://www.facebook.com/pages/Its-The-Peoples-Data/1599854626919671>

http://www.jtjohnson.com<http://www.jtjohnson.com/>   
t...@jtjohnson.com<mailto:t...@jtjohnson.com>






On Wed, Oct 24, 2018 at 11:45 AM Nick Thompson 
mailto:nickthomp...@earthlink.net>> wrote:

Thanks, Tom,



Still trying to figure out logistics.  I have written the NM-ican to find out 
how to submit a letter from many signers, but got no response.  Do you have any 
experience with this?



Nick



Nicholas S. Thompson

Emeritus Professor of Psychology and Biology

Clark University

http://home.earthlink.net/~nickthompson/naturaldesigns/



Re: [FRIAM] Open Letter, draft #2

2018-10-27 Thread John Kennison
I like the idea of a non-sequential machine, or perhaps, a being, whose 
operation is NOT determined by knowing how its component parts function. I 
don’t see how to go about constructing such a thing, unless I assume that there 
are laws of physics which remain undiscovered. So for now, I will settle on 
trying to describe a ``machine’’ which is not a sequential machine. I think 
that Rosen is right in saying that having a parallel machine (in which various 
operations happen simultaneously) will not do the trick because given any 
parallel machine one can define a sequential machine that functions in the same 
way. One might make a machine in which the outputs only happen with a certain 
specified probability, but I don’t think that is different enough. So I 
rephrase the problem as describing an entity that receives inputs and produces 
outputs that is cannot be duplicated (or reasonably modeled) by a probabilistic 
sequential machine (one in which the outputs happen with a specified 
probability). I thought of starting with a sequential machine, which has rules 
for how to react to inputs when in a given state. But now let’s suppose that 
the rules may change. The entity is capable of learning new rules. This seems 
more biological –the entity can reprogram itself. But I find that naive ways of 
reprograming can probably be duplicated by a sequential machine. For example, 
if the machine reprograms when dissatisfied with previous performance, then 
there is a new state, of being dissatisfied, and the reprograming activity can, 
it seems, be re-expressed as more sophisticated rules for producing an output.

--John

From: Friam  on behalf of Tom Johnson 

Sent: Wednesday, October 24, 2018 4:23:01 PM
To: Friam@redfish. com
Subject: Re: [FRIAM] Open Letter, draft #2

Yes, but not with multiple signatories.  Sorry.


Tom Johnson
Institute for Analytic Journalism   -- Santa Fe, NM USA
505.577.6482(c)505.473.9646(h)
NM Foundation for Open Government
Check out It's The People's 
Data
http://www.jtjohnson.com   
t...@jtjohnson.com



On Wed, Oct 24, 2018 at 11:45 AM Nick Thompson 
mailto:nickthomp...@earthlink.net>> wrote:

Thanks, Tom,



Still trying to figure out logistics.  I have written the NM-ican to find out 
how to submit a letter from many signers, but got no response.  Do you have any 
experience with this?



Nick



Nicholas S. Thompson

Emeritus Professor of Psychology and Biology

Clark University

http://home.earthlink.net/~nickthompson/naturaldesigns/



From: Friam 
[mailto:friam-boun...@redfish.com] On Behalf 
Of Tom Johnson
Sent: Wednesday, October 24, 2018 11:14 AM
To: Friam@redfish. com mailto:friam@redfish.com>>
Subject: Re: [FRIAM] Open Letter, draft #2



I will sign on, Nick.

Tom Johnson, Professor Emeritus, San Francisco State University



Tom Johnson
Institute for Analytic Journalism   -- Santa Fe, NM USA
505.577.6482(c)505.473.9646(h)
NM Foundation for Open Government
Check out It's The People's 
Data

http://www.jtjohnson.com   
t...@jtjohnson.com






On Tue, Oct 23, 2018 at 6:09 PM Nick Thompson 
mailto:nickthomp...@earthlink.net>> wrote:

Hi, all,



Here is a new draft of the open letter, pared down to meet the New Mexican’s 
requirements.  I have a few signatories, would love some more.  I will bring 
the letter with me to our service on Friday.  Even if you don’t plan to sign, 
please feel free to point out typos and other infelicities.



Nick



To the New Mexican

We are industrial researchers and retired college professors, living in Santa 
Fe.  We urge all Santa Feans to vote, but particularly young voters and their 
parents.  Institutions of learning and their students are under stress.
Every day, we meet young people as dedicated to learning as our best research 
students in the 70’s and 80’s, yet are working as cashiers, ride hail drivers, 
waiters and waitresses.  Under present conditions, these talented young people 
cannot afford to go to university and, without that training, will never take 
up the leadership positions their talent should make possible.  The nation will 
need these students as our generation retires from institutes, government 
laboratories, colleges, and universities.  Please take time to vote and to tell 
your representatives to support education at every level.  The future safety 
and prosperity of our nation 

Re: [FRIAM] On old question

2018-10-25 Thread John Kennison
Hi Nick,


I'm thinking I should look at a newer book by Rosen and see if it seems. better 
than "Life Itself". Do you think that the book you ordered (I'm not certain 
what it was) would be good? Or, alternatively, what is the best recent book by 
Rosen?


--John


From: Friam  on behalf of Nick Thompson 

Sent: Wednesday, October 24, 2018 3:38:32 PM
To: 'The Friday Morning Applied Complexity Coffee Group'
Subject: Re: [FRIAM] On old question


Thanks, Marcus,



I ordered the book.  Time I revived old memory traces.



Nick



Nicholas S. Thompson

Emeritus Professor of Psychology and Biology

Clark University

http://home.earthlink.net/~nickthompson/naturaldesigns/



From: Friam [mailto:friam-boun...@redfish.com] On Behalf Of Marcus Daniels
Sent: Wednesday, October 24, 2018 12:56 AM
To: The Friday Morning Applied Complexity Coffee Group ; 
Roger Critchlow 
Subject: Re: [FRIAM] On old question



Nick,



It sounds like you are describing mutual information.  This is ancient, but a 
nice overview of related topics:



https://www.amazon.com/Information-Theory-Qualitative-Quantitative-Applications/dp/0803921322



Marcus



From: Friam mailto:friam-boun...@redfish.com>> on 
behalf of Nick Thompson 
mailto:nickthomp...@earthlink.net>>
Reply-To: The Friday Morning Applied Complexity Coffee Group 
mailto:Friam@redfish.com>>
Date: Wednesday, October 24, 2018 at 12:22 AM
To: Roger Critchlow mailto:r...@elf.org>>, Friam 
mailto:Friam@redfish.com>>
Subject: [FRIAM] On old question



Dear Roger, and anybody else who wants to play,



While waiting for my paper, Signs and Designs, to be rejected, I have gone back 
to thinking about an old project, whose working title has been “A Sign 
Language.”  And this has led me back to Robert Rosen, whose Life Itself I 
bought almost 9 years ago and it has remained almost pristine, ever since.  In 
the chapter I am now looking at, Rosen is talking about “organization.”  Now, I 
have been thinking about organization ever since I read C. Ray Carpenter’s 
early work on primate groups back in the late 50’s.  It seemed to me at the 
time, and it seems to me reasonable now, to define the organization of a set of 
entities as related in some way to the degree to which one can predict the 
behavior of one entity from knowledge about another.  Now the relationship is 
not straightforward, because neither total unpredictability (every monkey 
behaves exactly the same as every other monkey in every situation) nor total 
unpredictability (no monkey behaves like any other monkey in ANY situation) 
smacks of great organization.  The highest levels organization, speaking 
inexpertly and intuitively, seem to correspond to intermediate levels of 
predictability, where there were several classes of individuals within a group 
and within class predictability was strong but cross-class predictability was 
weak.  On my account, the highest levels of organization involve hierarchies of 
predictability.  Thus honey bees and ants are more organized than starling 
flocks, say.



This is where the matter stood at the point that I came to Santa Fe and started 
interacting with you guys 14 years ago.  You-all introduced me to a totally 
different notion of organization based – shudder – on the second law.  But I 
have never been able to deploy your concept with any assurance.  So, for 
instance, when I shake the salad dressing, I feel like I am disorganizing it, 
and when it reasserts itself into layers, I feel like it ought to be called 
more organized.  But I have a feeling that you are going to tell me that the 
reverse is true.  That, given the molecules of fat and water/acid, that the 
layered state is the less organized state.



Now this confusion of mine takes on importance when I try to read Rosen.  He 
defines a function as the difference that occurs when one removes a component 
of a system.  I can see no reason why the oil or the water in my salad dressing 
cannot be thought of components of a system and if, for instance, I were to 
siphon out the water from the bottom of my layered salad dressing, I could 
claim that the function of the water had been to hold the water up.  This seems 
a rather lame notion of function.



Some of you who have been on this list forever will remember that I raised the 
same kind of worry almost a decade back when I noticed the drainage of water 
from a basin was actually slowed by the formation of a vortex.  This seemed to 
dispel any notion that vortices are structures whose function is to efficiently 
dispel a gradient.  It also violated my intuition from traffic flows, where I 
imagine that rigid rules of priority would facilitate the flow of people 
crossing bridges to escape Zozobra.



It’s quite possible that my confusions in this matter are of no great general 
applicability, in which case, I look forward to being ignored.



Nick



Nicholas S. Thompson

Emeritus Professor of Psychology and Biology

Clark 

Re: [FRIAM] On old question

2018-10-24 Thread John Kennison
I remember reading, or trying to read, Rosen's "Life Itself". For a long time 
it seemed like Rosen was onto something very important and exciting but, to my 
mind, he never even came close to delivering. The conclusion of "Life Itself" 
was, for me, a complete disappointment.


The material presented here seems similar. I can feel the same kind of 
excitement, but I wonder if any real progress will be made on the issues that 
are raised.


I guess I have missed much of the conversation on this issue. Maybe my comments 
are way too late, but I would appreciate it if someone with a more positive 
view of Rosen would try to explain what it is that Rosen achieved.


--John




From: Friam  on behalf of Nick Thompson 

Sent: Wednesday, October 24, 2018 4:42:18 PM
To: 'The Friday Morning Applied Complexity Coffee Group'
Subject: Re: [FRIAM] On old question


Thanks, everybody, for your responses.



Most of them are way above my pay-grade, but watching you all work together is 
inspiring, and I always, ALWAYS, get a rich harvest of crumbs off your table.  
Back in old days, I did a brief visitorship at the Philadelphia Child Guidance 
Clinic, toying with the idea of giving up ethology to be a family therapist.  
They quickly sent me back home to academia, insisting that I would more good 
for group therapy theory by pushing on as an ethologist.  I took that as a 
compliment, at the time.  (}8-0]



A few years later, a clinical graduate student came to me and asked me to help 
her think about the problem of “function” in the family therapy literature.  At 
the time, Salvador Munichin, Jay Haley, and others were toying with the idea of 
the Indicated Patient.  The notion was that every troubled family designates 
one of its individuals to be the patient, and unites in support of that 
person’s illness.  Because the family has a stake in the illness, the illness 
cannot be cured without the whole family present.



This entailed the notion that the patient’s illness FUNCTIONED to hold the 
family together.  The patient, and his/her illness were like an organ of the 
body, or like a soldier ant in a colony, etc.  The patients served their 
families and their families directed or regulated their service.  I thought it 
was a fascinating theoretical problem, but the Department stopped hiring family 
systems people, and that was the end of that.   Another lesson noted.



So here it is.  The problem of defining the system and what

it means for a system as a whole to cause changes in one of its components.  I 
sent the graduate student off to find a simple, straightforward biological 
model, and she never came back and was rumored to have had a nervous breakdown 
and left graduate school.



Let that be a lesson to you.



Here is another example to chew on.  It is often said that young male baboons 
serve as the patrols of a baboon troop, a kind of trip wire over which 
attacking predators have to pass in order to attack the troop.  The mechanistic 
explanation is that a young maturing male baboon is seen as a threat to the 
dominant males and is kept away from the females (at the center of the troop) 
by his betters.  He is certainly not a trip wire by choice; but is he so by 
design?   Presumably his “trip-wire-ness” is a spandrel.  So, in what sense is 
he functioning for anything?



Nick





Nicholas S. Thompson

Emeritus Professor of Psychology and Biology

Clark University

http://home.earthlink.net/~nickthompson/naturaldesigns/





-Original Message-
From: Friam [mailto:friam-boun...@redfish.com] On Behalf Of Prof David West
Sent: Wednesday, October 24, 2018 12:16 PM
To: friam@redfish.com
Subject: Re: [FRIAM] On old question



Glen, I was trying very hard to be metaphorical and general and avoid 
addressing details, like the one you raise, that would require pages to discuss 
properly. But,  I would definitely and immediately concede that Rosen, in 
particular, recognizes and deals with, sometimes more cleverly than others, 
with the kind of qualitative differences I mention. But, even there, I would 
suggest that the foundations of his work are still primarily derived from 
"Entropic Sciences" and their concepts and formulations.



davew





On Wed, Oct 24, 2018, at 11:07 AM, uǝlƃ ☣ wrote:

> So ... Rosen's openness to material flow, closure to operational flow,

> allows *both* endo- and exothermic sub-systems.  But his (M,R)-systems

> focus on maintaining organization using energy-material harvested from

> the gradient, ignoring sub-systems that produce energy-material?

>

> On 10/24/18 9:20 AM, Prof David West wrote:

> > I am pretty sure that the questions you pose, and the ideas of

> > people like Rosen, arise from a failure to recognize the qualitative

> > difference among structures and therefore miss the essence of 
> > "organization."

> > This, in turn is largely attributable to the fact that we enjoy a

> > highly developed "science" of Entropy 

Re: [FRIAM] Google self-evolving AlphaZero artificial intelligence program mastered chess from scratch in 4 hours: Rich Murray 2017.12.10

2017-12-11 Thread John Kennison
Hi Nick et al,


The three AlphaZero ganders I enjoy are discussed at


https://www.youtube.com/watch?v=lFXJWPhDsSY

Deep Mind Alpha Zero's "Immortal Zugzwang Game" against 
Stockfish<https://www.youtube.com/watch?v=lFXJWPhDsSY>
www.youtube.com
Read more about Deep Mind Alpha Zero here https://arxiv.org/pdf/1712.01815.pdf 
Link to the other games https://lichess.org/study/wxrovYNH A chess game 
betwee...




https://www.youtube.com/watch?v=lb3_eRNoH_w

Google Deep Mind AI Alpha Zero Devours 
Stockfish<https://www.youtube.com/watch?v=lb3_eRNoH_w>
www.youtube.com
Read more about Deep Mind here https://arxiv.org/pdf/1712.01815.pdf Link to the 
other games https://lichess.org/study/wxrovYNH A chess game between Deep M...




https://www.youtube.com/watch?v=pcdpgn9OINs

[https://www.bing.com/th?id=OVF.7i%2bi7ksmwRzpI1lsqMzJ%2bQ=Api]<https://www.youtube.com/watch?v=pcdpgn9OINs>

Deep Mind AI Alpha Zero Dismantles Stockfish's French 
Defense<https://www.youtube.com/watch?v=pcdpgn9OINs>
www.youtube.com
Check out all my videos on this match 
https://www.youtube.com/playlist?list=PLDnx7w_xuguHIxbL7akaYgEvV4spwYkmn Read 
more about Deep Mind Alpha Zero here http...




--John


From: Friam <friam-boun...@redfish.com> on behalf of Nick Thompson 
<nickthomp...@earthlink.net>
Sent: Monday, December 11, 2017 11:41:20 PM
To: 'The Friday Morning Applied Complexity Coffee Group'
Subject: Re: [FRIAM] Google self-evolving AlphaZero artificial intelligence 
program mastered chess from scratch in 4 hours: Rich Murray 2017.12.10


John,



Is one of these “exciting” games played through and commented on any engine 
that mere mortals can access?



n



Nicholas S. Thompson

Emeritus Professor of Psychology and Biology

Clark University

http://home.earthlink.net/~nickthompson/naturaldesigns/



From: Friam [mailto:friam-boun...@redfish.com] On Behalf Of John Kennison
Sent: Monday, December 11, 2017 2:23 PM
To: The Friday Morning Applied Complexity Coffee Group <friam@redfish.com>
Subject: Re: [FRIAM] Google self-evolving AlphaZero artificial intelligence 
program mastered chess from scratch in 4 hours: Rich Murray 2017.12.10



I agree that it's not clear that AlphaZero would excel at the supergame.I 
described. Still human intuition is probably not the indispensable ingredient 
that it once might have seemed to be.



On the other hand, many chess commentators think that AlphaZero has a playing 
style that is "more human" than the styles of other chess computers. When 
humans play chess, we can often discern themes --maybe one player is trying to 
breakthrough in the center while the other  is trying to breakthrough on a 
flank. In contrast, most computers are simply trying to maximize a position 
evaluation function. This only leads to a successful breakthrough if the 
computer can see in advance that the breakthrough will lead, in a relatively 
short time, to a measurable advantage, such as the forced win of a pawn.



Humans sometimes say that they need a plan -even a bad plan is said to be 
better than playing without a plan. AlphaZero's games against the computer 
Stockfish seem to pursue clear-cut plans (at least the games that have been 
made available). It may be the case that having a plan leads to better play. 
The point is that the plan changes the evaluation function --if you want to 
breakthrough in the center, you try to post your pieces differently than if you 
are planning to breakthrough in a flank. Having a plan, even a bad plan, may 
lead to better coordination of your pieces --even for a computer.



Magnus Carlsen, the current human world chess champion, said that if you play 
against a top computer you will surely lose but you will also be bored. I think 
that you find a game you are watching interesting when you can sense competing 
plans behind the moves. AlphaZero's games are quite exciting.



From: Friam <friam-boun...@redfish.com<mailto:friam-boun...@redfish.com>> on 
behalf of Russ Abbott <russ.abb...@gmail.com<mailto:russ.abb...@gmail.com>>
Sent: Monday, December 11, 2017 12:36:53 PM
To: The Friday Morning Applied Complexity Coffee Group
Subject: Re: [FRIAM] Google self-evolving AlphaZero artificial intelligence 
program mastered chess from scratch in 4 hours: Rich Murray 2017.12.10



Not clear that AlphaZero would do well on John's SuperGame.  It won on chess 
(and Go) by playing against itself in advance. If it doesn't have the 
opportunity to do that it won't have that advantage. It's strategy would have 
to be something like on-the-fly playing the selected game against itself in the 
background at the same time as it is playing the human opponent. The question 
then is how fast it can teach itself the new game.  It's strategy would have to 
be to slow down the game against the opponent as much as possible to give 
itself time to lea

Re: [FRIAM] Google self-evolving AlphaZero artificial intelligence program mastered chess from scratch in 4 hours: Rich Murray 2017.12.10

2017-12-11 Thread John Kennison
ed with the industrial revolution and more 
importantly perhaps, the mechanization of war where chess strategy, now 
somewhat more "scientific" began to eventually give rise to "hypermodernism" 
which focus more on controlling the center of the board from afar (a parallel 
to mechanized warfare where power could be projected over a great distance in a 
short amount of time).   Algorithmic play and mathematical analysis has been 
considered since the late Romanitc period but didn't come into it's own  until 
the modern digital computer, with Claude Shannon taking an early swipe at the 
problem as early as 1950!   The fact that it took more than 50 years to get to 
Deep Blue's thin victory over Kasparov is more a testimony to how subtle and 
hard Chess is than how intelligent humans are, etc.

"Deep Learning" itself seems like nothing more (and nothing less) than the 
latest innovation in machine learning (game theory, neural nets, cellular 
automata, genetic algorithms, learning classifiers, etc.) which *could* very 
well portend the breakaway point of the AI-driven technological singularity.  
I'm not THAT up on "Deep Learning" but things like Generative Antagonistic 
Networks (and other unsupervised machine learning) seem to have the key quality 
of not needing supervision by humans to learn...  there may be one more level 
of indirection to be had before things go ape-shit (exponentially speaking)...

 I personally don't imagine that a *single* AI will be the source of this, but 
rather a Cambrian-explosion-like plethora of AI's, though they may be so 
pervasive and promiscuous as to cross-fertilize so thoroughly that they will be 
a single "organism" for all practical purposes.



- Steve




From: Friam [mailto:friam-boun...@redfish.com] On Behalf Of John Kennison
Sent: Monday, December 11, 2017 7:17 AM
To: The Friday Morning Applied Complexity Coffee Group 
<friam@redfish.com><mailto:friam@redfish.com>
Subject: Re: [FRIAM] Google self-evolving AlphaZero artificial intelligence 
program mastered chess from scratch in 4 hours: Rich Murray 2017.12.10



I once thought I had a sure-fire way to make games between humans and computers 
fairer. Start with a large set of chess-like games that use different boards, 
different pieces, different rules. Enumerate the games so that each one 
corresponds to a n-digit binary numeral (for large n). Then make a "super game" 
in which the players start by creating a n digit binary numeral by taking turns 
in which they can specify one of the n binary digits. The super game would 
continue by playing the chess-like game that corresponds to the created number.



In a super game between a human and a computer, the computer would not have 
access to all the insights into the nature of chess that humans have 
established over hundreds of years of playing chess and which chess playing 
computers use to defeat humans.  Of course, the human player would also be 
deprived of all the years of research into chess, but humans can use their 
marvelous intuition to figure out a reasonable set of strategies even for a 
game they haven't studied before. The computer, without a reasonable set of 
strategies, would (I assumed) find little benefit from  its massive computing 
power.



The new AlphaZero game playing computer refutes my idea.





From: Friam <friam-boun...@redfish.com<mailto:friam-boun...@redfish.com>> on 
behalf of Rich Murray <rmfor...@gmail.com<mailto:rmfor...@gmail.com>>
Sent: Monday, December 11, 2017 12:16:26 AM
To: Rich Murray
Subject: [FRIAM] Google self-evolving AlphaZero artificial intelligence program 
mastered chess from scratch in 4 hours: Rich Murray 2017.12.10







https://futurism.com/4-hours-googles-ai-mastered-chess-knowledge-history/



Chess isn’t an easy game, by human standards. But for an artificial 
intelligence powered by a formidable, almost alien mindset, the trivial 
diversion can be mastered in a few spare hours.



In a new paper, Google researchers detail how their latest AI evolution, 
AlphaZero, developed “superhuman performance” in chess, taking just four hours 
to learn the rules before obliterating the world champion chess program, 
Stockfish.



In other words, all of humanity’s chess knowledge – and beyond – was absorbed 
and surpassed by an AI in about as long as it takes to drive from New York City 
to Washington, DC.



After being programmed with only the rules of chess (no strategies), in just 
four hours AlphaZero had mastered the game to the extent it was able to best 
the highest-rated chess-playing program Stockfish.



In a series of 100 games against Stockfish, AlphaZero won 25 games while 
playing as white (with first mover advantage), and picked up three games 
playing as black.

The rest of the contests were draws, with Stockfish recording no wins and 
AlphaZero no losses.



[https://ipmc

Re: [FRIAM] Google self-evolving AlphaZero artificial intelligence program mastered chess from scratch in 4 hours: Rich Murray 2017.12.10

2017-12-11 Thread John Kennison
I once thought I had a sure-fire way to make games between humans and computers 
fairer. Start with a large set of chess-like games that use different boards, 
different pieces, different rules. Enumerate the games so that each one 
corresponds to a n-digit binary numeral (for large n). Then make a "super game" 
in which the players start by creating a n digit binary numeral by taking turns 
in which they can specify one of the n binary digits. The super game would 
continue by playing the chess-like game that corresponds to the created number.


In a super game between a human and a computer, the computer would not have 
access to all the insights into the nature of chess that humans have 
established over hundreds of years of playing chess and which chess playing 
computers use to defeat humans.  Of course, the human player would also be 
deprived of all the years of research into chess, but humans can use their 
marvelous intuition to figure out a reasonable set of strategies even for a 
game they haven't studied before. The computer, without a reasonable set of 
strategies, would (I assumed) find little benefit from  its massive computing 
power.


The new AlphaZero game playing computer refutes my idea.


From: Friam  on behalf of Rich Murray 

Sent: Monday, December 11, 2017 12:16:26 AM
To: Rich Murray
Subject: [FRIAM] Google self-evolving AlphaZero artificial intelligence program 
mastered chess from scratch in 4 hours: Rich Murray 2017.12.10



https://futurism.com/4-hours-googles-ai-mastered-chess-knowledge-history/

Chess isn’t an easy game, by human standards. But for an artificial 
intelligence powered by a formidable, almost alien mindset, the trivial 
diversion can be mastered in a few spare hours.

In a new paper, Google researchers detail how their latest AI evolution, 
AlphaZero, developed “superhuman performance” in chess, taking just four hours 
to learn the rules before obliterating the world champion chess program, 
Stockfish.

In other words, all of humanity’s chess knowledge – and beyond – was absorbed 
and surpassed by an AI in about as long as it takes to drive from New York City 
to Washington, DC.

After being programmed with only the rules of chess (no strategies), in just 
four hours AlphaZero had mastered the game to the extent it was able to best 
the highest-rated chess-playing program Stockfish.

In a series of 100 games against Stockfish, AlphaZero won 25 games while 
playing as white (with first mover advantage), and picked up three games 
playing as black.
The rest of the contests were draws, with Stockfish recording no wins and 
AlphaZero no losses.

[https://ipmcdn.avast.com/images/icons/icon-envelope-tick-round-orange-animated-no-repeat-v1.gif]
Virus-free. 
www.avast.com

FRIAM Applied Complexity Group listserv
Meets Fridays 9a-11:30 at cafe at St. John's College
to unsubscribe http://redfish.com/mailman/listinfo/friam_redfish.com
FRIAM-COMIC http://friam-comic.blogspot.com/ by Dr. Strangelove

Re: [FRIAM] Proofs of God?

2017-10-13 Thread John Kennison
I don't think that a rigorous proof of how evolution works would be all that 
earth-shaking. Most openly non-scientific religions have had much experience at 
simply ignoring such proofs and the more liberal religions have found ways to 
co-exist with science ("Maybe God used evolution to create the world". In my 
own religion (Unitarian-Universalism) sermons that mention God usually include 
formulations such as "God, as you understand the term". Buddhism does not 
require a belief in God.


From: Friam  on behalf of George Duncan 

Sent: Friday, October 13, 2017 2:44:18 PM
To: Stephen Guerin; The Friday Morning Applied Complexity Coffee Group
Subject: Re: [FRIAM] Proofs of God?

By
Jeremy England
Oct. 12, 2017 6:29 p.m. ET
311 
COMMENTS

I recently learned that I play a role in Dan Brown’s new novel, “Origin.” Mr. 
Brown writes that Jeremy England, an MIT physics professor, “was currently the 
toast of Boston academia, having caused a global stir” with his work on 
biophysics. The description is flattering, but Mr. Brown errs when he gets to 
the meaning of my research. One of his characters explains that my literary 
doppelgänger may have “identified the underlying physical principle driving the 
origin and evolution of life.” If the fictional Jeremy England’s theory is 
right, the suggestion goes, it would be an earth-shattering disproof of every 
other story of creation. All religions might even become obsolete.

It would be easy to criticize my fictional self’s theories based on Mr. Brown’s 
brief description, but it would also be unfair. My actual 
research on how lifelike behaviors 
emerge in inanimate matter is widely available, whereas the Dan Brown 
character’s work is only vaguely described. There’s no real science in the book 
to argue over.

My true concern is with my double’s attitude in the book. He is a prop for a 
billionaire futurist whose mission is to demonstrate that science has made God 
irrelevant. In that role, Jeremy England says he is just “trying to describe 
the way things ‘are’ in the universe” and that he “will leave the spiritual 
implications to the clerics and philosophers.”

Two years ago I wrote in Commentary magazine that it is impossible simply to 
describe “the way things are” without first making the significant choice of 
what language to speak in. The language of physics can be extremely useful in 
talking about the world, but it can never address everything that needs to be 
said about human life. Equations can elegantly explain how an airplane stays in 
the air, but they cannot convey the awe someone feels when flying above the 
clouds. I’m disappointed in my fictional self for being so blithely 
uninterested in what lies beyond the narrow confines of his technical field.

I’m a scientist, but I also study and live by the Hebrew Bible. To me, the idea 
that physics could prove that the God of Abraham is not the creator and ruler 
of the world reflects a serious misunderstanding—of both the scientific method 
and the function of the biblical text.

Science is an approach to common experience. It addresses what is objectively 
measurable by inventing models that summarize the world’s partial 
predictability. In contrast, the biblical God tells Moses at the burning bush: 
“I will be what I will be.” He is addressing the uncertainty the future brings 
for all. No prediction can ever fully answer the question of what will happen 
next.

Humans will always face a choice about how to react to the unknowable future. 
Encounters between God and the Hebrew prophets are often described in terms of 
covenants, partly to emphasize that seeing the hand of God at work starts with 
a conscious decision to view the world a certain way.

Consider someone who assumes that all existence is the work of a creator who 
speaks through the events of the world. He can follow that assumption down the 
road and decide whether God seems to be keeping his side of the bargain. Many 
of us live like this and feel that with time our trust in him has been 
affirmed. There’s no scientific argument for this way of drawing meaning from 
experience. But there’s no way science could disprove it either, because it is 
outside the scope of scientific inquiry.

Some religious adherents do make claims that deserve to be disputed by science. 
For instance, they may openly acknowledge that their deepest beliefs are 
incompatible with the existence of dinosaurs. The fictional me—and perhaps Mr. 
Brown too—might hope to put these holdouts back on their heels. But disputes 
like this never answer the most important question: Do we need to keep learning 
about God? For my part, in light of everything I know, I am certain that we do.

Mr. England is a professor of physics at the Massachusetts 

Re: [FRIAM] Calculus for 9 year olds

2016-03-30 Thread John Kennison
Nick,

Do NOT try to teach your 9 year old granddaughter calculus. Instead give her 
challenging problems that can be solved by elementary methods. If you try to 
teach her something she is not ready for, math will become a chore --possibly a 
chore for which she is greatly praised, but still a chore. Something bad 
happens when people see math as a chore. They try to memorize a lot of methods 
and go further than their understanding will normally take them. Doing math by 
memorizing without understanding is a dead end street.

Many high schools, maybe most, now teach calculus and think they have taken a 
big step forward in their math education. But if they rush through algebra and 
geometry to do this, they have ruined their teaching of math. Instead of 
turning out people who understand algebra and geometry, they produce people who 
get by on memory. A Clark U., we have, for quite some time, required that 
students pass placement exams in algebra, geometry and trig before being 
admitted to Calculus. The first year we did this (in the 1980s) a student came 
to my office saying there was a serious mistake in his math placement because 
he was placed in remedial algebra when, in fact, he had already taken calculus 
and gotten an A in it, so was ready for second year calculus. I thought Clark 
had made a mistake, but, to be sure, I asked him some elementary algebra 
questions, such as to expand (x+1)^2  (IN THIS email I use ^2 for "2 in the 
exponent"
when I talked to the student I simply wrote the 2 as a superscript.) The 
student produced x^2 + 4. When I suggested that this was incomplete, he seemd 
to feel that I was asking him an unfair question because that sort of math 
problem was what he had studied 4 years ago and it was unreasonable for me to 
expect him to remember things from a long ago. --I have heard similar stories 
from colleagues in other schools.

 
Give her some simple word problems and allow her to solve them by trying out 
values. (Algebra teachers disallow this approach --for the seemingly good 
reason that it shortcircuits the learning of algebra) but trying out values 
allows the student to develop intuitions about how a problem works. Tell your 
granddaughter to make a note of any patterns she sees in the different numbers 
she tries. When she can detect patterns and express what they are coherently, 
that is the time to teach her algebraic notation.

--John

From: Friam [friam-boun...@redfish.com] on behalf of Nick Thompson 
[nickthomp...@earthlink.net]
Sent: Tuesday, March 29, 2016 10:42 PM
To: 'The Friday Morning Applied Complexity Coffee Group'
Subject: Re: [FRIAM] Calculus for 9 year olds

Thanks, Russ,

I appreciate the help.

Myself, I never got the "primary directive" of the calculus, or whatever it
is called (that integration is the inverse of differentiation) until I
graphed it.

I hope you are well,

Nick

Nicholas S. Thompson
Emeritus Professor of Psychology and Biology
Clark University
http://home.earthlink.net/~nickthompson/naturaldesigns/


-Original Message-
From: Friam [mailto:friam-boun...@redfish.com] On Behalf Of Russell Standish
Sent: Tuesday, March 29, 2016 4:45 PM
To: Friam 
Subject: Re: [FRIAM] Calculus for 9 year olds

On Tue, Mar 29, 2016 at 04:15:25PM -0600, Nick Thompson wrote:
> Hi, everybody,
>
>
>
> I have a granddaughter on vacation who is showing some interest in maths.
> We have been fooling around with graph paper, you know, "the squaw
> upon the hippopotamus is equal to the suns of the squaw's on the other
> two hides", etc., and playing race track on graph paper (which didn't
> grab her (used squares that were too small) but that's about all I have in
my repertoire.
>
>
>
> Any suggestions for really nifty stuff on the web (or that I could
> learn from the web quick enough) for 9 year olds.  I;ve been told that
> early childhood is the best time to teach calculus, but not by anybody
> who actually knew how to do it.  She is quick on a computer.
>

Not sure about the web, but you would need to get in algebra first. A bright
9yo should easily be able to handle the concept that letters can stand
abstractly for a number. Lack of algebra prevent the ancient Greeks from
getting calculus. I'd avoid trig, though, it's not necessary for getting the
concepts of differentiation and integration (unless Norm Wildberger's
approach helps?).

Then once you have algebra to hand, you need to teach the concept of limits.
eg If x->0 and y->0 twice as fast, what is the limit of y/x?
The answer is 1/2, not 0/0.

With limits and algebra on hand, you can tackle differentiation and
integration of polynomial functions. If she's any good at computer
programming (eg perhaps using Scratch or Alice*), then get her to write a
program printing out the value of something like (x+1)/x as
x->infinity. Its a really good way (IMHO) of grokking limits. Then you
can write a program to estimate the area of some random 

Re: [FRIAM] Subjectivity and cartoons

2016-03-04 Thread John Kennison
Eric,
Why would you ask about the pain IN the video? Shouldn't the person reply. "I 
don't believe in pain IN anything because, for me, pain is not internal it is 
external."

Nick,
That was a neat way of touching there square root of 2. If we changed it to the 
cube root of 2, we have a classic unsolved problem of Ancient Geek geometry 
--Duplicating the Cube: "To construct, with ruler and compasses, the length of 
the side of a cube which has twice the volume of a given cube". It has been 
proven (to the satisfaction of mathematicians)  that this is impossible.

Nick,
How can a person learn when he is hungry by observing other people? Perhaps he 
can recognize socially induced hunger (as in "at a party, we expect food") but 
do people usually detect low blood sugar in themselves  by observing the 
behavior of others? 


From: Friam [friam-boun...@redfish.com] on behalf of Eric Charles 
[eric.phillip.char...@gmail.com]
Sent: Friday, March 04, 2016 9:23 AM
To: The Friday Morning Applied Complexity Coffee Group
Subject: Re: [FRIAM] Subjectivity and cartoons

I'm not sure what to make of the cartoon comment either. Let's say we all agree 
that a person in front of us is in pain. Let's say we video tape that person 
and show it to someone else.

We ask our viewer afterwards "Did you see pain in that video?"

And he say "Yes."

Then we say, "Wait, you mean to tell me that those flat, pixilated colors on 
the screen were in pain?"

"No, no," they insist, "the person you video taped was in pain, the image 
itself wasn't!"

"But we asked you about the video," we assert confidently, "we want to know if 
there is pain in the video."

"Well, look... this is getting weird," he replies, "I'm leaving."

I kind of feel like we would end up in the same place if we tried to have a 
serious discussion about cartoons.

It is not, in Nick's position, an issue of a "sufficiently convincing 
performance." Certainly one can be fooled by people through various means, so 
we don't even need robots for that discussion. When we say, of a person that 
they gave a "convincing performance" what we mean is something like "When you 
look at a wider swath of that guy's behavior, you find that the chunk of 
behavior you originally studied is part of a very different pattern than you 
had originally assumed."

For example, a person who looks terribly dejected on a street corner holding a 
sign that speaks of their woes, but if you watch when they leave their post, 
they travel back to a perfectly middle-class house, change into nice clean 
clothes, and go about a normal life. That would be a "convincing performance." 
Note that we can speculate about whether it is a performance based on much less 
than that. Ultimately, however, we become certain it was "a performance" only 
by observing a larger swath of the world.





---
Eric P. Charles, Ph.D.
Lab Manager
Center for Teaching, Research, and Learning
American University, Hurst Hall Room 203A
4400 Massachusetts Avenue, N.W.
Washington, DC 20016
phone: (202) 885-3867   fax: (202) 885-1190
email: echar...@american.edu

On Fri, Mar 4, 2016 at 1:49 AM, Nick Thompson 
> wrote:
Russ,

I am torn between judging your cartoon comment as silly or profound.

I have said that if a robot could be devised that was embedded in a social 
network of other robots, that systematically avoided injurious events and 
stimuli, that engaged in some communicative behavior when injured to which 
other robots responded by coming to its rescue, then I would have to entertain 
the notion that these robots experience pain.  To me, pain is all of that.  
Hard to imagine a cartoon doing that.  Hence my first judgment that the idea is 
frivolous.  (But probably not a lot more frivolous than my idea that motivation 
is like the first derivative of behavior.)   I think perhaps the comment 
confuses the map with the territory, as Bateson used to say.

So, now I am stuck with trying to figure out why I might possibly think it 
profound.  But let’s make the example as favorable to your case as we can.  Let 
it be the case that you experience me being horrible tortured by the CIA.  Do 
you experience pain.  If you are not a psychopath, probably yes.  Do you 
experience MY pain.  No, because my pain occurs against an entirely different 
history of experiences, including, by the way, the occlusion of my airway by 
the wet washcloth and the poured water. .

Something like that.

Nick





Nicholas S. Thompson
Emeritus Professor of Psychology and Biology
Clark University
http://home.earthlink.net/~nickthompson/naturaldesigns/

From: Friam 
[mailto:friam-boun...@redfish.com] On Behalf 
Of Russ Abbott
Sent: Thursday, March 03, 2016 10:12 PM
To: The Friday Morning Applied Complexity Coffee Group 
>
Subject: [FRIAM] 

Re: [FRIAM] Subjectivity and intimacy (lost in the weeks?)

2016-02-29 Thread John Kennison
Hi, Russ, Eric, Nick, et al,

I hope you will humor me and carry out my request to help me get a clearer 
sense of your exchanges. Below, I present a series of observations about a man 
who is driving a car followed by a conclusion that one might draw. I am asking 
each of you whether the conclusion might be considered a scientific conclusion. 
Moreover, which of the statements in the "series of observations" are: 
(1) Scientifically meaningful.
(2) Might be scientifically meaningful if some of the terms were further 
defined.
(3) Are not scientifically meaningful.
I would also like each of you to explain the basis for your answers.

Then I would ask each of you whether there is a valid non-scientific way of 
reaching the same conclusion and to discuss this briefly.

Observations
Suppose it is reported that a person driving a car looks tense and frustrated. 
His blood pressure is well above what is normal for him. He has driven along 
the same stretch of road three times in the last 10 minutes. He pulls over and 
consults a map. Then he calms down. His blood pressure returns to a normal 
level. He drives on and, over the next two hours, he does not drive along that 
stretch of road again.

Conclusions
Based on this information, it is concluded that it is likely that there was a 
destination point he was trying to reach. He was frustrated because he was not 
making progress in reaching the destination. By consulting the map, he was able 
to find his way to the desired point. Moreover, he retained a visual image of 
the map and this image helped him find his way. 



From: Friam [friam-boun...@redfish.com] on behalf of Russ Abbott 
[russ.abb...@gmail.com]
Sent: Sunday, February 28, 2016 10:42 PM
To: The Friday Morning Applied Complexity Coffee Group
Subject: Re: [FRIAM] Subjectivity and intimacy (lost in the weeks?)

Eric: Now, whatever you are experiencing, you are experiencing it as somehow 
akin to a visual experience or, presumably, you wouldn't be using such terms, 
right?

Russ: When I use terms like "experiencing" or "visual experience" you seem to 
say that my language lacks meaning. You just used those terms. What do you mean 
by them?

On Sun, Feb 28, 2016 at 3:44 PM Eric Charles 
> wrote:
Frank,
When someone is exactly asking me about speaking without moving their lips, I'm 
not sure how it is a fatal flaw if I reply back that I would describe it as 
"speaking without moving you lips."

At any rate, like many Skinnerian descriptions, the claim is a developmental 
one. It is a claim about what you would see a kid learning if you started 
watching them when they only counted out loud, and kept watching until the 
counted "in their head." Having kids fairly close to that transition, it seemed 
like a pretty good description. They also learned to work things out "in their 
head" while looking at number lines they had previously needed to touch with 
their fingers. The developmental trajectory of ever-smaller bodily movements 
seemed to play out in that scenario as well.

As many times as I have heard cognitivist poo poo that explanation, I have 
never heard an alternative developmental explanation offered. Do let me know if 
you have one to offer.
And no, some hand waving at what a 4 year old is capable of, then what a 5 year 
old is capable of, then what a 6 year old is capable of, is not a developmental 
explanation. The laying out of such a sequence is just a clever way to avoid 
saying what the actual process of development is. :- )


Best,
Eric




---
Eric P. Charles, Ph.D.
Lab Manager
Center for Teaching, Research, and Learning
American University, Hurst Hall Room 203A
4400 Massachusetts Avenue, N.W.
Washington, DC 20016
phone: (202) 885-3867   fax: (202) 885-1190
email: echar...@american.edu

On Sat, Feb 27, 2016 at 2:24 PM, Frank Wimberly 
> wrote:
Russ,

In 1967 I took a course in cognitive processes at Carnegie Mellon.  One aspect 
of that course could be called “what’s wrong with behaviorism?”  At one point 
it was said, “When behaviorists talk about ‘sub-vocal speech’ you have won the 
argument.”

The “hard problem” is hard.

Nick and I have been arguing (in a friendly way) about these issues for years.  
There’s the “rabbit hole”.

In my opinion and for what it’s worth.

Frank


Frank C. Wimberly
140 Calle Ojo Feliz
Santa Fe, NM 87505

wimber...@gmail.com 
wimbe...@cal.berkeley.edu
Phone:  (505) 995-8715  Cell:  (505) 
670-9918

From: Friam 
[mailto:friam-boun...@redfish.com] On Behalf 
Of Eric Charles
Sent: Saturday, February 27, 2016 11:58 AM
To: The Friday Morning Applied Complexity Coffee Group
Subject: Re: [FRIAM] Subjectivity and 

Re: [FRIAM] Weeds of pragmatism: Subjectivity and intimacy

2016-02-25 Thread John Kennison
out
Charlemagne's breakfast menu.

The point you raise about transubstantiation is I think one that neatly
divides pragmatists.  "If it makes a difference in the behavior of
Christians, the assertion, 'This is the blood and body of christ' has
meaning" is I think the more Jamesian view.  The more Peircean view would
be, I think, "On sober examination by humans examining their experiences
with the substances offered in communion, and the behavior of humans with
respect to the substances, we would have to conclude not only that they were
wine and a wafer, but that everybody in the room KNEW they were was wine and
a wafer.  Who, after all, would drink raw blood that had been left out on
the counter all night?  Hence, the ritual assertion of the nature of the
wine and wafer is nonsense.

As always, I hope the Peirce experts on the list will correct me.  There are
at least two who have not spoken yet.

Nick

Nicholas S. Thompson
Emeritus Professor of Psychology and Biology
Clark University
http://home.earthlink.net/~nickthompson/naturaldesigns/

-Original Message-
From: Friam 
[mailto:friam-boun...@redfish.com<mailto:friam-boun...@redfish.com>] On Behalf 
Of John Kennison
Sent: Wednesday, February 24, 2016 12:52 PM
To: The Friday Morning Applied Complexity Coffee Group 
<friam@redfish.com<mailto:friam@redfish.com>>
Subject: Re: [FRIAM] Weeds of pragmatism: Subjectivity and intimacy

Eric and Nick,

The issue of whether Charlemagne ate eggs for breakfast is not the question
I am raising, it is only an illustration of my question. My actual question
(as I now understand it) is whether there is a reality to what did and what
did not happen in the past that is independent of what we can figure out
scientifically (which, I think, only addresses the issue of what probably
happened). This is a real question with real consequences, even though I
chose to illustrate with a question that is, admittedly, trivial. (But which
I thought was so simple it would make my meaning clear.)

I liked the example of transubstantiation vs consubstantiation and agree
that it was pretty silly (even though lots of Christians killed other
Christians because they disagreed).

Wait a second, does that mean it is a real issue because it clearly had
consequences? Perhaps I should say that people were killed because they said
they disagreed.

--John


From: Friam [friam-boun...@redfish.com<mailto:friam-boun...@redfish.com>] on 
behalf of Nick Thompson
[nickthomp...@earthlink.net<mailto:nickthomp...@earthlink.net>]
Sent: Wednesday, February 24, 2016 1:55 PM
To: 'The Friday Morning Applied Complexity Coffee Group'; 'M. D. Bybee';
'John Shook'
Subject: Re: [FRIAM] Weeds of pragmatism: Subjectivity and intimacy

John,

Below, reforwarded, is the message from Eric that I so enthusiastically
endorsed.

To be completely honest, I have never quite gotten the old Peirce  / young
Peirce thing.  I know it's a thing because many wise people have told me
about it.  And there are areas of Peirce's thought  where I think I do
detect a change . say with "abduction", which seems to begin as affirming
the consequent and end up as "inspired guessing".  In that case, I like
Eric, prefer the early Peirce.

As to the rest, I cannot speak and, as the philosopher said, "Of that about
which I cannot speak, I should remain silent."

Nicholas S. Thompson
Emeritus Professor of Psychology and Biology Clark University
http://home.earthlink.net/~nickthompson/naturaldesigns/

From: Friam 
[mailto:friam-boun...@redfish.com<mailto:friam-boun...@redfish.com>] On Behalf 
Of Eric Charles
Sent: Tuesday, February 23, 2016 9:55 AM
To: The Friday Morning Applied Complexity Coffee Group 
<friam@redfish.com<mailto:friam@redfish.com>>;
M. D. Bybee <mikeby...@earthlink.net<mailto:mikeby...@earthlink.net>>; John 
Shook <jsh...@pragmatism.org<mailto:jsh...@pragmatism.org>>
Subject: Re: [FRIAM] Weeds of pragmatism: Subjectivity and intimacy

Nick,
I worked hard on phrasing those parts ;- ) The first part is specifically
worded to avoid the red flags that pragmatism-talk might throw up for some
people... I don't think an argument about the psychology that flows from
pragmatism need obligate first discussing Pierce's philosophy of science.
But we can certainly do it that way if you want.

Regarding the philosophy of science: Peirce would say that it is startling
that investigations about anything leads to convergence of agreement amongst
serious investigators. It is borderline miraculous, he asserts, and we must
latch onto such situations with tenacity whenever we should encounter them.
We would have no reason, were we blank slates, to assume that worlds should
contain such consistencies regarding any things that might be investigated.
Indeed, were we interested in the past, we would find that the past is
dominated by earnest investigators 

Re: [FRIAM] Weeds of pragmatism: Subjectivity and intimacy

2016-02-24 Thread John Kennison
Glen,

What you said now looks very good to me.

--John

From: Friam [friam-boun...@redfish.com] on behalf of glen [geprope...@gmail.com]
Sent: Wednesday, February 24, 2016 3:20 PM
To: The Friday Morning Applied Complexity Coffee Group
Subject: Re: [FRIAM] Weeds of pragmatism: Subjectivity and intimacy

This implies what I tried to state explicitly with the idea of 0th, 1st, 2nd 
... order science.  It's only indirectly related to the concept of probability 
of truth[†].  0th order would be an approachable, extant, limit ... aka a 
"fact".  1st order would be the inferences (by whatever inferential method you 
choose) you can make from facts.  2nd order would be inferences you can make 
from both 0th and 1st order sentences.  Etc. This sort of structure handles 
both likelihood and cumulative consequences.  But it depends fundamentally on 
the structures of the inferential methods at and between orders.

[†] A probability some 0th order fact obtains is a kind of "flattening" or 
projection of all the other ith order sentences down to 0th order (perhaps a 
0th order with a different structure than the 0th order used to establish the 
initial facts).

On 02/24/2016 11:52 AM, John Kennison wrote:
> The issue of whether Charlemagne ate eggs for breakfast is not the question I 
> am raising, it is only an illustration of my question. My actual question (as 
> I now understand it) is whether there is a reality to what did and what did 
> not happen in the past that is independent of what we can figure out 
> scientifically (which, I think, only addresses the issue of what probably 
> happened). This is a real question with real consequences, even though I 
> chose to illustrate with a question that is, admittedly, trivial. (But which 
> I thought was so simple it would make my meaning clear.)
>
> I liked the example of transubstantiation vs consubstantiation and agree that 
> it was pretty silly (even though lots of Christians killed other Christians 
> because they disagreed).
>
> Wait a second, does that mean it is a real issue because it clearly had 
> consequences? Perhaps I should say that people were killed because they said 
> they disagreed.


--
⇔ glen


FRIAM Applied Complexity Group listserv
Meets Fridays 9a-11:30 at cafe at St. John's College
to unsubscribe http://redfish.com/mailman/listinfo/friam_redfish.com


FRIAM Applied Complexity Group listserv
Meets Fridays 9a-11:30 at cafe at St. John's College
to unsubscribe http://redfish.com/mailman/listinfo/friam_redfish.com


Re: [FRIAM] Weeds of pragmatism: Subjectivity and intimacy

2016-02-24 Thread John Kennison
le" to be a 
hypothesis! It is akin to the failed hypothesis that we could make a science 
about bumbs on people's heads, and it is akin to the successful hypothesis that 
we could make a science of "atoms".
James (IMHO) understands the ramifications of this much better than Peirce 
does. If a science of psychology is possible, then the very foundation of 
western thinking about psychology must be thrown out? Why? Because the 
foundation declares a priori that such a science is impossible. Whatever 
combination of beliefs and logical inferences led those Big Names to declare 
such a science to be impossible, that amalgamation of ideas must be wrong. Of 
most obvious note, metaphysical dualism must be wrong. The mind cannot be 
something independent of the rest of the investigatable world.

But Peirce has been swayed too much by Kant to see how wrong Kant's views about 
psychology are. Kant takes the difficulties of doing ANY science, and acts as 
if they are only problems of doing psychology. It is a bum move.

If we are not a priori dualists, then we have no reason to believe that the 
problem of knowing another person's mind are any different than the problem of 
knowing anything else about the world.
Best,
Eric




---
Eric P. Charles, Ph.D.
Lab Manager
Center for Teaching, Research, and Learning
American University, Hurst Hall Room 203A
4400 Massachusetts Avenue, N.W.
Washington, DC 20016
phone: (202) 885-3867   fax: (202) 885-1190
email: echar...@american.edu<mailto:echar...@american.edu>

On Tue, Feb 23, 2016 at 2:51 AM, John Kennison 
<jkenni...@clarku.edu<mailto:jkenni...@clarku.edu>> wrote:
I don't know it I am following all this correctly, but I would like to apply it 
to the question of doing History scientifically. At the start all we have are 
relics from the past  --maybe we are uncertain which objects and/or documents 
really go back to a historical period under examination--but we have some way 
of testing for various relations between these relics. Se then look for a 
theory of the past which best accounts for the relics that we have. We may be 
able to measure how well different theories do this accounting. And the set opf 
measutres we arrive at is then history.

But would some historian be dualists if they say there is a real truth about 
what happened in the past, it's just that we this real truth may not be 
recoverable.



From: Friam [friam-boun...@redfish.com<mailto:friam-boun...@redfish.com>] on 
behalf of Nick Thompson 
[nickthomp...@earthlink.net<mailto:nickthomp...@earthlink.net>]
Sent: Tuesday, February 23, 2016 1:34 AM
To: 'The Friday Morning Applied Complexity Coffee Group'
Subject: Re: [FRIAM] Weeds of pragmatism: Subjectivity and intimacy

Eric,

I really like all of you said.  What follows is a cavil, and will problem 
interest most readers as much as the bickering among monks about how often to 
wash their hair shirts.   Please do not reply to this message unless you are 
interested in what follows.


ONLY PERSONS UNINTERESTED IN THE WEEDIEST THICKETS OF PRAGMATISM SHOULD READ 
BELOW THIS LINE

I think, Eric,  you left the door open for dualism, when you describe the 
settlement of scientific opinion, and I need to close it behind you.  You wrote:

3) In order to do science about something, we need only one thing to be true: 
It can be investigated empirically. That is, it is something, "out there" which 
we can turn our machinations towards, and which will yield stable results once 
we find the appropriate methods for its investigation.


Here you imply that science is possible because there is Truth out there, 
concerning which human experience is fated to converge.  The real world is 
somehow responsible for the convergence of opinion among scientists.   Note the 
subtle difference in the way you presented it only a few paragraphs later:

7) And that's where we find ourselves. If a science of psychology is possible, 
then de facto the subject matter of psychology is some swath of empirically 
investigatable happenings, about which a community of investigators would 
eventually reach a consensus as the scientific process takes its course.

Here, you focus exclusively on the convergence of opinion, eliminating 
altogether the notion of a truth outside human experience.  Peirce would say – 
and perhaps James would not agree – that  “that opinion upon which human 
scientists are fated to agree “ is exactly, and only, what is meant by the 
truth.  So, there is a truth “out there”, beyond what you, or I, or any other 
individuals might come to believe,  but not a truth beyond what all humans 
might come to believe.  All we know now is that those opinions which are 
enduring and coherent with other enduring opinions have the best mathematical 
chance of being those opinions upon which we will ultimately conv

Re: [FRIAM] History and subjectivity and intimacy

2016-02-24 Thread John Kennison
Hi Nick,

At one point you suggested I work on digesting some things that eric Charles 
said. The part that I think I understood was the distinction between Young 
Peirce and Old Peirce, That young Peirce would say  " 'Truth' as the scientist 
understands it does not come into it." while Old Peirce would say  " 'Truth' 
does not come into it". You sound like a Old Peirecian, Eric seems to like 
young Peirce  and I would prefer Young Peirce but long to go beyond that to 
assert that there are truths that are not scientifically comprehensible. 

As an example, suppose I am a night watchman for a building and at 10pm I hear 
a funny sound in the basement. It occurs to me that I can check the continuous 
video surveillance to see if it caught anything that might have produced the 
sound. So the question is: "Did the video record anything that might have 
produced the funny sound?" . The question makes sense because I can resolve it 
by viewing the video. But further suppose that I get involved in helping 
someone in need (for example, a person stuck in the elevator. When I have 
finished doing that, I realize the video is on a continuous loop which erases 
over and reuses the tape unless  new tape is installed. Now there is no longer 
a way to ascertain whether the (old) video recorded anything that might have 
produced the funny noise. It seems to say that a question about the past may 
once have been a reasonable question but later was no longer a legitimate 
question. Why can't I ask whether the video, at one time, caught a relevant im
 age?  




From: Friam [friam-boun...@redfish.com] on behalf of Nick Thompson 
[nickthomp...@earthlink.net]
Sent: Tuesday, February 23, 2016 10:38 PM
To: 'The Friday Morning Applied Complexity Coffee Group'
Subject: Re: [FRIAM] History and subjectivity and intimacy

Hi, John,

Not sure Peirce would go with "truths".  I think he regarded scientific
explanation as the formalization of a universal human cognitive capacity and
therefore we are all fated to believe what scientists are fated to believe.
Truth is what we will all come to believe in the very long run.  Others may
correct me on this.  But notice how we are proceeding -- as if there is some
state of belief with respect to Peirce's beliefs that will, in time, settle
the matter.   If there is such a settling belief, than there is, by
definition, a truth; if there is no such ultimate settling of  belief, than
there is no truth of the matter.  Take note of Eric's observation that,
according to Peirce, order in experience is kind of a miracle.  A rarity in
a world that is essentially random.  We are still in the weeds, here.

Nick

Nicholas S. Thompson
Emeritus Professor of Psychology and Biology
Clark University
http://home.earthlink.net/~nickthompson/naturaldesigns/


-Original Message-
From: Friam [mailto:friam-boun...@redfish.com] On Behalf Of John Kennison
Sent: Tuesday, February 23, 2016 5:33 PM
To: The Friday Morning Applied Complexity Coffee Group <friam@redfish.com>
Subject: Re: [FRIAM] History and subjectivity and intimacy

Hi Eric,

Thanks for responding. I didn't follow all that you said, but I like what
young Peirce (who might have said)  "Truth as the scientist understands it"
--and therefore seems to allow for a different kind of truth. And I could
have chosen a better example --say the dispute between Bush and Gore about
who really won the crucial state of Florida. There the truth didn't seem to
affect the decision, but if the  court had been divided 4-4 with 1
independent, it might have. The courts do have ways of deciding cases based
on truth as the scientist understands it --that is clearly the only way the
judicial system can function --but I gather that Old Peirce would say there
is no other truth. Am I under a delusion that there may be actual truths
about the past that are simply lost?

--John

From: Friam [friam-boun...@redfish.com] on behalf of Eric Charles
[eric.phillip.char...@gmail.com]
Sent: Tuesday, February 23, 2016 3:44 PM
To: The Friday Morning Applied Complexity Coffee Group; M. D. Bybee; John
Shook
Subject: Re: [FRIAM] History and subjectivity and intimacy

Trying to bring things to this thread... John said:
The crucial questions then, are do you see any talk about "what really
happened in the past" as some sort of delusion? Would you go one step
further and say the concept that there is a truth about what happened in the
past is delusional, for example "We have no way of knowing for certain
whether Charlemagne ate eggs on a particulr day during his life(Say January
1, 800 ad) but there is a truth about the matter (either he did or did not)
even though it is a truth we can never fully determine?

I think Peirce would try to dissolve this controversy . He would say, I
think, that the question "Did Charlemagne eat e

Re: [FRIAM] History and subjectivity and intimacy

2016-02-23 Thread John Kennison
 should, but others shouldn't. Let me lay 
out the problem:

We wonder whether a given baseball is the one thrown for a particular strike 
during a particular game. What unique properties would such a ball have? There 
are two types of answers we might seek.

One is a unique configuration of differences, i.e., the ball had a mixture of 
resin and spit used by a particular pitcher to doctor their pitch, and was the 
only one of its kind to have been made at the factory with a one-off improper 
spelling of the company name. In such a case we may apply the scientific 
process to determine if the ball in question has those properties, and 
(potentially) determine the extent to which those properties are actually 
unique. That is, we could test through empirical method, all consequences of 
the claim that the ball in question is The Ball.

The other type of answer we might seek is via "providence"; i.e., there is 
nothing unique about the ball itself, but there is other evidence it might be 
of historic importance. We might have a picture of the ump tossing a ball to a 
particular kid, record of that kid giving it over to an auction house several 
years later, that auction house selling it to Rich Guy A, who owned a storage 
unit, in which Rich Guy B found a ball (after A's death) along with the 
paperwork referenced and associated photos. Here we could test, through the 
empirical method, all sorts of claims about the providence itself (was it 
printed on the right type of paper, does the signature look authentic, etc.) 
but there is nothing to test about the ball pre se.

In either case it is unclear that investigating history would resemble making a 
scientific claim, because scientific claims are about generalities, not 
individual events. To claim that vinegar will dissolve calcium build ups is a 
very different type of thing than to claim that this ball is The Ball. The 
science above is not direct at the veracity of the historical claims, but 
rather it is directed at some things that would be true about the objects in 
front of us, were those historic claims true.

Even worse, I think Peirce would assert that unless there is something about 
the ball itself that is in question, the question of whether it is The Ball is 
nonsensical.

Whew! I hope some of that was relevant.

Best,
Eric



---
Eric P. Charles, Ph.D.
Lab Manager
Center for Teaching, Research, and Learning
American University, Hurst Hall Room 203A
4400 Massachusetts Avenue, N.W.
Washington, DC 20016
phone: (202) 885-3867<tel:%28202%29%20885-3867>   fax: (202) 
885-1190<tel:%28202%29%20885-1190>
email: echar...@american.edu<mailto:echar...@american.edu>

On Tue, Feb 23, 2016 at 2:51 AM, John Kennison 
<jkenni...@clarku.edu<mailto:jkenni...@clarku.edu>> wrote:
I don't know it I am following all this correctly, but I would like to apply it 
to the question of doing History scientifically. At the start all we have are 
relics from the past  --maybe we are uncertain which objects and/or documents 
really go back to a historical period under examination--but we have some way 
of testing for various relations between these relics. Se then look for a 
theory of the past which best accounts for the relics that we have. We may be 
able to measure how well different theories do this accounting. And the set opf 
measutres we arrive at is then history.

But would some historian be dualists if they say there is a real truth about 
what happened in the past, it's just that we this real truth may not be 
recoverable.




FRIAM Applied Complexity Group listserv
Meets Fridays 9a-11:30 at cafe at St. John's College
to unsubscribe http://redfish.com/mailman/listinfo/friam_redfish.com


Re: [FRIAM] Weeds of pragmatism: Subjectivity and intimacy

2016-02-23 Thread John Kennison
Hi Nick,

Thanks for liking my metaphor. In my previous statement I should have noted 
that "dualism" for a historian is the analog of dualism for a psychologist 
studying conscioisness. You seem to rejerct both. The crucial questions then, 
are do you see any talk about "what really happened in the past" as some sort 
of delusion? Would you go one step further and say the concept that there is a 
truth about what happened in the past is delusional, for example "We have no 
way of knowing for certain whether Charlemagne ate eggs on a particulr day 
during his life(Say January 1, 800 ad) but there is a truth about the matter 
(either he did or did not) even though it is a truth we can never fully 
determine?

--John

From: Friam [friam-boun...@redfish.com] on behalf of Nick Thompson 
[nickthomp...@earthlink.net]
Sent: Tuesday, February 23, 2016 1:56 PM
To: 'The Friday Morning Applied Complexity Coffee Group'
Subject: Re: [FRIAM] Weeds of pragmatism: Subjectivity and intimacy

Hi, John,

Welcome to the Weeds!

I like the history metaphor.  What I think Peirce would say is everything you 
describe is experience NOW.  Some of those experiences NOW get understood as 
experiences THEN.  So, an "experience-then" is just a way of organizing some of 
the "experiences-now".  There is no experience beyond experience.  Put baldly, 
that sounds like nothing more than a trivial tautology.   For me, it is a 
chastening reminder that any knowledge I assert beyond experience is either (1) 
nonsense or (2) a statement about some experiences.

I know my Peirce-guru occasionally lurks, here, and I hope he will correct me.  
 In particular, I wish he could remind me why I care about this enough even to 
devote a sub thread to it.

Nick

Nicholas S. Thompson
Emeritus Professor of Psychology and Biology
Clark University
http://home.earthlink.net/~nickthompson/naturaldesigns/


-Original Message-
From: Friam [mailto:friam-boun...@redfish.com] On Behalf Of John Kennison
Sent: Tuesday, February 23, 2016 12:51 AM
To: The Friday Morning Applied Complexity Coffee Group <friam@redfish.com>
Subject: Re: [FRIAM] Weeds of pragmatism: Subjectivity and intimacy

I don't know it I am following all this correctly, but I would like to apply it 
to the question of doing History scientifically. At the start all we have are 
relics from the past  --maybe we are uncertain which objects and/or documents 
really go back to a historical period under examination--but we have some way 
of testing for various relations between these relics. Se then look for a 
theory of the past which best accounts for the relics that we have. We may be 
able to measure how well different theories do this accounting. And the set opf 
measutres we arrive at is then history.

But would some historian be dualists if they say there is a real truth about 
what happened in the past, it's just that we this real truth may not be 
recoverable.



From: Friam [friam-boun...@redfish.com] on behalf of Nick Thompson 
[nickthomp...@earthlink.net]
Sent: Tuesday, February 23, 2016 1:34 AM
To: 'The Friday Morning Applied Complexity Coffee Group'
Subject: Re: [FRIAM] Weeds of pragmatism: Subjectivity and intimacy

Eric,

I really like all of you said.  What follows is a cavil, and will problem 
interest most readers as much as the bickering among monks about how often to 
wash their hair shirts.   Please do not reply to this message unless you are 
interested in what follows.


ONLY PERSONS UNINTERESTED IN THE WEEDIEST THICKETS OF PRAGMATISM SHOULD READ 
BELOW THIS LINE  I think, Eric,  you left the 
door open for dualism, when you describe the settlement of scientific opinion, 
and I need to close it behind you.  You wrote:

3) In order to do science about something, we need only one thing to be true: 
It can be investigated empirically. That is, it is something, "out there" which 
we can turn our machinations towards, and which will yield stable results once 
we find the appropriate methods for its investigation.


Here you imply that science is possible because there is Truth out there, 
concerning which human experience is fated to converge.  The real world is 
somehow responsible for the convergence of opinion among scientists.   Note the 
subtle difference in the way you presented it only a few paragraphs later:

7) And that's where we find ourselves. If a science of psychology is possible, 
then de facto the subject matter of psychology is some swath of empirically 
investigatable happenings, about which a community of investigators would 
eventually reach a consensus as the scientific process takes its course.

Here, you focus exclusively on the convergence of opinion, eliminating 
altogether the notion of a truth outside human experience.  Peirce would say – 
and p

Re: [FRIAM] Weeds of pragmatism: Subjectivity and intimacy

2016-02-22 Thread John Kennison
I don't know it I am following all this correctly, but I would like to apply it 
to the question of doing History scientifically. At the start all we have are 
relics from the past  --maybe we are uncertain which objects and/or documents 
really go back to a historical period under examination--but we have some way 
of testing for various relations between these relics. Se then look for a 
theory of the past which best accounts for the relics that we have. We may be 
able to measure how well different theories do this accounting. And the set opf 
measutres we arrive at is then history.

But would some historian be dualists if they say there is a real truth about 
what happened in the past, it's just that we this real truth may not be 
recoverable.



From: Friam [friam-boun...@redfish.com] on behalf of Nick Thompson 
[nickthomp...@earthlink.net]
Sent: Tuesday, February 23, 2016 1:34 AM
To: 'The Friday Morning Applied Complexity Coffee Group'
Subject: Re: [FRIAM] Weeds of pragmatism: Subjectivity and intimacy

Eric,

I really like all of you said.  What follows is a cavil, and will problem 
interest most readers as much as the bickering among monks about how often to 
wash their hair shirts.   Please do not reply to this message unless you are 
interested in what follows.


ONLY PERSONS UNINTERESTED IN THE WEEDIEST THICKETS OF PRAGMATISM SHOULD READ 
BELOW THIS LINE

I think, Eric,  you left the door open for dualism, when you describe the 
settlement of scientific opinion, and I need to close it behind you.  You wrote:

3) In order to do science about something, we need only one thing to be true: 
It can be investigated empirically. That is, it is something, "out there" which 
we can turn our machinations towards, and which will yield stable results once 
we find the appropriate methods for its investigation.


Here you imply that science is possible because there is Truth out there, 
concerning which human experience is fated to converge.  The real world is 
somehow responsible for the convergence of opinion among scientists.   Note the 
subtle difference in the way you presented it only a few paragraphs later:

7) And that's where we find ourselves. If a science of psychology is possible, 
then de facto the subject matter of psychology is some swath of empirically 
investigatable happenings, about which a community of investigators would 
eventually reach a consensus as the scientific process takes its course.

Here, you focus exclusively on the convergence of opinion, eliminating 
altogether the notion of a truth outside human experience.  Peirce would say – 
and perhaps James would not agree – that  “that opinion upon which human 
scientists are fated to agree “ is exactly, and only, what is meant by the 
truth.  So, there is a truth “out there”, beyond what you, or I, or any other 
individuals might come to believe,  but not a truth beyond what all humans 
might come to believe.  All we know now is that those opinions which are 
enduring and coherent with other enduring opinions have the best mathematical 
chance of being those opinions upon which we will ultimately converge.  Lets 
say we are in a group of geographers wandering in a blizzard.  We are 
completely disoriented and we have no consensus concerning what is the right 
direction home.  Some propose going down hill, some up hill, some following the 
slope to the right or left.  What is the function of “home” in our discussion.  
It is the place which, when we get there, we will terminate the discussion of 
where it is.

To all intents and purposes, this distinction is monk’s work.  The kind of 
question only true believers in pragmatism could trouble themselves with.  For 
the rest of you, who need to get on with your work of building bridges and 
electing politicians, you need only say to yourselves (quietly, please, so 
Peirce will not hear you), “the REASON that scientists converge on some opinion 
is that there is something outside the world of human experience that beckons 
them toward it.  Some Truth.  But that is dualism, and Peirce never would have 
tolerated it.

SEE.  I TOLD YOU THIS WOULD BORE YOU!

If you want to continue the previous conversation, but don’t want to go into 
the Weeds Of Pragmatism with me  and Eric, I suggest you reply to an earlier 
message, not to this one.

Nick


Nicholas S. Thompson
Emeritus Professor of Psychology and Biology
Clark University
http://home.earthlink.net/~nickthompson/naturaldesigns/

From: Friam [mailto:friam-boun...@redfish.com] On Behalf Of Eric Charles
Sent: Monday, February 22, 2016 8:09 PM
To: The Friday Morning Applied Complexity Coffee Group 
Subject: Re: [FRIAM] Subjectivity and intimacy

Russ said: "Eric's point that the world must be a certain way if we are to do 
science doesn't make sense to me. 

Re: [FRIAM] Subjectivity and intimacy

2016-02-22 Thread John Kennison
rofessor of Psychology and Biology
Clark University
http://home.earthlink.net/~nickthompson/naturaldesigns/

From: Friam [mailto:friam-boun...@redfish.com] On Behalf Of Russ Abbott
Sent: Friday, February 19, 2016 10:33 PM
To: The Friday Morning Applied Complexity Coffee Group 
<friam@redfish.com<mailto:friam@redfish.com>>
Subject: Re: [FRIAM] Subjectivity and intimacy

Intimacy is not necessarily about sex, but it is also not about just about 
knowing something about someone that isn't generally known, e.g., where the 
person went to elementary school or her mother's maiden name. It's more than 
just being able to answer the sorts of questions web sites ask as a way to 
establish one's identity. Intimacy has to do with the kinds of things that are 
known, in particular with knowing about the subjective experience of another 
person. At least that's how I would describe it -- and that's why I raised the 
question.

On Fri, Feb 19, 2016 at 3:39 PM Nick Thompson 
<nickthomp...@earthlink.net<mailto:nickthomp...@earthlink.net>> wrote:

Dear John and Russ,



Well, you question is an example of itself.  Who is best qualified to explain 
the basis of Nick's denial of subjectivity?  Is this a question about etiology: 
I.e., the causal history of Nick's coming to deny subjectivity?  Or is it a 
question of what rational arguments Nick might make for his denial of 
subjectivity.  Note that there is nothing particularly private about either of 
those forms of the question.  FRIAM could get to work on answering them and 
Nick could stand aside and wonder at the quality and perspicacity of your 
answers.  My own most recent and condensed and approachable attempt to answer 
both versions of the question can be found in the manuscript that is attached.  
I can’t find cc of the published vsn at the moment.



I will think about the intimacy issue.  I think it’s about having some others 
who know things about you that are not generally known.  I would argue that 
when you get into bed with somebody naked, it’s a metaphor.  But then, I am old.



Nick



Nicholas S. Thompson

Emeritus Professor of Psychology and Biology

Clark University

http://home.earthlink.net/~nickthompson/naturaldesigns/



-Original Message-
From: Friam 
[mailto:friam-boun...@redfish.com<mailto:friam-boun...@redfish.com>] On Behalf 
Of John Kennison
Sent: Friday, February 19, 2016 2:30 PM
To: The Friday Morning Applied Complexity Coffee Group 
<friam@redfish.com<mailto:friam@redfish.com>>
Subject: Re: [FRIAM] Subjectivity and intimacy



One thing I wonder about (or perhaps have forgotten) in this discussion and 
Nick's denial is what the denial is based on. Is the absence of subjectivity 
supposed to be a scientific fact? If so, we should be discussing the 
experimental foundations of this fact. I have read of some experiments which 
seem to indicate that subjectiviity is not exactly what we (or what I) used to 
think it is --but which do not seem to disprove subjectivity.



--John



From: Friam [friam-boun...@redfish.com<mailto:friam-boun...@redfish.com>] on 
behalf of Russ Abbott [russ.abb...@gmail.com<mailto:russ.abb...@gmail.com>]

Sent: Friday, February 19, 2016 3:27 PM

To: The Friday Morning Applied Complexity Coffee Group

Subject: [FRIAM] Subjectivity and intimacy



We've had discussions on and off about subjectivity -- with me getting 
frustrated at Nick's denial thereof (if I understood him correctly).



It occurred to me recently that intimacy is defined -- as I understand it -- in 
terms of subjectivity, i.e., the sharing of one's (most private) subjective 
experiences with another.



I'm wondering what Nick thinks about this and whether anyone else has something 
to say about it. In particular, if there is no such thing as subjective 
experience, does that imply in your view that the same goes for intimacy?





FRIAM Applied Complexity Group listserv

Meets Fridays 9a-11:30 at cafe at St. John's College to unsubscribe 
http://redfish.com/mailman/listinfo/friam_redfish.com

FRIAM Applied Complexity Group listserv
Meets Fridays 9a-11:30 at cafe at St. John's College
to unsubscribe http://redfish.com/mailman/listinfo/friam_redfish.com


Nicholas S. Thompson
Emeritus Professor of Psychology and Biology
Clark University
http://home.earthlink.net/~nickthompson/naturaldesigns/

From: Friam 
[mailto:friam-boun...@redfish.com<mailto:friam-boun...@redfish.com>] On Behalf 
Of Russ Abbott
Sent: Friday, February 19, 2016 10:33 PM
To: The Friday Morning Applied Complexity Coffee Group 
<friam@redfish.com<mailto:friam@redfish.com>>
Subject: Re: [FRIAM] Subjectivity and intimacy

Intimacy is not necessarily about sex, but it is also not about just about 
knowing something about someone that isn't generally know

Re: [FRIAM] Subjectivity and intimacy

2016-02-20 Thread John Kennison
Hi Nick,

Yes, I do, of course, remember this, but I keep on thinking that something is 
being overlooked. Your writing seems fine to me --but I'll have to parse it and 
think about it before I give a more thoughtful response. 

Cheers, John

From: Friam [friam-boun...@redfish.com] on behalf of Nick Thompson 
[nickthomp...@earthlink.net]
Sent: Saturday, February 20, 2016 3:33 PM
To: 'The Friday Morning Applied Complexity Coffee Group'
Subject: Re: [FRIAM] Subjectivity and intimacy

John,

What I used to say, when you knew me back at Clark, is that MY consciousness
is the slice that is cut through the world by MY behavior.  This is the New
Realist view, laid out by Holt in the early 20th century.  A slightly
different way of saying this is that I am just (= am exactly, and only) a
point from which the world is seen.  You remember "I am an extentionless
dot."  It's laid out in the paper I attached.

Since coming to Santa Fe, I have come more under the influence of Peirce.
For Peirce, everything that is real arises from the stream of human
experience.  Each of us parses that stream into subject and object through
processes of inference.  What is subjective, on this account, is just as
much a cognitive achievement as what is objective.  What we take to be
"inner" is just features of our stream of experience that do not necessarily
coincide with the reported experience of others or future experiences of our
own.  Dreams and hallucinations are experiences that don't, in the long run,
pan out.  My private world is just those parts of my experience that, in
some sense, move with me.

I am sorry.  Those sentences only an author's mother could love.  Best a
toothache and hydrocodone would allow.

Nick

Nicholas S. Thompson
Emeritus Professor of Psychology and Biology
Clark University
http://home.earthlink.net/~nickthompson/naturaldesigns/


-Original Message-
From: Friam [mailto:friam-boun...@redfish.com] On Behalf Of John Kennison
Sent: Saturday, February 20, 2016 2:26 AM
To: The Friday Morning Applied Complexity Coffee Group <friam@redfish.com>
Subject: Re: [FRIAM] Subjectivity and intimacy


I think Russ is raising an important point.
It seems that Nick is saying that consciousness is something that is
external. But, assuming we accept that view, some of a person's
consciousness may be out there to only a small group of close friends. Could
this be what is called intimate knowledge of that consciousness?

--John

From: Friam [friam-boun...@redfish.com] on behalf of Russ Abbott
[russ.abb...@gmail.com]
Sent: Saturday, February 20, 2016 12:32 AM
To: The Friday Morning Applied Complexity Coffee Group
Subject: Re: [FRIAM] Subjectivity and intimacy

Intimacy is not necessarily about sex, but it is also not about just about
knowing something about someone that isn't generally known, e.g., where the
person went to elementary school or her mother's maiden name. It's more than
just being able to answer the sorts of questions web sites ask as a way to
establish one's identity. Intimacy has to do with the kinds of things that
are known, in particular with knowing about the subjective experience of
another person. At least that's how I would describe it -- and that's why I
raised the question.

On Fri, Feb 19, 2016 at 3:39 PM Nick Thompson
<nickthomp...@earthlink.net<mailto:nickthomp...@earthlink.net>> wrote:

Dear John and Russ,



Well, you question is an example of it self.  Who is best qualified to
explain the basis of Nick's denial of subjectivity?  Is this a question
about aetiology: I.e., the causal history of Nick's coming to deny
subjectivity?  Or is it a question of what rational arguments Nick might
make for his denial of subjectivity.  Note that there is nothing
particularly private about either of those forms of the question.  FRIAM
could get to work on answering them and Nick could stand aside and wonder at
the quality and perspicacity of your answers.  My own most recent and
condensed and approachable attempt to answer both versions of the question
can be found in the manuscript that is attached.  I can't find cc of the
published vsn at the moment.



I will think about the intimacy issue.  I think it's about having some
others who know things about you that are not generally known.  I would
argue that when you get into bed with somebody naked, it's a metaphor.  But
then, I am old.



Nick



Nicholas S. Thompson

Emeritus Professor of Psychology and Biology

Clark University

http://home.earthlink.net/~nickthompson/naturaldesigns/



-Original Message-
From: Friam
[mailto:friam-boun...@redfish.com<mailto:friam-boun...@redfish.com>] On
Behalf Of John Kennison
Sent: Friday, February 19, 2016 2:30 PM
To: The Friday Morning Applied Complexity Coffee Group
<friam@redfish.com<mailto:friam@redfish.com>>
Subject: Re: [FRIAM] Subjectivity and intimac

Re: [FRIAM] Subjectivity and intimacy

2016-02-20 Thread John Kennison

I think Russ is raising an important point.
It seems that Nick is saying that consciousness is something that is external. 
But, assuming we accept that view, some of a person's consciousness may be out 
there to only a small group of close friends. Could this be what is called 
intimate knowledge of that consciousness?

--John

From: Friam [friam-boun...@redfish.com] on behalf of Russ Abbott 
[russ.abb...@gmail.com]
Sent: Saturday, February 20, 2016 12:32 AM
To: The Friday Morning Applied Complexity Coffee Group
Subject: Re: [FRIAM] Subjectivity and intimacy

Intimacy is not necessarily about sex, but it is also not about just about 
knowing something about someone that isn't generally known, e.g., where the 
person went to elementary school or her mother's maiden name. It's more than 
just being able to answer the sorts of questions web sites ask as a way to 
establish one's identity. Intimacy has to do with the kinds of things that are 
known, in particular with knowing about the subjective experience of another 
person. At least that's how I would describe it -- and that's why I raised the 
question.

On Fri, Feb 19, 2016 at 3:39 PM Nick Thompson 
<nickthomp...@earthlink.net<mailto:nickthomp...@earthlink.net>> wrote:

Dear John and Russ,



Well, you question is an example of it self.  Who is best qualified to explain 
the basis of Nick's denial of subjectivity?  Is this a question about 
aetiology: I.e., the causal history of Nick's coming to deny subjectivity?  Or 
is it a question of what rational arguments Nick might make for his denial of 
subjectivity.  Note that there is nothing particularly private about either of 
those forms of the question.  FRIAM could get to work on answering them and 
Nick could stand aside and wonder at the quality and perspicacity of your 
answers.  My own most recent and condensed and approachable attempt to answer 
both versions of the question can be found in the manuscript that is attached.  
I can’t find cc of the published vsn at the moment.



I will think about the intimacy issue.  I think it’s about having some others 
who know things about you that are not generally known.  I would argue that 
when you get into bed with somebody naked, it’s a metaphor.  But then, I am old.



Nick



Nicholas S. Thompson

Emeritus Professor of Psychology and Biology

Clark University

http://home.earthlink.net/~nickthompson/naturaldesigns/



-Original Message-
From: Friam 
[mailto:friam-boun...@redfish.com<mailto:friam-boun...@redfish.com>] On Behalf 
Of John Kennison
Sent: Friday, February 19, 2016 2:30 PM
To: The Friday Morning Applied Complexity Coffee Group 
<friam@redfish.com<mailto:friam@redfish.com>>
Subject: Re: [FRIAM] Subjectivity and intimacy



One thing I wonder about (or perhaps have forgotten) in this discussion and 
Nick's denial is what the denial is based on. Is the absence of subjectivity 
supposed to be a scientific fact? If so, we should be discussing the 
experimental foundations of this fact. I have read of some experiments which 
seem to indicate that subjectiviity is not exactly what we (or what I) used to 
think it is --but which do not seem to disprove subjectivity.



--John



From: Friam [friam-boun...@redfish.com<mailto:friam-boun...@redfish.com>] on 
behalf of Russ Abbott [russ.abb...@gmail.com<mailto:russ.abb...@gmail.com>]

Sent: Friday, February 19, 2016 3:27 PM

To: The Friday Morning Applied Complexity Coffee Group

Subject: [FRIAM] Subjectivity and intimacy



We've had discussions on and off about subjectivity -- with me getting 
frustrated at Nick's denial thereof (if I understood him correctly).



It occurred to me recently that intimacy is defined -- as I understand it -- in 
terms of subjectivity, i.e., the sharing of one's (most private) subjective 
experiences with another.



I'm wondering what Nick thinks about this and whether anyone else has something 
to say about it. In particular, if there is no such thing as subjective 
experience, does that imply in your view that the same goes for intimacy?





FRIAM Applied Complexity Group listserv

Meets Fridays 9a-11:30 at cafe at St. John's College to unsubscribe 
http://redfish.com/mailman/listinfo/friam_redfish.com


FRIAM Applied Complexity Group listserv
Meets Fridays 9a-11:30 at cafe at St. John's College
to unsubscribe http://redfish.com/mailman/listinfo/friam_redfish.com


FRIAM Applied Complexity Group listserv
Meets Fridays 9a-11:30 at cafe at St. John's College
to unsubscribe http://redfish.com/mailman/listinfo/friam_redfish.com


Re: [FRIAM] Subjectivity and intimacy

2016-02-19 Thread John Kennison
One thing I wonder about (or perhaps have forgotten) in this discussion and 
Nick's denial is what the denial is based on. Is the absence of subjectivity 
supposed to be a scientific fact? If so, we should be discussing the 
experimental foundations of this fact. I have read of some experiments which 
seem to indicate that subjectiviity is not exactly what we (or what I) used to 
think it is --but which do not seem to disprove subjectivity. 

--John 

From: Friam [friam-boun...@redfish.com] on behalf of Russ Abbott 
[russ.abb...@gmail.com]
Sent: Friday, February 19, 2016 3:27 PM
To: The Friday Morning Applied Complexity Coffee Group
Subject: [FRIAM] Subjectivity and intimacy

We've had discussions on and off about subjectivity -- with me getting 
frustrated at Nick's denial thereof (if I understood him correctly).

It occurred to me recently that intimacy is defined -- as I understand it -- in 
terms of subjectivity, i.e., the sharing of one's (most private) subjective 
experiences with another.

I'm wondering what Nick thinks about this and whether anyone else has something 
to say about it. In particular, if there is no such thing as subjective 
experience, does that imply in your view that the same goes for intimacy?


FRIAM Applied Complexity Group listserv
Meets Fridays 9a-11:30 at cafe at St. John's College
to unsubscribe http://redfish.com/mailman/listinfo/friam_redfish.com


Re: [FRIAM] FW: Meat

2015-11-03 Thread John Kennison
I'm having trouble keeping up with this thread, but how does it relate to 
severely autistic people with almost zero social skills who, nonetheless, can 
prove deep mathematical theorems? 

From: Friam [friam-boun...@redfish.com] on behalf of Nick Thompson 
[nickthomp...@earthlink.net]
Sent: Tuesday, November 03, 2015 2:33 AM
To: c...@plektyx.com; 'The Friday Morning Applied Complexity Coffee Group'
Subject: Re: [FRIAM] FW: Meat

Hi Carl,

I like Steve's idea (Was it Steve's? I am having a hard time following!)
that as scientists we live in a network of other scientists, and our own
behavior is not so important as the constraints enforced on us as part of
the network.  So, my wonder at the fact that we make decisions in our
personal lives so unscientifically is really based on what Ryle would call a
category error ... an expectation that the behavior of an entity at once
level of organization (the scientist) should mirror the behavior of the
entity at the next level up (the science).  So, we shouldn't expect our
decision making processes with respect to hot dogs to be any more scientific
than the ordinary Joe's.

But then I get back to where I got started on this quest -- our Friday
morning discussions on global warming, or some of these other tricky issues
on which there is a scientific consensus, and yet there is a public debate.
Why do we not ALL -- as scientists -- agree that there exists a scientific
consensus on this matter and that that is the end of the discussion, until
further notice.  Not only do we not think scientifically with respect to
these issues, but we fail to accept the authority of the network of which we
are part.

Isn't that odd?

By the way, I think the heart disease thing is caused by sugar, not fat.
See, now I am doing it.

Nick

Nicholas S. Thompson
Emeritus Professor of Psychology and Biology
Clark University
http://home.earthlink.net/~nickthompson/naturaldesigns/


-Original Message-
From: Friam [mailto:friam-boun...@redfish.com] On Behalf Of Carl
Sent: Monday, November 02, 2015 10:07 PM
To: friam@redfish.com
Subject: Re: [FRIAM] FW: Meat

Speaking as a mammal, who has gone to some efforts to be well preserved,
it now appears that I am carcinogenic if consumed.   I am unsure as to
whether to be disappointed or elated.  Certainly a caution to those of
cannibalistic bent.

This does raise the issue that if I am indeed carcinogenic, why am I less so
to myself in the similar way that I  might be to others?

Ticks carry other nasty things, even without the Lyme problem.  I know
people who have Lyme, it is unpleasant for them.   I like beets, and
curry, irrespective of their supposed benefits, so am predisposed to
hear nice things about their benefits.   I believe most diagnoses of
arthritis are bogus because I don't wish to believe I am of an age
susceptible to such and because I can with some attention to detail fix
it most of the time.   Smoking is an obnoxious habit, perpetrated by
those who have little regard for themselves or others.   I'm sorry if it
causes them cancer.   I believed animal fats caused cardiovascular
issues since some hyperlipidemia runs in my family and have seen first
hand the devastation cardiovascular diseases  can cause.   However, I
have become convinced that (a) I feel noticeably better with some meat in my
diet and (b) my social group thinks that cholesterol is perhaps not as
strong a factor in heart disease as other things and (c) as I read up on
these things I see that the disease (like cancer) is not a
single thing with unique causes per individual.   I live next to pigs
and goats and chickens and am likely under no illusions about what "farm
fresh" in the grocery store means.   I am trying to make an organic
garden at my hut and am increasingly impressed with the difficulty of
defining the term.   Generally, I try to follow a fish and rice and
veggies Japanese diet; it feels "cleaner", which is an mental model brought
on by my very long standing Japanophilia as much as anything else.

There.   I see statements to the effect of "statistically, meat causes
cancer" and I laugh.  Thanks for playing.   Two Martian potatoes out of
a possible five.

C


On 10/27/15 11:06 PM, Nick Thompson wrote:
> Dear Friam members,
>
> As those of you in the mother church are already aware, I have been
> trying to foment a conversation about what rationality consists of and
> how does it relate to a purported scientific consensus.  I assume that
> you are all, more or less, rational people.  How exactly, then, did
> each of you come to the conclusion that, say, animal fats do or do not
> cause heart disease, smoking does or does not cause cancer, human
> activity does or does not cause global warming, that tick bites do (or
> do not) cause a syndrome called chronic Lyme disease, that, say, beet
> powder improves metabolism (?), or that turmeric does or does not
> alleviate arthritis.  Or, perhaps more important, how did 

Re: [FRIAM] BBC News - Ant colony 'personalities' shaped by environment

2014-08-24 Thread John Kennison
Eric,

As I understand it, Dennett's position and Chalmers' are not only incompatible, 
their difference is more extreme than one simply being the denial of the other. 
Dennett says that a zombie is simply impossible. If we tried to create a 
computer that could think like a human, it would be conscious --perhaps even if 
it just did a good job of analyzing things the way humans did --even without 
loving pets, etc. (I say perhaps, because I'm not sure what Dennett actually 
means.)
Chalmers says (I think) that even if we created a physically object that was 
identical to a human,  it wouldn't necessarily be conscious --which I find too 
extreme. When I said I favored Chalmers, I meant that it seems plausible that 
consciousness might not simply emerge if a system behaves in a sufficiently 
sophisticated way. --the way the system is constructed could make a difference. 
  But these are only top of my head guesses.

--John 


From: Friam [friam-boun...@redfish.com] on behalf of Eric Charles 
[eric.phillip.char...@gmail.com]
Sent: Sunday, August 24, 2014 10:04 AM
To: The Friday Morning Applied Complexity Coffee Group
Subject: Re: [FRIAM] BBC News - Ant colony 'personalities' shaped by
environment

John,
So, in a snapshot I think A conscious system and a non conscious one could 
be physically identical, however, I think it would be disingenuous to say that 
we could not tell them apart through interaction over time. This issue is not 
whether or not it is easy, but merely whether it is possible.

I guess the question boils down to how you respond to challenges about 
philosophical zombies. These discussions normally begin with someone asserting 
You can imagine things that behave exactly like you and I in all ways, but not 
conscious. The presenter then goes on to lay out a series of riddles these 
creatures lead to. However, I am not sure I buy the premise. I would assert 
that you CANNOT imagine such creatures. Can you really imagine a creature that 
acts exactly like you without consciousness? Perhaps you can imagine a creature 
that appears to act lovingly towards your dog (if you have a dog) without 
feeling the love that you feel. But can you imagine a creature that appears to 
act lovingly towards your dog with being aware of your dog?!?

It seems like the type of claim we allow people to get away with at the start 
of a philosophical discussion, because it is a pretty normal seeming premise, 
and we all like to play such games... but if we really stopped to consider the 
premise, we would not let it pass.

(Obviously, this need not be read as a question to you, it is a challenge to 
Chalmers and others who hold those views.)

Eric



---
Eric P. Charles, Ph.D.
Lab Manager
Center for Teaching, Research, and Learning
American University, Hurst Hall Room 203A
4400 Massachusetts Avenue, N.W.
Washington, DC 20016
phone: (202) 885-3867   fax: (202) 885-1190
email: echar...@american.edumailto:echar...@american.edu


On Fri, Aug 22, 2014 at 1:16 PM, John Kennison 
jkenni...@clarku.edumailto:jkenni...@clarku.edu wrote:
Thanks Nick,

I found a few statements I would revise in what I wrote.
Perhaps, I should have said that my argument seems valid rather correct.
I was careless in describing Chalmers' view (He said something like: A 
conscious system and a non conscious one could be physically identical).
And I was being presumptuous  in describing Dennett as giving a great tour of 
the issues  --I don't know that much about the issues.
--John

From: Friam [friam-boun...@redfish.commailto:friam-boun...@redfish.com] on 
behalf of Nick Thompson 
[nickthomp...@earthlink.netmailto:nickthomp...@earthlink.net]
Sent: Friday, August 22, 2014 12:37 PM
To: 'The Friday Morning Applied Complexity Coffee Group'
Subject: Re: [FRIAM]BBC News-   Ant colony  'personalities' 
shaped  by  environment

John,

Thanks for this.  But now I have to read Dennett again.  I am afraid my copy
is in a box in Santa Fe, so may have to come over and borrow yours for a few
days.  But I am in somebody else's vacation cabin in NH for the moment, so
it will be a while.

 The following is from my shaky memory.  Please don't flame me, anybody;
just put your arm around my shoulders and lead me from error.

There appears to be a divide amongst philosophers of science concerning how
much to be a rationalist.  Thomas Kuhn is the classic IRRATIONALIST An awful
lot of the philosophy of science that we were all taught in graduate school
is irrationalist in this sense.   Even Popper, who stressed the logic of
deduction in his philosophy (falsification) was irrationalist in his
account of where good scientific ideas come from (bold conjectures).  The
hallmark of an irrationalist is a tendency to put logic words in ironic
quotes, such as proof or inference or truth , or to use persuasion
words (intuition pumps) that avoid invoking logical

Re: [FRIAM] BBC News - Ant colony 'personalities' shaped by environment

2014-08-24 Thread John Kennison
Nick,

Consciousness is a term that is discussed by philosophers. If you don't have 
one you have proved half of Chalmers' position that it is possible for zombies 
(humans who lack this mysterious thing called consciousness) to exist. Th other 
half of Chalmers' position is that conscious humans also exist. I think I 
provide such an example. Chalmers would then (I suspect) conclude that 
consciousness is not completely physical as there seem to be no obvious 
physical differences that would explain which humans have consciousness and 
which do not. 

--John

From: Friam [friam-boun...@redfish.com] on behalf of Nick Thompson 
[nickthomp...@earthlink.net]
Sent: Sunday, August 24, 2014 12:05 PM
To: 'The Friday Morning Applied Complexity Coffee Group'
Subject: Re: [FRIAM] BBC News - Ant colony 'personalities'  shaped  by  
environment

Hey, wait a minute, guys!  You have lost me.  What is this consciousness
of which you speak.  I am not sure I have one and I need you to describe it
to me in a way that I can recognize it.

N

Nicholas S. Thompson
Emeritus Professor of Psychology and Biology
Clark University
http://home.earthlink.net/~nickthompson/naturaldesigns/

-Original Message-
From: Friam [mailto:friam-boun...@redfish.com] On Behalf Of John Kennison
Sent: Sunday, August 24, 2014 11:50 AM
To: The Friday Morning Applied Complexity Coffee Group
Subject: Re: [FRIAM] BBC News - Ant colony 'personalities' shaped by
environment

Eric,

As I understand it, Dennett's position and Chalmers' are not only
incompatible, their difference is more extreme than one simply being the
denial of the other.
Dennett says that a zombie is simply impossible. If we tried to create a
computer that could think like a human, it would be conscious --perhaps even
if it just did a good job of analyzing things the way humans did --even
without loving pets, etc. (I say perhaps, because I'm not sure what Dennett
actually means.)
Chalmers says (I think) that even if we created a physically object that was
identical to a human,  it wouldn't necessarily be conscious --which I find
too extreme. When I said I favored Chalmers, I meant that it seems plausible
that consciousness might not simply emerge if a system behaves in a
sufficiently sophisticated way. --the way the system is constructed could
make a difference.   But these are only top of my head guesses.

--John


From: Friam [friam-boun...@redfish.com] on behalf of Eric Charles
[eric.phillip.char...@gmail.com]
Sent: Sunday, August 24, 2014 10:04 AM
To: The Friday Morning Applied Complexity Coffee Group
Subject: Re: [FRIAM] BBC News - Ant colony 'personalities' shaped by
environment

John,
So, in a snapshot I think A conscious system and a non conscious one
could be physically identical, however, I think it would be disingenuous to
say that we could not tell them apart through interaction over time. This
issue is not whether or not it is easy, but merely whether it is possible.

I guess the question boils down to how you respond to challenges about
philosophical zombies. These discussions normally begin with someone
asserting You can imagine things that behave exactly like you and I in all
ways, but not conscious. The presenter then goes on to lay out a series of
riddles these creatures lead to. However, I am not sure I buy the premise. I
would assert that you CANNOT imagine such creatures. Can you really imagine
a creature that acts exactly like you without consciousness? Perhaps you can
imagine a creature that appears to act lovingly towards your dog (if you
have a dog) without feeling the love that you feel. But can you imagine a
creature that appears to act lovingly towards your dog with being aware of
your dog?!?

It seems like the type of claim we allow people to get away with at the
start of a philosophical discussion, because it is a pretty normal seeming
premise, and we all like to play such games... but if we really stopped to
consider the premise, we would not let it pass.

(Obviously, this need not be read as a question to you, it is a challenge to
Chalmers and others who hold those views.)

Eric



---
Eric P. Charles, Ph.D.
Lab Manager
Center for Teaching, Research, and Learning American University, Hurst Hall
Room 203A
4400 Massachusetts Avenue, N.W.
Washington, DC 20016
phone: (202) 885-3867   fax: (202) 885-1190
email: echar...@american.edumailto:echar...@american.edu


On Fri, Aug 22, 2014 at 1:16 PM, John Kennison
jkenni...@clarku.edumailto:jkenni...@clarku.edu wrote:
Thanks Nick,

I found a few statements I would revise in what I wrote.
Perhaps, I should have said that my argument seems valid rather correct.
I was careless in describing Chalmers' view (He said something like: A
conscious system and a non conscious one could be physically identical).
And I was being presumptuous  in describing Dennett as giving a great tour
of the issues  --I don't know that much

Re: [FRIAM] BBC News - Ant colony 'personalities' shaped by environment

2014-08-24 Thread John Kennison

Nick,

I hope that most people will give the answer yes to the question of whether I 
am conscious. I don't think of the criterion I gave as an operating definition 
--I don't claim it is useful in that way. It's sort of like trying to figure 
out whether someone did something deliberately. The actual meaning of doing 
something deliberately depends on certain assumptions (perhaps about 
consciousness) which might not be verifiable. But we can come up with a set of 
criteria for deciding whether we think that someone acted deliberately. We 
realize these criteria may mislead us, but they are better than nothing if we 
need to make a decision. 

--John

From: Friam [friam-boun...@redfish.com] on behalf of Nick Thompson 
[nickthomp...@earthlink.net]
Sent: Sunday, August 24, 2014 5:03 PM
To: 'The Friday Morning Applied Complexity Coffee Group'
Subject: Re: [FRIAM] BBC News - Ant colony  'personalities' shaped  by  
environment

John,

Well, actually my position is not that I am not conscious, but that your
operating definition of consciousness has little to do with anybody's answer
to the question Are you conscious? and everything to do with patterns of
doing.  Frank is the only participant in this argument who disagrees with me
about what consciousness is, and yet applies his definition consistently.  I
think. 

N

Nicholas S. Thompson
Emeritus Professor of Psychology and Biology
Clark University
http://home.earthlink.net/~nickthompson/naturaldesigns/

-Original Message-
From: Friam [mailto:friam-boun...@redfish.com] On Behalf Of John Kennison
Sent: Sunday, August 24, 2014 2:49 PM
To: The Friday Morning Applied Complexity Coffee Group
Subject: Re: [FRIAM] BBC News - Ant colony 'personalities' shaped by
environment

Nick,

Consciousness is a term that is discussed by philosophers. If you don't
have one you have proved half of Chalmers' position that it is possible for
zombies (humans who lack this mysterious thing called consciousness) to
exist. Th other half of Chalmers' position is that conscious humans also
exist. I think I provide such an example. Chalmers would then (I suspect)
conclude that consciousness is not completely physical as there seem to be
no obvious physical differences that would explain which humans have
consciousness and which do not.

--John

From: Friam [friam-boun...@redfish.com] on behalf of Nick Thompson
[nickthomp...@earthlink.net]
Sent: Sunday, August 24, 2014 12:05 PM
To: 'The Friday Morning Applied Complexity Coffee Group'
Subject: Re: [FRIAM] BBC News - Ant colony 'personalities'  shaped  by
environment

Hey, wait a minute, guys!  You have lost me.  What is this consciousness
of which you speak.  I am not sure I have one and I need you to describe it
to me in a way that I can recognize it.

N

Nicholas S. Thompson
Emeritus Professor of Psychology and Biology Clark University
http://home.earthlink.net/~nickthompson/naturaldesigns/

-Original Message-
From: Friam [mailto:friam-boun...@redfish.com] On Behalf Of John Kennison
Sent: Sunday, August 24, 2014 11:50 AM
To: The Friday Morning Applied Complexity Coffee Group
Subject: Re: [FRIAM] BBC News - Ant colony 'personalities' shaped by
environment

Eric,

As I understand it, Dennett's position and Chalmers' are not only
incompatible, their difference is more extreme than one simply being the
denial of the other.
Dennett says that a zombie is simply impossible. If we tried to create a
computer that could think like a human, it would be conscious --perhaps even
if it just did a good job of analyzing things the way humans did --even
without loving pets, etc. (I say perhaps, because I'm not sure what Dennett
actually means.) Chalmers says (I think) that even if we created a
physically object that was identical to a human,  it wouldn't necessarily be
conscious --which I find too extreme. When I said I favored Chalmers, I
meant that it seems plausible that consciousness might not simply emerge if
a system behaves in a sufficiently sophisticated way. --the way the system
is constructed could
make a difference.   But these are only top of my head guesses.

--John


From: Friam [friam-boun...@redfish.com] on behalf of Eric Charles
[eric.phillip.char...@gmail.com]
Sent: Sunday, August 24, 2014 10:04 AM
To: The Friday Morning Applied Complexity Coffee Group
Subject: Re: [FRIAM] BBC News - Ant colony 'personalities' shaped by
environment

John,
So, in a snapshot I think A conscious system and a non conscious one
could be physically identical, however, I think it would be disingenuous to
say that we could not tell them apart through interaction over time. This
issue is not whether or not it is easy, but merely whether it is possible.

I guess the question boils down to how you respond to challenges about
philosophical zombies. These discussions normally begin with someone
asserting You can

Re: [FRIAM] BBC News - Ant colony 'personalities' shaped by environment

2014-08-24 Thread John Kennison
I am willing to speak about definitions of consciousness or self-consciousness 
or deliberateness that depend on metaphysical assumptions and to speak of 
operating definitions that do not depend on these assumptions.

From: Friam [friam-boun...@redfish.com] on behalf of Nick Thompson 
[nickthomp...@earthlink.net]
Sent: Sunday, August 24, 2014 6:54 PM
To: 'The Friday Morning Applied Complexity Coffee Group'
Subject: Re: [FRIAM] BBC News   -   Ant colony  'personalities' shaped  
by  environment

Hi, John,

I think a third person definition of doing something deliberately would
come very close to what I mean by self-conscious.   (What we call self
consciousness in ordinary language usually refers to being conscious of
somebody else being conscious of what we are doing.)  So, I see promise in
what you say here.

Nick

Nicholas S. Thompson
Emeritus Professor of Psychology and Biology
Clark University
http://home.earthlink.net/~nickthompson/naturaldesigns/

-Original Message-
From: Friam [mailto:friam-boun...@redfish.com] On Behalf Of John Kennison
Sent: Sunday, August 24, 2014 6:38 PM
To: The Friday Morning Applied Complexity Coffee Group
Subject: Re: [FRIAM] BBC News - Ant colony 'personalities' shaped by
environment


Nick,

I hope that most people will give the answer yes to the question of
whether I am conscious. I don't think of the criterion I gave as an
operating definition --I don't claim it is useful in that way. It's sort of
like trying to figure out whether someone did something deliberately. The
actual meaning of doing something deliberately depends on certain
assumptions (perhaps about consciousness) which might not be verifiable. But
we can come up with a set of criteria for deciding whether we think that
someone acted deliberately. We realize these criteria may mislead us, but
they are better than nothing if we need to make a decision.

--John

From: Friam [friam-boun...@redfish.com] on behalf of Nick Thompson
[nickthomp...@earthlink.net]
Sent: Sunday, August 24, 2014 5:03 PM
To: 'The Friday Morning Applied Complexity Coffee Group'
Subject: Re: [FRIAM] BBC News - Ant colony  'personalities' shaped  by
environment

John,

Well, actually my position is not that I am not conscious, but that your
operating definition of consciousness has little to do with anybody's answer
to the question Are you conscious? and everything to do with patterns of
doing.  Frank is the only participant in this argument who disagrees with me
about what consciousness is, and yet applies his definition consistently.  I
think. 

N

Nicholas S. Thompson
Emeritus Professor of Psychology and Biology Clark University
http://home.earthlink.net/~nickthompson/naturaldesigns/

-Original Message-
From: Friam [mailto:friam-boun...@redfish.com] On Behalf Of John Kennison
Sent: Sunday, August 24, 2014 2:49 PM
To: The Friday Morning Applied Complexity Coffee Group
Subject: Re: [FRIAM] BBC News - Ant colony 'personalities' shaped by
environment

Nick,

Consciousness is a term that is discussed by philosophers. If you don't
have one you have proved half of Chalmers' position that it is possible for
zombies (humans who lack this mysterious thing called consciousness) to
exist. Th other half of Chalmers' position is that conscious humans also
exist. I think I provide such an example. Chalmers would then (I suspect)
conclude that consciousness is not completely physical as there seem to be
no obvious physical differences that would explain which humans have
consciousness and which do not.

--John

From: Friam [friam-boun...@redfish.com] on behalf of Nick Thompson
[nickthomp...@earthlink.net]
Sent: Sunday, August 24, 2014 12:05 PM
To: 'The Friday Morning Applied Complexity Coffee Group'
Subject: Re: [FRIAM] BBC News - Ant colony 'personalities'  shaped  by
environment

Hey, wait a minute, guys!  You have lost me.  What is this consciousness
of which you speak.  I am not sure I have one and I need you to describe it
to me in a way that I can recognize it.

N

Nicholas S. Thompson
Emeritus Professor of Psychology and Biology Clark University
http://home.earthlink.net/~nickthompson/naturaldesigns/

-Original Message-
From: Friam [mailto:friam-boun...@redfish.com] On Behalf Of John Kennison
Sent: Sunday, August 24, 2014 11:50 AM
To: The Friday Morning Applied Complexity Coffee Group
Subject: Re: [FRIAM] BBC News - Ant colony 'personalities' shaped by
environment

Eric,

As I understand it, Dennett's position and Chalmers' are not only
incompatible, their difference is more extreme than one simply being the
denial of the other.
Dennett says that a zombie is simply impossible. If we tried to create a
computer that could think like a human, it would be conscious --perhaps even
if it just did a good job of analyzing things the way humans did --even
without loving pets, etc. (I say

Re: [FRIAM] BBC News - Ant colony 'personalities' shaped by environment

2014-08-22 Thread John Kennison
Nick: 
I find your distinction between metaphysical questions and factual questions 
helpful because it clarifies the vague feeling I expressed about making some 
sort of error when I said that consciousness is having an inner subjective 
life. I no longer feel it is an error but I should categorize it as a 
metaphysical position rather than a scientific fact. (I prefer the term 
``scientific fact`` to your term ``fact``.) It still seems like a good argument 
(I know consciousness exists because I experience it) even though this cannot 
be a scientific argument.

Eric, Steve, et al:
Thanks for your very interesting comments. I would like to add some further 
comments about Dennett. I both enjoyed and was frustrated by his book 
Consciousness Explained. I recommend it highly but with the following 
caveats: 

(1) I wish the book were organized differently. I think it should have started 
with
The Challenge (section 5 of chapter 2, p.39-42). I paraphrase this challenge 
as:
 First, Dennett says he wants to explain Consciousness in 
scientific terms, without invoking anything beyond contemporary science. I take 
this to mean that he wants to show that we can analyze and explain human 
behavior entirely in scientific, materialistic terms without appealing to any 
'mysterious' forces.  (Therefore, to focus on the behavior rather than the 
motives, of conscious people, Dennett starts by telling speculative stories 
about the phenomenology of consciousness.)  
Secondly, he doesn't want to be like behaviorists who pretend they 
don't have the experiences we know darn well they share they share with us. If 
I [Dennett] wish to deny the existence of some controversial feature of 
consciousness, the burden falls on me to that it is somehow illusory. (p.40 of 
the book).
 Thirdly he wants to do an honest job of explaining the empirical 
evidence. 
This challenge intrigued me. The first and second goals seem almost 
contradictory. I wondered how he could possibly pull it off.
   
(2) As far as I remember, Dennett never summarizes how he met this challenge.  
(I read this book over 15 years ago and I might have forgotten the summary.  At 
any rate, as I go over the book now, I can't find the kind of summary I would 
like to see.)
So here is my summary of how Dennett did:
(a) After having read the book, I feel there is no theoretical barrier to 
explaining all of the behavior of apparently conscious beings in purely 
materialistic terms. 
(b) My memory is that Dennett explains the feeling of being conscious in terms 
of the strong AI hypothesis, which says that any system that carries out a 
sufficiently complex task will automatically be conscious. I am not certain if 
I believe this, but it or something like it seems necessary if we take the 
first two goals seriously.  Dennett apparently believes that the emergence of 
consciousness depends only on the behavior exhibited. By contrast, Chalmers 
argues that a conscious systems and a non-conscious system could exhibit the 
same type of behavior. I don't see any reason to favor either position, but I 
prefer Chalmers.

(3) On Dennett's style: This is what I find both frustrating and intriguing. He 
seems to discuss various ideas without fully arranging them into an argument, 
as I would tend to do.  Dennett relies on this tendency of the reader to 
complete the argument. So Dennett spends less time on argumentation and more on 
telling stories. Sometimes it works, sometimes it doesn't. As mentioned above, 
I came away with a strong feeling about the first part of the challenge. I also 
had a strong feeling that our consciousness often fools us into thinking it is 
in control when it isn't. I liked Dennett's presentation of the Pandemonium 
model of language (based on work of Selfridge, Dawkins and others) and I feel 
it explains a lot of things that would otherwise be murky. On the other hand, I 
was dissatisfied with the chapter on Qualia Disqualified. I even found myself 
agreeing with his students (and others) that he hasn't really explained 
consciousness --but I think he gave us a great tour of the issu
 es.  (If I had written the book, and arranged it more logically, the thread of 
the arguments might have been clearer, but it would have been a much less 
effective book.) 

--John

From: Friam [friam-boun...@redfish.com] on behalf of Eric Smith 
[desm...@santafe.edu]
Sent: Saturday, August 16, 2014 12:31 PM
To: The Friday Morning Applied Complexity Coffee Group
Subject: Re: [FRIAM] BBC News   -   Ant colony  'personalities' shaped  
by  environment

Hi Steve,

I am neither knowledgeable, nor do I have time to report even my own 
experiences, without making a mess of things.  But perhaps I can give some 
titles of things people have pointed out to me.

There seem to be several schools of approach (meaning, groups of people who 
criticize each other a lot).  I't hard even 

Re: [FRIAM] BBC News - Ant colony 'personalities' shaped by environment

2014-08-22 Thread John Kennison
Thanks Nick,

I found a few statements I would revise in what I wrote.
Perhaps, I should have said that my argument seems valid rather correct.
I was careless in describing Chalmers' view (He said something like: A 
conscious system and a non conscious one could be physically identical).
And I was being presumptuous  in describing Dennett as giving a great tour of 
the issues  --I don't know that much about the issues.
--John

From: Friam [friam-boun...@redfish.com] on behalf of Nick Thompson 
[nickthomp...@earthlink.net]
Sent: Friday, August 22, 2014 12:37 PM
To: 'The Friday Morning Applied Complexity Coffee Group'
Subject: Re: [FRIAM]BBC News-   Ant colony  'personalities' 
shaped  by  environment

John,

Thanks for this.  But now I have to read Dennett again.  I am afraid my copy
is in a box in Santa Fe, so may have to come over and borrow yours for a few
days.  But I am in somebody else's vacation cabin in NH for the moment, so
it will be a while.

 The following is from my shaky memory.  Please don't flame me, anybody;
just put your arm around my shoulders and lead me from error.

There appears to be a divide amongst philosophers of science concerning how
much to be a rationalist.  Thomas Kuhn is the classic IRRATIONALIST An awful
lot of the philosophy of science that we were all taught in graduate school
is irrationalist in this sense.   Even Popper, who stressed the logic of
deduction in his philosophy (falsification) was irrationalist in his
account of where good scientific ideas come from (bold conjectures).  The
hallmark of an irrationalist is a tendency to put logic words in ironic
quotes, such as proof or inference or truth , or to use persuasion
words (intuition pumps) that avoid invoking logical relations.  So,
Dennett's failure to organize the book in the manner you suggest is part and
parcel of his irrationalism, as is, by the way, your observation that an
argument can be effective without being clear.

I want to pull back a bit my distinction between metaphysical and factual.
I guess I REALLY think the distinction is relative to a particular argument.
In any argument, there are the facts we argue from and the facts we argue
about.  There is a sense in which metaphysics consists in the facts we
ALWAYS argue from.  I hope I haven't shot my own high horse out from under
me, here.

Nick

Nicholas S. Thompson
Emeritus Professor of Psychology and Biology
Clark University
http://home.earthlink.net/~nickthompson/naturaldesigns/

-Original Message-
From: Friam [mailto:friam-boun...@redfish.com] On Behalf Of John Kennison
Sent: Friday, August 22, 2014 8:35 AM
To: The Friday Morning Applied Complexity Coffee Group
Subject: Re: [FRIAM] BBC News - Ant colony 'personalities' shaped by
environment

Nick:
I find your distinction between metaphysical questions and factual questions
helpful because it clarifies the vague feeling I expressed about making
some sort of error when I said that consciousness is having an inner
subjective life. I no longer feel it is an error but I should categorize it
as a metaphysical position rather than a scientific fact. (I prefer the term
``scientific fact`` to your term ``fact``.) It still seems like a good
argument (I know consciousness exists because I experience it) even though
this cannot be a scientific argument.

Eric, Steve, et al:
Thanks for your very interesting comments. I would like to add some further
comments about Dennett. I both enjoyed and was frustrated by his book
Consciousness Explained. I recommend it highly but with the following
caveats:

(1) I wish the book were organized differently. I think it should have
started with
The Challenge (section 5 of chapter 2, p.39-42). I paraphrase this
challenge as:
 First, Dennett says he wants to explain Consciousness in
scientific terms, without invoking anything beyond contemporary science. I
take this to mean that he wants to show that we can analyze and explain
human behavior entirely in scientific, materialistic terms without appealing
to any 'mysterious' forces.  (Therefore, to focus on the behavior rather
than the motives, of conscious people, Dennett starts by telling speculative
stories about the phenomenology of consciousness.)
Secondly, he doesn't want to be like behaviorists who pretend
they don't have the experiences we know darn well they share they share with
us. If I [Dennett] wish to deny the existence of some controversial feature
of consciousness, the burden falls on me to that it is somehow illusory.
(p.40 of the book).
 Thirdly he wants to do an honest job of explaining the
empirical evidence.
This challenge intrigued me. The first and second goals seem almost
contradictory. I wondered how he could possibly pull it off.

(2) As far as I remember, Dennett never summarizes how he met this
challenge.  (I read this book over 15 years ago and I might have forgotten
the summary.  At any rate

Re: [FRIAM] BBC News - Ant colony 'personalities' shaped by environment

2014-08-16 Thread John Kennison
 to
imagine that, perhaps it's sides are not entirely opaque, or not opaque all
the time.  Perhaps they are sometimes translucent?

How about a different metaphor altogether?  How about the metaphor of point
of view?  My consciousness is just that what is seen from the  point of
view on the world from where I stand.  It is mine only in the sense that it
is indexed to me, not in the sense that I own it or that it is in me.  For
example, there is a cup on my desk whose inscription is turned toward me so
that if you were sitting across my desk from me right now, you would not
have access to it.   The inscription is, ONLY MUGS PAY POLL TAX.   I am
conscious of it in the sense that my behavior points to it.  From your point
of view, my consciousness is just all that my behavior designates.   When
your behavior designates the relations between me and some of the objects in
your environment, you become conscious of what I am conscious.  When my
behavior designates those same relations,  I become self-conscious.  I think
self-consciousness is what we are principally arguing about, here.

I hope this answer is somewhat satisfying.  Thanks for running me around the
track.  I am trying to write some on this subject this summer.  I really
need the exercise.

Best,

Nick




Nicholas S. Thompson
Emeritus Professor of Psychology and Biology
Clark University
http://home.earthlink.net/~nickthompson/naturaldesigns/

-Original Message-
From: Friam [mailto:friam-boun...@redfish.com] On Behalf Of John Kennison
Sent: Friday, August 15, 2014 12:52 PM
To: The Friday Morning Applied Complexity Coffee Group
Subject: Re: [FRIAM] BBC News - Ant colony 'personalities' shaped by
environment

Hi Nick,

I certainly don't think of what you said as rude  --in fact I asked you to
tell what errors you might see in what I said.
And in any case, I am very glad to agree that we are old friends and can, if
necessary, forgive what might appear as rudeness.

I am willing to accept your conclusion that the words inner subjective
life are not really very useful and do no contribute much to my idea of
what consciousness is. I don't think I claimed that they are either of these
things.

I am having difficulty seeing the connection between these words and a
quasi-legal understanding that I and only I get to speak for myself.
I guess I would say that my sense of what my consciousness is all about
will be different from yours because I have access to my thoughts and vague
feelings etc. that differs from the kind of access you have. It's okay with
me if you speak for myself (so to speak)  --and I imagine you will, perhaps
over the previous sentence.  I invite and will (I think) welcome your
analysis.

--John


From: Friam [friam-boun...@redfish.com] on behalf of Nick Thompson
[nickthomp...@earthlink.net]
Sent: Friday, August 15, 2014 11:38 AM
To: 'The Friday Morning Applied Complexity Coffee Group'
Cc: James Laird
Subject: Re: [FRIAM] BBC News - Ant colony  'personalities' shaped  by
environment

Hi, John,

Nothing like a sober, quiet, good question to knock an old warrior off his
high horse.

Ok.  Now that I am standing on the ground ...

First, let us stipulate, we are talking about self-consciousness, here, ...
something beyond sentience, right?  If so, then I think your question is a
wonderful example of a mystery, like we talked about yesterday.  A mystery
is a state of pleasurable confusion generated by using words outside their
realm of usefulness.  So, I would predict that if we sat down and unpacked
inner, subjective, and life we would discover that these words have
really nothing to contribute beyond the assertion that I, and only I, get
to speak for me.  In other words, under your use of consciousness,  it is
really a quasi-legal understanding central to human interaction that, in the
absence of a legal certification of incompetence, our assertions about our
own needs, wants, thoughts, etc., are to be taken as definitive.   So, for
instance, what I just said -- that your view of consciousness is not quite
what you think it is -- would be (may be) seen as RUDE, in polite society,
because, on your own understanding of consciousness, you and only you get to
say what you think it is.  Because we have been friends for more than 40
years, I hoping you will let that rudeness pass.

On my account, an entity is conscious of something when it acts with respect
to it, and SELF-conscious, when it acts with reference to itself.  On that
account, a simple thermostat is clearly conscious, but not self-conscious.
A more complicated thermostat, which calibrates its own sensitivity (which
most modern thermostats do), would probably have to be admitted as
self-conscious.

Nick
Nicholas S. Thompson
Emeritus Professor of Psychology and Biology Clark University
http://home.earthlink.net/~nickthompson/naturaldesigns/

-Original Message-
From: Friam [mailto:friam-boun...@redfish.com] On Behalf Of John Kennison

Re: [FRIAM] BBC News - Ant colony 'personalities' shaped by environment

2014-08-15 Thread John Kennison
Nick,

I guess my criterion for consciousness would be something like has an inner 
subjective life. It's not something that I can measure and it has the problem 
of circularity  --if you ask me what I mean by an inner subjective life I 
will soon be making a circular definition. I am willing to concede that I don't 
have a suitable definition for a scientific study of consciousness. Still the 
question of whether a thermostat has consciousness seems meaningful to me. (I 
don't have an answer 
--other than I doubt it. )
Perhaps, I am making some kind of error. If so, could you explain what my 
mistake is. 

--John

From: Friam [friam-boun...@redfish.com] on behalf of Nick Thompson 
[nickthomp...@earthlink.net]
Sent: Friday, August 15, 2014 10:20 AM
To: 'The Friday Morning Applied Complexity Coffee Group'
Subject: Re: [FRIAM] BBC News - Ant colony 'personalities' shaped   by  
environment

So, I looked up David Chalmers …  Yeh, I know:  I shouldn’t have HAD  to look 
up David Chalmers.   Here from Philosophy Index

A potential problem with this speculation, which Chalmers acknowledges, is that 
it may imply the consciousness of things that we would not normally consider to 
have consciousness at all. For instance, Chalmers wonders if this means that a 
thermostat may have some experiential properties, even if they are especially 
dull. He does not commit to the notion that they do, but the possibility 
remains in the more speculative area of his thought.

This is one of those “TED” insights, to which the only rational response is, 
“Duh!”  Why exactly is that a problem?  What exactly would it have meant to say 
that “humans are conscious” if it were not possible to discover that (1) things 
other than humans are conscious and/or that humans are not, in fact, conscious. 
 Either we have a criterion for consciousness or we don’t; once we have a 
criterion, we either apply it rigorously or … we are dishonest.  It’s really 
quite simple, actually.


N

Nicholas S. Thompson
Emeritus Professor of Psychology and Biology
Clark University
http://home.earthlink.net/~nickthompson/naturaldesigns/

From: Friam [mailto:friam-boun...@redfish.com] On Behalf Of Eric Smith
Sent: Friday, August 15, 2014 9:45 AM
To: The Friday Morning Applied Complexity Coffee Group
Subject: Re: [FRIAM] BBC News - Ant colony 'personalities' shaped by environment

Quick, somebody call David Chalmers!


On Aug 15, 2014, at 9:25 AM, Eric Charles wrote:


Weird that they want to call it personality instead of more simply saying 
that ant colonies seem to adapt to their local environment. Of course, the 
flashiness of the claim is the only reason it is being covered on the BBC, so I 
guess it isn't that weird after all.


---
Eric P. Charles, Ph.D.
Lab Manager
Center for Teaching, Research, and Learning
American University, Hurst Hall Room 203A
4400 Massachusetts Avenue, N.W.
Washington, DC 20016
phone: (202) 885-3867   fax: (202) 885-1190
email: echar...@american.edumailto:echar...@american.edu

On Fri, Aug 8, 2014 at 3:57 PM, Gillian Densmore 
gil.densm...@gmail.commailto:gil.densm...@gmail.com wrote:
A few swarm inteligence from the 90s described that.  Scott Kelly's Fast Cheap 
and Out of Controll  touched on that. In his case they knew ants (and often 
uncles) could pass around experience- and displayed something simillar to 
hummans sense of experience they didn't have a explination. Then again his 
forray into science was from the 90s.

On Fri, Aug 8, 2014 at 9:01 AM, Tom Johnson 
t...@jtjohnson.commailto:t...@jtjohnson.com wrote:

So who is going to integrate this into the sugar model?

http://www.bbc.com/news/science-environment-28658268

===
Tom Johnson - Inst. for Analytic Journalism
Santa Fe, NM
t...@jtjohnson.commailto:t...@jtjohnson.com.
505-473-9646tel:505-473-9646
===


FRIAM Applied Complexity Group listserv
Meets Fridays 9a-11:30 at cafe at St. John's College
to unsubscribe http://redfish.com/mailman/listinfo/friam_redfish.com



FRIAM Applied Complexity Group listserv
Meets Fridays 9a-11:30 at cafe at St. John's College
to unsubscribe http://redfish.com/mailman/listinfo/friam_redfish.com


FRIAM Applied Complexity Group listserv
Meets Fridays 9a-11:30 at cafe at St. John's College
to unsubscribe http://redfish.com/mailman/listinfo/friam_redfish.com



FRIAM Applied Complexity Group listserv
Meets Fridays 9a-11:30 at cafe at St. John's College
to unsubscribe http://redfish.com/mailman/listinfo/friam_redfish.com


Re: [FRIAM] BBC News - Ant colony 'personalities' shaped by environment

2014-08-15 Thread John Kennison
Hi Nick,

I certainly don't think of what you said as rude  --in fact I asked you to 
tell what errors you might see in what I said. 
And in any case, I am very glad to agree that we are old friends and can, if 
necessary, forgive what might appear as rudeness. 

I am willing to accept your conclusion that the words inner subjective life 
are not really very useful and do no contribute much to my idea of what 
consciousness is. I don't think I claimed that they are either of these things. 

I am having difficulty seeing the connection between these words and a 
quasi-legal understanding that I and only I get to speak for myself. 
I guess I would say that my sense of what my consciousness is all about  will 
be different from yours because I have access to my thoughts and vague feelings 
etc. that differs from the kind of access you have. It's okay with me if you 
speak for myself (so to speak)  --and I imagine you will, perhaps  over the 
previous sentence.  I invite and will (I think) welcome your analysis. 

--John


From: Friam [friam-boun...@redfish.com] on behalf of Nick Thompson 
[nickthomp...@earthlink.net]
Sent: Friday, August 15, 2014 11:38 AM
To: 'The Friday Morning Applied Complexity Coffee Group'
Cc: James Laird
Subject: Re: [FRIAM] BBC News - Ant colony  'personalities' shaped  by  
environment

Hi, John,

Nothing like a sober, quiet, good question to knock an old warrior off his
high horse.

Ok.  Now that I am standing on the ground ...

First, let us stipulate, we are talking about self-consciousness, here, ...
something beyond sentience, right?  If so, then I think your question is a
wonderful example of a mystery, like we talked about yesterday.  A mystery
is a state of pleasurable confusion generated by using words outside their
realm of usefulness.  So, I would predict that if we sat down and unpacked
inner, subjective, and life we would discover that these words have
really nothing to contribute beyond the assertion that I, and only I, get
to speak for me.  In other words, under your use of consciousness,  it is
really a quasi-legal understanding central to human interaction that, in the
absence of a legal certification of incompetence, our assertions about our
own needs, wants, thoughts, etc., are to be taken as definitive.   So, for
instance, what I just said -- that your view of consciousness is not quite
what you think it is -- would be (may be) seen as RUDE, in polite society,
because, on your own understanding of consciousness, you and only you get to
say what you think it is.  Because we have been friends for more than 40
years, I hoping you will let that rudeness pass.

On my account, an entity is conscious of something when it acts with respect
to it, and SELF-conscious, when it acts with reference to itself.  On that
account, a simple thermostat is clearly conscious, but not self-conscious.
A more complicated thermostat, which calibrates its own sensitivity (which
most modern thermostats do), would probably have to be admitted as
self-conscious.

Nick
Nicholas S. Thompson
Emeritus Professor of Psychology and Biology
Clark University
http://home.earthlink.net/~nickthompson/naturaldesigns/

-Original Message-
From: Friam [mailto:friam-boun...@redfish.com] On Behalf Of John Kennison
Sent: Friday, August 15, 2014 11:00 AM
To: The Friday Morning Applied Complexity Coffee Group
Subject: Re: [FRIAM] BBC News - Ant colony 'personalities' shaped by
environment

Nick,

I guess my criterion for consciousness would be something like has an inner
subjective life. It's not something that I can measure and it has the
problem of circularity  --if you ask me what I mean by an inner subjective
life I will soon be making a circular definition. I am willing to concede
that I don't have a suitable definition for a scientific study of
consciousness. Still the question of whether a thermostat has consciousness
seems meaningful to me. (I don't have an answer --other than I doubt it. )
Perhaps, I am making some kind of error. If so, could you explain what my
mistake is.

--John

From: Friam [friam-boun...@redfish.com] on behalf of Nick Thompson
[nickthomp...@earthlink.net]
Sent: Friday, August 15, 2014 10:20 AM
To: 'The Friday Morning Applied Complexity Coffee Group'
Subject: Re: [FRIAM] BBC News - Ant colony 'personalities' shaped   by
environment

So, I looked up David Chalmers .  Yeh, I know:  I shouldn't have HAD  to
look up David Chalmers.   Here from Philosophy Index

A potential problem with this speculation, which Chalmers acknowledges, is
that it may imply the consciousness of things that we would not normally
consider to have consciousness at all. For instance, Chalmers wonders if
this means that a thermostat may have some experiential properties, even if
they are especially dull. He does not commit to the notion that they do, but
the possibility remains in the more speculative area of his

Re: [FRIAM] BBC News - Ant colony 'personalities' shaped by environment

2014-08-15 Thread John Kennison
Nick,

I'm game for going slowly, particularly as I will soon go to bed and I know, 
from experience, that I shouldn't write emails when I'm tired. 
In any case, there's a lot to chew over and I will eagerly start to work on it 
in the morning.
--John

From: Friam [friam-boun...@redfish.com] on behalf of Nick Thompson 
[nickthomp...@earthlink.net]
Sent: Friday, August 15, 2014 9:07 PM
To: 'The Friday Morning Applied Complexity Coffee Group'
Subject: Re: [FRIAM] BBC News - Ant colony  'personalities' shaped  by  
environment

John,

Ok.  I am in.  But we have to go slowly, because, as somebody
famously said, In philosophy, if you are not moving slowly, you aren't
moving.   Not clear where to start.  I don't want to try to defend my
insight that our vernacular understanding of consciousness  arises not
because it is accurate but because it makes society possible. I will say  in
its defense only that the McNauton Rule which  forms the basis for our
notion of legal responsibility, states that I can only be considered
criminally responsible If I know the nature and quality of my own acts.
This phrase, knowing nature and quality of one's acts sounds a heckuva lot
like a definition of [self] consciousness to me.

I thought we perhaps could start with unpacking interior, since it
appears in both of your messages (access).  What does it mean to say that
my thoughts  are inside me.  It ought to mean, if we play the language
game of inside by the rules, that there is some sort of container that my
thoughts are enclosed within.   The use of the word, access, would seem to
suggest that I have ways of getting at the insides of the box to see my
thoughts that you do not have.  Perhaps the box is a 5-sided box, and it's
open side faces me, so I can see inside and you cannot?   If that is how the
metaphor works, then you should be able to come around  to my side of the
box and look in examine its contents with me.  Or, if my access is provided
by a key, you should be able to use that key to get inside my box.  In other
words, there should be some set of conditions under which you can see
exactly what I see.  Since this entailment of the box metaphor undermines
the essential privacy of mind, I assume that you would rule it out by, say,
asserting that only I have the key to my box, and I cannot loan it to you.


But now we encounter another problem.  I think you would agree that you do
have some access to the inside of my box, beyond the access that I might
provide you by telling you what is inside it.   Certainly, if I wrote you
now the words, I really have no interest in issues in the philosophy of
mind, you would have every reason to assert that I had misrepresented the
contents of my box to you.  So, to make the metaphor work, we would have to
imagine that, perhaps it's sides are not entirely opaque, or not opaque all
the time.  Perhaps they are sometimes translucent?

How about a different metaphor altogether?  How about the metaphor of point
of view?  My consciousness is just that what is seen from the  point of
view on the world from where I stand.  It is mine only in the sense that it
is indexed to me, not in the sense that I own it or that it is in me.  For
example, there is a cup on my desk whose inscription is turned toward me so
that if you were sitting across my desk from me right now, you would not
have access to it.   The inscription is, ONLY MUGS PAY POLL TAX.   I am
conscious of it in the sense that my behavior points to it.  From your point
of view, my consciousness is just all that my behavior designates.   When
your behavior designates the relations between me and some of the objects in
your environment, you become conscious of what I am conscious.  When my
behavior designates those same relations,  I become self-conscious.  I think
self-consciousness is what we are principally arguing about, here.

I hope this answer is somewhat satisfying.  Thanks for running me around the
track.  I am trying to write some on this subject this summer.  I really
need the exercise.

Best,

Nick




Nicholas S. Thompson
Emeritus Professor of Psychology and Biology
Clark University
http://home.earthlink.net/~nickthompson/naturaldesigns/

-Original Message-
From: Friam [mailto:friam-boun...@redfish.com] On Behalf Of John Kennison
Sent: Friday, August 15, 2014 12:52 PM
To: The Friday Morning Applied Complexity Coffee Group
Subject: Re: [FRIAM] BBC News - Ant colony 'personalities' shaped by
environment

Hi Nick,

I certainly don't think of what you said as rude  --in fact I asked you to
tell what errors you might see in what I said.
And in any case, I am very glad to agree that we are old friends and can, if
necessary, forgive what might appear as rudeness.

I am willing to accept your conclusion that the words inner subjective
life are not really very useful and do no contribute much to my idea of
what consciousness is. I don't think I claimed

Re: [FRIAM] rational

2014-01-08 Thread John Kennison

Okay, here's my dilemma in a slightly different form. Suppose a person acts 
entirely on the basis of messages received from a Ouija board. This certainly 
appears to be irrational. But it could be said to be based on a premise that 
the Ouija board is infallible. If we accept this, then I doubt that there is 
any such thing as an irrational action.

Or, if I get angry and punch the wall, leaving an awful hole in the wall and a 
painful bruise on my hand, was I acting rationally on a fleetingly held 
premise, that the wall needed punching? What can we do to rescue the term 
irrational ? I had thought that Glen's approach was going far beyond what 
irrationality really is, but now it looks like the best one out there. 

--John


From: Friam [friam-boun...@redfish.com] on behalf of Nick Thompson 
[nickthomp...@earthlink.net]
Sent: Tuesday, January 07, 2014 11:03 PM
To: 'The Friday Morning Applied Complexity Coffee Group'
Subject: Re: [FRIAM] rational

IS THERE A LOGICIAN IN THE HOUSE?

John kennison asked:

AT any rate, my question would be: Is there a sense in which the [below]
type of thinking (based on the premises Nick assumed) is irrational.

[see below]

Nick

Nicholas S. Thompson
Emeritus Professor of Psychology and Biology
Clark University
http://home.earthlink.net/~nickthompson/naturaldesigns/

-Original Message-
From: Friam [mailto:friam-boun...@redfish.com] On Behalf Of John Kennison
Sent: Tuesday, January 07, 2014 3:51 PM
To: The Friday Morning Applied Complexity Coffee Group
Subject: Re: [FRIAM] rational

Nick,  There are several perfectly rational ways in which someone might
decide to oppose gay marriage. I indicated one way, based on the premise
that homosexuality is perverse. You indicated another way. A third
possibility is that the speaker really believes the stated reason that gay
marriage would threaten traditional marriage.
Your example of a possible reconstruction of the reasoning is interesting
because, if correct, it would appear to be highly irrational as the
opposition to gay marriage seems to be based on homophobia.

Watching men neck in public makes me uncomfortable Married people are
allowed to neck in public.
Anything that makes me uncomfortable should be banned Therefore Gay marriage
should be banned.

My dilemma (which I imagine you intended) is that I would like to say that
the above thought process is a good example of irrational thinking but,
strangely enough, it has the appearance, and structure, of a logical
argument. I think I would wonder how the supposed speaker would acquire the
belief or premise that anything that makes me feel uncomfortable should be
banned. I doubt that right-wingers would feel that you are defending them
well if you said that this is a typical right wing rationale.

AT any rate, my question would be: Is there a sense in which the above type
of thinking (based on the premises Nick assumed) is irrational.









From: Friam [friam-boun...@redfish.com] on behalf of Nick Thompson
[nickthomp...@earthlink.net]
Sent: Tuesday, January 07, 2014 1:57 PM
To: 'The Friday Morning Applied Complexity Coffee Group'
Subject: Re: [FRIAM] rational

At last.
Ok, so to claim that this is irrational, we have to know the chain of
premises and conclusions that leads to this conclusion.
As you rightly point out, we can supply premises that make the reasoning
look crazy or we can supply premises that make it look reasonable.   But
isn't it bad reasoning to claim that reasoning is bad without having done
that?

I assume that the reasoning behind bans on gay marriage goes something like
this:

Watching men neck in public makes me uncomfortable Married people are
allowed to neck in public.
Anything that makes me uncomfortable should be banned Therefore Gay marriage
should be banned.

Perfectly RATIONAL
N

Nicholas S. Thompson
Emeritus Professor of Psychology and Biology Clark University
http://home.earthlink.net/~nickthompson/naturaldesigns/

-Original Message-
From: Friam [mailto:friam-boun...@redfish.com] On Behalf Of John Kennison
Sent: Tuesday, January 07, 2014 10:35 AM
To: The Friday Morning Applied Complexity Coffee Group
Subject: Re: [FRIAM] rational


I think one example might be I oppose gay marriage because it would
undermine my own (straight) marriage.
One could interpret the reason given as a false premise (you could always do
that whenever someone gives a reason for a belief) but here it looks more
like a rationalization. Of course I can't be certain, but I suspect that the
real reason is the false premise that homosexuality is a pervasion and a
different premise is substituted because it was felt to be more effective
politically.

From: Friam [friam-boun...@redfish.com] on behalf of Nick Thompson
[nickthomp...@earthlink.net]
Sent: Tuesday, January 07, 2014 12:53 AM
To: 'The Friday Morning Applied Complexity Coffee Group

Re: [FRIAM] rational

2014-01-07 Thread John Kennison

I think one example might be I oppose gay marriage because it would undermine 
my own (straight) marriage.
One could interpret the reason given as a false premise (you could always do 
that whenever someone gives a reason for a belief) but here it looks more like 
a rationalization. Of course I can't be certain, but I suspect that the real 
reason is the false premise that homosexuality is a pervasion and a different 
premise is substituted because it was felt to be more effective politically.

From: Friam [friam-boun...@redfish.com] on behalf of Nick Thompson 
[nickthomp...@earthlink.net]
Sent: Tuesday, January 07, 2014 12:53 AM
To: 'The Friday Morning Applied Complexity Coffee Group'
Subject: Re: [FRIAM] rational

Speaking of shoddy reasoning, I wish somebody would give an example of
shoddy reasoning by a Right Winger that was NOT an example of reasoning from
false premises.

n

Nicholas S. Thompson
Emeritus Professor of Psychology and Biology
Clark University
http://home.earthlink.net/~nickthompson/naturaldesigns/

-Original Message-
From: Friam [mailto:friam-boun...@redfish.com] On Behalf Of Steve Smith
Sent: Monday, January 06, 2014 9:28 PM
To: The Friday Morning Applied Complexity Coffee Group
Subject: Re: [FRIAM] rational

Glen sed, responding to what REC sed about what Bob Altemeyer sed:
 Bob Altemeyer's research on right-wing authoritarian (RWA)
 personalities
 -- pdf at http://home.cc.umanitoba.ca/~altemey/
 http://home.cc.umanitoba.ca/%7Ealtemey/ -- finds that high scoring
 RWAs suffer from severe cognitive disabilities which essentially
 render them immune to reason.  (Note that right-wing here is a
 technical term meaning adherent of the status quo.)

  But research reveals that authoritarian followers drive through life
  under the influence
  of impaired thinking a lot more than most people do, exhibiting
  sloppy reasoning,
  highly compartmentalized beliefs, double standards, hypocrisy,
  self-blindness, a
  profound ethnocentrism, and--to top it all off--a ferocious
  dogmatism that makes it
  unlikely anyone could ever change their minds with evidence or
logic.
 Excellent!
I do have a strong sympathy for this description, though I don't fully defer
it to Right Wing only... after a year in Berkeley, a bastion of Left Wing
thinking,  I can say that both wings can fumble their way to an extremism.
For example, try to get anything done that requires either city council or
citizen referendum, and you will get a *real*
taste of Authoritarianism.In Berkeley (to their credit) there was
an annual celebration/parade entitled How Berkeley can you be?  which
allowed the populace to lampoon themselves (or actually, one another) in
a semi-self-aware way that I could only hope for the far Right.   I have
to admit that I can't even imagine what that would look like.
This helps refine reasoning about reasoning in the way that
 worries me.  The idea being that a brain in a vat might still be
 rational in some technical/strict sense of the word.  But that's not
 what normal people mean when they _use_ the word rational.  What
 normal people mean is a combination of the ability to think well and
 be open to multiple options.  It seems like the openness is the
 fulcrum of the concept.
I think that people who I find familiar, comfortable, easy to converse with
do roughly hold that connotation of the term.  And I'm thankful for that.

That said, I present that *most* people (normal or not) mean rational
thought to be thought and descriptions of said thought which is familiar
and aligned with their own thinking.  In that sense, I feel most people
conflate rational thought with the colloquial common sense.
 One of the aspects that worries me most is the _surety_ with which
 most people go about their daily thinking.  But I find this in lots of
 people who would normally be considered quite rational.  To me, it
 doesn't much matter how intelligent one is, or how many facts they may
 claim to have at their fingertips.  What matters is the confidence
 with which they hold their own beliefs.  The more confident you are,
 the _less_ rational you are.
And I go about my daily activities with as much of this form of
_irrationality_ (confidence?) as possible.  Not because I think it is more
defensible or will lead to a better outcome in the moment, than a more open
and thought through (well thought) set of responses, but because A) I can
be hyper self-conscious which can lead to overthinking and getting stuck
and B) because I am aware that my _best self_, my _best problem solver_ is
my self (body/brain/sensorium + extended phenotype (technology mostly) )
when it is highly trained as roughly a learning classifier system... which
requires lots of variation and
testing.My best self _satisfices_ for the immediate problem (good
enough for GubMent work) while _optimizing_ against the long haul.   I
know that by being _confident_ 

Re: [FRIAM] rational

2014-01-07 Thread John Kennison
Nick,  There are several perfectly rational ways in which someone might decide 
to oppose gay marriage. I indicated one way, based on the premise that 
homosexuality is perverse. You indicated another way. A third possibility is 
that the speaker really believes the stated reason that gay marriage would 
threaten traditional marriage. 
Your example of a possible reconstruction of the reasoning is interesting 
because, if correct, it would appear to be highly irrational as the opposition 
to gay marriage seems to be based on homophobia. 

Watching men neck in public makes me uncomfortable
Married people are allowed to neck in public.
Anything that makes me uncomfortable should be banned
Therefore Gay marriage should be banned.

My dilemma (which I imagine you intended) is that I would like to say that the 
above thought process is a good example of irrational thinking but, strangely 
enough, it has the appearance, and structure, of a logical argument. I think I 
would wonder how the supposed speaker would acquire the belief or premise that 
anything that makes me feel uncomfortable should be banned. I doubt that 
right-wingers would feel that you are defending them well if you said that this 
is a typical right wing rationale.

AT any rate, my question would be: Is there a sense in which the above type of 
thinking (based on the premises Nick assumed) is irrational. 









From: Friam [friam-boun...@redfish.com] on behalf of Nick Thompson 
[nickthomp...@earthlink.net]
Sent: Tuesday, January 07, 2014 1:57 PM
To: 'The Friday Morning Applied Complexity Coffee Group'
Subject: Re: [FRIAM] rational

At last.
Ok, so to claim that this is irrational, we have to know the chain of
premises and conclusions that leads to this conclusion.
As you rightly point out, we can supply premises that make the reasoning
look crazy or we can supply premises that make it look reasonable.   But
isn't it bad reasoning to claim that reasoning is bad without having done
that?

I assume that the reasoning behind bans on gay marriage goes something like
this:

Watching men neck in public makes me uncomfortable
Married people are allowed to neck in public.
Anything that makes me uncomfortable should be banned
Therefore Gay marriage should be banned.

Perfectly RATIONAL
N

Nicholas S. Thompson
Emeritus Professor of Psychology and Biology
Clark University
http://home.earthlink.net/~nickthompson/naturaldesigns/

-Original Message-
From: Friam [mailto:friam-boun...@redfish.com] On Behalf Of John Kennison
Sent: Tuesday, January 07, 2014 10:35 AM
To: The Friday Morning Applied Complexity Coffee Group
Subject: Re: [FRIAM] rational


I think one example might be I oppose gay marriage because it would
undermine my own (straight) marriage.
One could interpret the reason given as a false premise (you could always do
that whenever someone gives a reason for a belief) but here it looks more
like a rationalization. Of course I can't be certain, but I suspect that the
real reason is the false premise that homosexuality is a pervasion and a
different premise is substituted because it was felt to be more effective
politically.

From: Friam [friam-boun...@redfish.com] on behalf of Nick Thompson
[nickthomp...@earthlink.net]
Sent: Tuesday, January 07, 2014 12:53 AM
To: 'The Friday Morning Applied Complexity Coffee Group'
Subject: Re: [FRIAM] rational

Speaking of shoddy reasoning, I wish somebody would give an example of
shoddy reasoning by a Right Winger that was NOT an example of reasoning from
false premises.

n

Nicholas S. Thompson
Emeritus Professor of Psychology and Biology Clark University
http://home.earthlink.net/~nickthompson/naturaldesigns/

-Original Message-
From: Friam [mailto:friam-boun...@redfish.com] On Behalf Of Steve Smith
Sent: Monday, January 06, 2014 9:28 PM
To: The Friday Morning Applied Complexity Coffee Group
Subject: Re: [FRIAM] rational

Glen sed, responding to what REC sed about what Bob Altemeyer sed:
 Bob Altemeyer's research on right-wing authoritarian (RWA)
 personalities
 -- pdf at http://home.cc.umanitoba.ca/~altemey/
 http://home.cc.umanitoba.ca/%7Ealtemey/ -- finds that high scoring
 RWAs suffer from severe cognitive disabilities which essentially
 render them immune to reason.  (Note that right-wing here is a
 technical term meaning adherent of the status quo.)

  But research reveals that authoritarian followers drive through life
  under the influence
  of impaired thinking a lot more than most people do, exhibiting
  sloppy reasoning,
  highly compartmentalized beliefs, double standards, hypocrisy,
  self-blindness, a
  profound ethnocentrism, and--to top it all off--a ferocious
  dogmatism that makes it
  unlikely anyone could ever change their minds with evidence or
logic.
 Excellent!
I do have a strong sympathy for this description, though I don't fully defer
it to Right Wing only

Re: [FRIAM] rational

2014-01-07 Thread John Kennison
Good point. I think I have a better understanding of what you have been saying. 
I guess there are two ways of using the word rational --one might be called 
logical (regardless of the crazyness of the premises), the other might be 
reasonable or responsible or something like that.

--John

From: Friam [friam-boun...@redfish.com] on behalf of glen [g...@ropella.name]
Sent: Tuesday, January 07, 2014 6:24 PM
To: The Friday Morning Applied Complexity Coffee Group
Subject: Re: [FRIAM] rational

On 01/07/2014 02:50 PM, John Kennison wrote:
 Watching men neck in public makes me uncomfortable
 Married people are allowed to neck in public.
 Anything that makes me uncomfortable should be banned
 Therefore Gay marriage should be banned.

 AT any rate, my question would be: Is there a sense in which the above type 
 of thinking (based on the premises Nick assumed) is irrational.

Well, we've been discussing exactly this.  If discomfort is a) opaque to
analysis and b) not shared by the audience, then it would fail to meet
some of the definitions of rational we've been discussing.

Similarly, if we adopt the distinction made by Roger's citing of
Altemeyer (or Steve's satisficing for the immediate and optimizing the
distal), then we could say that although the behavior may be beneficial,
it contains gaps in rationality.

But I also think it's flawed in the sense I tried to identify with
libertarians. (Or as Marcus just pointed out.) The dependence between
discomfort and banning behavior is myopic because it _selectively_
considers only a subset of the logical consequences of that premise.

--
⇒⇐ glen


FRIAM Applied Complexity Group listserv
Meets Fridays 9a-11:30 at cafe at St. John's College
to unsubscribe http://redfish.com/mailman/listinfo/friam_redfish.com

FRIAM Applied Complexity Group listserv
Meets Fridays 9a-11:30 at cafe at St. John's College
to unsubscribe http://redfish.com/mailman/listinfo/friam_redfish.com

Re: [FRIAM] rational

2014-01-05 Thread John Kennison
Concerning the statement:

 My intuition tells me that all thinking is rational – it’s just that most of 
 it is weak or founded on truly crazy premises.

I think this is one of the issues to be explored. It seems to work for the 
person who believes that every statement in the bible is literally true. (And 
maybe has a further belief ambiguities and apparent contradictions can be 
resolved by contacting God through prayer.) My own tendency to believe what I 
see seems to require that I don't have hallucinations --or could distinguish 
them from true visual perceptions.

But what about the thinking done by an artist when creating a work of art. Is 
it rational but based on strange axioms, or it is a different type of thinking 
which is non-rational
And if the former, how does the artist come up with the strange hypotheses? 
What about intuition, including the intuition that all thinking is rational but 
possibly with crazy hypotheses?


From: Friam [friam-boun...@redfish.com] on behalf of Nick Thompson 
[nickthomp...@earthlink.net]
Sent: Sunday, January 05, 2014 5:02 PM
To: 'The Friday Morning Applied Complexity Coffee Group'
Subject: Re: [FRIAM] rational

This is the kind of discussion that a Newly Minted Peircean, such as myself, 
should be all over, but I find myself oddly (thankfully?) reticient.  My 
intuition tells me that all thinking is rational – it’s just that most of it is 
weak or founded on truly crazy premises. Among valid inferences, Peirce made a 
distinction between strong inferences (All ravens are black, this bird is a 
raven, this bird is black) and weak ones such as “this bird is a raven, this 
bird is black, all ravens are black” (induction)  and “this bird is black, all 
ravens are black, this bird is a raven”(abduction).   But he regarded all three 
as valid forms of inference.  In this spirit, I might argue that right wing 
thinking is not irrational, but exceedingly weak.   But we should beware of 
falling for the syllogism, “This guy is wrong, all right-wingers are wrong, 
this guy is a right winger” which is valid, but horribly weak.

Nicholas S. Thompson
Emeritus Professor of Psychology and Biology
Clark University
http://home.earthlink.net/~nickthompson/naturaldesigns/

From: Friam [mailto:friam-boun...@redfish.com] On Behalf Of Roger Critchlow
Sent: Sunday, January 05, 2014 12:20 PM
To: The Friday Morning Applied Complexity Coffee Group
Subject: Re: [FRIAM] rational


On Sun, Jan 5, 2014 at 11:41 AM, glen 
g...@ropella.namemailto:g...@ropella.name wrote:
[ ... ]
Now, that carries us to how/whether/why humans would use irrational
inference procedures.  But I think we would _need_ some evidence that
people actually use irrational reasoning procedures.  I think even
so-called irrational things like _emotions_ are, somewhere deep down,
rational.  Those emotions are an evolutionarily selected decision-making
ability that has its own calculus.

Bob Altemeyer's research on right-wing authoritarian (RWA) personalities -- pdf 
at http://home.cc.umanitoba.ca/~altemey/ -- finds that high scoring RWAs suffer 
from severe cognitive disabilities which essentially render them immune to 
reason.  (Note that right-wing here is a technical term meaning adherent of 
the status quo.)

But research reveals that authoritarian followers drive through life under the 
influence
of impaired thinking a lot more than most people do, exhibiting sloppy 
reasoning,
highly compartmentalized beliefs, double standards, hypocrisy, self-blindness, a
profound ethnocentrism, and--to top it all off--a ferocious dogmatism that 
makes it
unlikely anyone could ever change their minds with evidence or logic.

There's an article in today's Times, 
http://www.nytimes.com/2014/01/05/business/media/banished-for-questioning-the-gospel-of-guns.html,
 which unintentionally makes the case that the gun rights lobby is essentially 
a coalition of right-wing authoritarians and gun manufacturers.  They cannot 
tolerate any discussion of the dogma because they are incapable of reasoning on 
the subject, only able to distinguish the party line from apostasy so they can 
attack the enemies.

Just because there is a reason to be a lynch mob doesn't make a lynch mob 
reasonable.  I think you're confounding the rationality of explanation with the 
rationality of the explained.

-- rec --


FRIAM Applied Complexity Group listserv
Meets Fridays 9a-11:30 at cafe at St. John's College
to unsubscribe http://redfish.com/mailman/listinfo/friam_redfish.com


Re: [FRIAM] rational

2014-01-04 Thread John Kennison
I consider myself a rational person because I believe what I observe and I 
believe in what is observed by any group of people I trust (such as a near 
consensus of scientists). I further believe in whatever follows logically. I 
believe I can predict the likely consequences of my actions and this helps make 
me a reasonably happy person. Belief in God or belief in the inerrancy of the 
bible do not pass my tests. But there is scientific evidence that religious 
people are healthier and happier than non-religious people. This seems to be so 
even though people who would apparently be neither healthy nor happy are almost 
always religious. So what should I make of this? 


From: Friam [friam-boun...@redfish.com] on behalf of glen [g...@ropella.name]
Sent: Friday, January 03, 2014 7:42 PM
To: The Friday Morning Applied Complexity Coffee Group
Subject: Re: [FRIAM] rational

On 01/03/2014 03:47 PM, Marcus G. Daniels wrote:
 Or the `successful' may just be apex predators, but still just one of
 many possible species of person.  They feed on the productivity of these
 other species.   Perhaps not wanting to be one of them, the drug addict
 (unconsciously) denies the predator that productivity...  As Arnade
 observes, everyone makes mistakes, so perhaps we can just enumerate the
 wolves and note that's what wolves do but that they get no further honor.

Well, it seems to me that the ascription of honor (or any other
honorific) is a dynamic thing.  Not only is society fickle like that,
but it's also difficult to predict what your arbitrary weirdo might take
_pride_ in. Witness:

   http://www.thesmokinggun.com/buster/man-dies-eating-roaches-587314

or

   http://en.wikipedia.org/wiki/Armin_Meiwes

So, we can't prescribe what honor the wolves get.  In fact, merely
counting them might encourage more people to want to be them.  I think
the answer lies in creating/facilitating wolf-eating species.

--
⇒⇐ glen


FRIAM Applied Complexity Group listserv
Meets Fridays 9a-11:30 at cafe at St. John's College
to unsubscribe http://redfish.com/mailman/listinfo/friam_redfish.com

FRIAM Applied Complexity Group listserv
Meets Fridays 9a-11:30 at cafe at St. John's College
to unsubscribe http://redfish.com/mailman/listinfo/friam_redfish.com

Re: [FRIAM] rational

2014-01-04 Thread John Kennison
Hi Merle,

Your workshop looks like an excellent place to find answers. I live in 
Massachusetts and won't be coming to the workshop, but would appreciate any 
suggested readings. 

  I am interested in two things. One is to explore the conundrum that it seems 
to be rational to sometimes engage in non-rational behavior. How can this be 
the case? Do humans have a need to be irrational at times?  If so, how what is 
the nature of this need? How deep is it? How did it arise? I have some theories 
(hardly original with me) but I first wanted to pose the question in a neutral 
way.

The other question is related, but more practical. What are reasonable ways for 
a supposedly rational people to deal with their apparent non-rational needs? 
Again I have done some personal exploring but didn't want to the question to 
get mixed up with what I take to be my religious outlook.  In any case, I am 
always seeking.

Thanks, John 


From: Friam [friam-boun...@redfish.com] on behalf of Merle Lefkoff 
[merlelefk...@gmail.com]
Sent: Saturday, January 04, 2014 10:54 AM
To: The Friday Morning Applied Complexity Coffee Group
Subject: Re: [FRIAM] rational

John,

If you were attending the Zen Brain workshop at Upaya in Santa Fe (where I 
teach applied complexity in the Buddhlst Chaplaincy program)--along with some 
of the most famous neuroscientists in the world and Neil Theise, a remarkable 
complexity guy--you might find the answer to your question.  The workshop 
starts the end of January.

Merle


On Sat, Jan 4, 2014 at 6:17 AM, John Kennison 
jkenni...@clarku.edumailto:jkenni...@clarku.edu wrote:
I consider myself a rational person because I believe what I observe and I 
believe in what is observed by any group of people I trust (such as a near 
consensus of scientists). I further believe in whatever follows logically. I 
believe I can predict the likely consequences of my actions and this helps make 
me a reasonably happy person. Belief in God or belief in the inerrancy of the 
bible do not pass my tests. But there is scientific evidence that religious 
people are healthier and happier than non-religious people. This seems to be so 
even though people who would apparently be neither healthy nor happy are almost 
always religious. So what should I make of this?


From: Friam [friam-boun...@redfish.commailto:friam-boun...@redfish.com] on 
behalf of glen [g...@ropella.namemailto:g...@ropella.name]
Sent: Friday, January 03, 2014 7:42 PM
To: The Friday Morning Applied Complexity Coffee Group
Subject: Re: [FRIAM] rational

On 01/03/2014 03:47 PM, Marcus G. Daniels wrote:
 Or the `successful' may just be apex predators, but still just one of
 many possible species of person.  They feed on the productivity of these
 other species.   Perhaps not wanting to be one of them, the drug addict
 (unconsciously) denies the predator that productivity...  As Arnade
 observes, everyone makes mistakes, so perhaps we can just enumerate the
 wolves and note that's what wolves do but that they get no further honor.

Well, it seems to me that the ascription of honor (or any other
honorific) is a dynamic thing.  Not only is society fickle like that,
but it's also difficult to predict what your arbitrary weirdo might take
_pride_ in. Witness:

   http://www.thesmokinggun.com/buster/man-dies-eating-roaches-587314

or

   http://en.wikipedia.org/wiki/Armin_Meiwes

So, we can't prescribe what honor the wolves get.  In fact, merely
counting them might encourage more people to want to be them.  I think
the answer lies in creating/facilitating wolf-eating species.

--
⇒⇐ glen


FRIAM Applied Complexity Group listserv
Meets Fridays 9a-11:30 at cafe at St. John's College
to unsubscribe http://redfish.com/mailman/listinfo/friam_redfish.com

FRIAM Applied Complexity Group listserv
Meets Fridays 9a-11:30 at cafe at St. John's College
to unsubscribe http://redfish.com/mailman/listinfo/friam_redfish.com



--
Merle Lefkoff, Ph.D.
President, Center for Emergent Diplomacy
Santa Fe, New Mexico, USA
me...@emergentdiplomacy.orgmailto:me...@emergentdiplomacy.org
mobile:  (303) 859-5609
skype:  merlelefkoff

FRIAM Applied Complexity Group listserv
Meets Fridays 9a-11:30 at cafe at St. John's College
to unsubscribe http://redfish.com/mailman/listinfo/friam_redfish.com

Re: [FRIAM] How do forces work?

2013-04-22 Thread John Kennison
I wonder if Russ's question relates to a point that was raised in another 
thread –one that I tried to follow --unsuccessfully because it was mostly over 
my head. Nick wrote that:

Again, acting in my capacity as the Village Pragmatist, I would assert that 
science is the only procedure capable of producing lasting consensus.  The 
other methods  various forms of torture, mostly ... do not produce such 
enduring results.   N.

My first thought was that we would first need language –without language it is 
hard to imagine what consensus would look like and hard to imagine science. How 
could we say that an experiment disproved a hypothesis, or even that one 
experiment is a repetition of another? But without consensus, how do we get 
language? Maybe science and language develop in tandem, --assuming we are 
programmed to believe that gestures and vocal sounds mean something --which can 
be determined through experimentation. This would explain why science seems to 
start with unsophisticated statements such as Objects tend to fall in a 
downward direction. And why it seems necessary, when grappling with new, 
abstract scientific (and mathematical) ideas to reduce them to simpler 
statements involving ideas we are already comfortable with.  And Russ's 
question might be part of what is needed to understand abstract concepts of 
modern Physics. In 1962 I had a grad course in quantum mechanics (given by the 
Math Dept). It started with a discussion of motion in the physical world and a 
look at some of the questions we would ask. But very soon we adopted the axiom 
that the set of all questions was isomorphic to the set of all closed subspaces 
of a Hilbert space. Even the instructor admitted that this was a bit hard to 
swallow, but once we swallowed all would eventually become clear. I learned a 
lot about operators on a Hilbert space and even got an A in the course, but I 
never connected it to any ideas I had about the physical world. 


From: Friam [friam-boun...@redfish.com] on behalf of Nicholas  Thompson 
[nickthomp...@earthlink.net]
Sent: Sunday, April 21, 2013 3:59 PM
To: 'The Friday Morning Applied Complexity Coffee Group'
Subject: Re: [FRIAM] How do forces work?

I know I am not qualified to join this discussion, but may I say just one thing?

As we struggle with our data from our accelerators n’ stuff, we bring to bear 
models from our experience … metaphors.  The language of your discussion is 
full of such metaphors, and full, also, of expressions of pain that these 
metaphors are not only incomplete  -- all metaphors are incomplete – but that 
they are incompletete in ways that are essential to the phenomena you are 
trying to account for.  Now, it seems to me, that this conversation is like the 
conversation that would ensure if we were to see a unicorn drinking out of the 
fountain at St. Johns, but did not have the mythology of unicorns, or even the 
word, unicorn, to bring to bear.  We would instantly start to apply incomplete 
models.  “It’s a whacking great horse!”  One of us would say.  “Yeah, but, it’s 
got a narwhale tooth sticking out of its forehead.”

Nick

From: Friam [mailto:friam-boun...@redfish.com] On Behalf Of Steve Smith
Sent: Sunday, April 21, 2013 1:40 PM
To: stephen.gue...@redfish.com; The Friday Morning Applied Complexity Coffee 
Group
Subject: Re: [FRIAM] How do forces work?

S -

I'd like to think Gil and I could take credit for running Bruce off with our 
Light/Dark Boson/Lepton nonsensery but I think he's hardier than that!

Carry On!
 - S
Aya, it turns out Bruce recently unsubscribed from FRIAM. I hope you guys on 
the list are happy with your signal to noise ratio ;-)Just kidding...keep 
it up.

Anyway, Bruce, as I had hoped, had a nice response, albeit offlist. If you want 
to respond to this thread, please cc: Bruce. I copy his response below.

//** Bruce Sherwood response offlist
Feynman diagrams give one visualization of forces. In this picture, consider 
two electrons moving near each other. With a calculable probability, one of the 
electrons may emit a photon, the carrier of the electromagnetic interaction, 
and this electron recoils. The other electron absorbs the photon and recoils. 
At least for electric repulsion, this is a nice way to think about the 
interaction, but it has obvious problems for talking about attraction. The 
exchanged photon is a virtual photon which unlike unbound photons has mass. 
At the individual interaction vertices (emission event and absorption event) 
momentum and energy need not be conserved, but for the two-electron system 
momentum and energy are conserved.

For the strong (nuclear) interaction, the interaction carrier is the gluon. It 
is thought that the gravitational interaction is carried by a gravitron but 
we have no direct evidence for this.

The weak interaction is mediated by the W and Z bosons and is so similar to 
electromagnetism that one speaks of the 

Re: [FRIAM] How do forces work?

2013-04-19 Thread John Kennison
Russ,

Before people knew about magnetism, it must have seemed miraculous that two 
stones would spontaneously start to move toward (or away from) each other. Now 
we can say,  Oh, it's just magnetism. But if we think about long enough, we 
may still wonder how two objects can move toward or away from each other. My 
question would be, Does magnetism still seem a bit miraculous, or do you feel 
your question is answered, at least for magnetism? In either case, what would a 
satisfying answer look like?

John


From: Friam [friam-boun...@redfish.com] on behalf of Russ Abbott 
[russ.abb...@gmail.com]
Sent: Friday, April 19, 2013 1:50 PM
To: FRIAM
Subject: [FRIAM] How do forces work?

Yesterday I asked this 
questionhttp://physics.stackexchange.com/questions/61542/how-do-forces-work?noredirect=1#comment123788_61542
 on StackExchange: physics.

Is there a mechanistic-type explanation for how forces work? For example, two 
electrons repel each other. How does that happen? Other than saying that there 
are force fields that exert forces, how does the electromagnetic force 
accomplish its effects. What is the interface/link/connection between the force 
(field) and the objects on which it acts. Or is all we can say is that it just 
happens: it's a physics primitive?

So far, there haven't been any answers that feel satisfying--although, please 
look at them yourselves. One of the comments pointed to a 7 1/2 minute video by 
Feynman, in which he talks around the problem before finally saying he can't 
provide an intuitive explanation. I don't think it was one of his better 
efforts. Does anyone on this list have an answer?

-- Russ Abbott
_
  Professor, Computer Science
  California State University, Los Angeles

  My paper on how the Fed can fix the economy: 
ssrn.com/abstract=1977688http://ssrn.com/abstract=1977688
  Google voice: 747-999-5105
  Google+: 
plus.google.com/114865618166480775623/https://plus.google.com/114865618166480775623/
  vita:  
sites.google.com/site/russabbott/http://sites.google.com/site/russabbott/
  CS Wikihttp://cs.calstatela.edu/wiki/ and the courses I teach
_


FRIAM Applied Complexity Group listserv
Meets Fridays 9a-11:30 at cafe at St. John's College
to unsubscribe http://redfish.com/mailman/listinfo/friam_redfish.com


Re: [FRIAM] This is truly thinking outside the box

2013-04-18 Thread John Kennison


It seems that the complexity of organisms would grow more quickly than 
exponential growth when the complexity is low and les quickly as the complexity 
increases. Here's my analysis. Let us say that an animal of complexity level 1 
evolves to complexity level 2 is T years. How long will the complexity level 2 
animal take to evolve to level 4? If you think of the complexity level 2 animal 
as a union of two unrelated level 1 parts, then each of these parts would grow 
to level 2 (so the union would grow to level 4) in another T years,. But the 
parts are related and this makes it difficult for them to get more complex in 
the same amount of time. If a mutation makes a part more complex and also more 
efficient it still might not be favored by evolution as it might disrupt the 
way one part must relate to the other. In other words the need to maintain a 
relationship between the parts retards the growth of complexity. As complexity 
grows, the retarding effect increases. So if we now observe that it takes 376 
million years to double the complexity, it probably took lees time when the 
animals were simpler.

From: Friam [friam-boun...@redfish.com] on behalf of Douglas Roberts 
[d...@parrot-farm.net]
Sent: Thursday, April 18, 2013 12:27 PM
To: The Friday Morning Applied Complexity Coffee Group
Subject: Re: [FRIAM] This is truly thinking outside the box

A (the only?) downside to not reading every FRIAM post.  Sorry for the dupe.

--Doug

On Thu, Apr 18, 2013 at 10:26 AM, glen e. p. ropella 
g...@tempusdictum.commailto:g...@tempusdictum.com wrote:

Douglas Roberts wrote at 04/18/2013 05:36 AM:
 Thinking along the lines of Moore's law, extrapolating it backwards.  I
 love stories that are told across cosmological time scales:

 http://www.technologyreview.com/view/513781/moores-law-and-the-origin-of-life/

We have to give credit where it's due! ;-)  Roger already posted this.

Roger Critchlow wrote at 04/16/2013 08:44 AM:
 I don't know if retrodicting an exponential growth curve back to
 it's origin is technically an extrapolation, but aside from that
 quibble this is very cute.

 Plot Moore's Law, it hits the origin in the 1960's when there were
 zero transistors on chips.

 A similar process works with scientific publications. Between 1990
 and 1960, they doubled in number every 15 years or so. Extrapolating
 this backwards gives the origin of scientific publication as 1710,
 about the time of Isaac Newton.

 Now make some assumptions about the time of origin of various
 genetic complexities evident in the history of life on earth, and
 plot the growth curve for that. When was its origin?

 http://www.technologyreview.com/view/513781/moores-law-and-the-origin-of-life/

 http://arxiv.org/abs/1304.3381

 -- rec --

--
glen e. p. ropella, 971-255-2847tel:971-255-2847, http://tempusdictum.com
The government solution to a problem is usually as bad as the problem.
-- Milton Friedman



FRIAM Applied Complexity Group listserv
Meets Fridays 9a-11:30 at cafe at St. John's College
to unsubscribe http://redfish.com/mailman/listinfo/friam_redfish.com



--
Doug Roberts
d...@parrot-farm.netmailto:d...@parrot-farm.net
http://parrot-farm.net/Second-Cousins
http://parrot-farm.net/Second-Cousins
505-455-7333 - Office
505-672-8213 - Mobile


FRIAM Applied Complexity Group listserv
Meets Fridays 9a-11:30 at cafe at St. John's College
to unsubscribe http://redfish.com/mailman/listinfo/friam_redfish.com


Re: [FRIAM] Digital Ecology

2013-01-25 Thread John Kennison
I'm a mathematician but I rarely use Mathematica or MatLab (only occasionally, 
for curios)--and don't know what R is. Some mathematicians use these tools in 
teaching, but I'm not that enthusiastic about them --and tend not to teach 
courses where they would be most appropriate. I do use Latex for writing 
papers. Prior to my retirement I sometimes used Latex for tests and handouts, 
but often used the Equations editor in Word, which isn't as good but quicker). 
I have Macs in home and office. Latex and Word run on all sorts of computers.

From: Friam [friam-boun...@redfish.com] on behalf of Owen Densmore 
[o...@backspaces.net]
Sent: Friday, January 25, 2013 11:49 AM
To: The Friday Morning Applied Complexity Coffee Group
Subject: Re: [FRIAM] Digital Ecology

On Fri, Jan 25, 2013 at 7:45 AM, Grant Holland 
grant.holland...@gmail.commailto:grant.holland...@gmail.com wrote:
Owen - Great post. Hope some other folks will respond in kind. Might be 
interesting to get an 'inventory of digital lifestyles'.  - Grant

Sent from my iPhone

Thanks!  Yeah, I sorta was surprised by how interconnected everything is .. not 
just hardware nor even software .. but the overall hive and interaction.  
Thus ecology.

I'm also a bit surprised how much I *don't* do/have. I haven't the slightest 
idea what high quality photographers do.  I've no fancy camera, just the phone. 
 RAW files and hardcopy certainly make a difference in one's DE. I also have no 
game systems (xbox, ps3 ..) and I *know* these devices have come on strong into 
household integration, mainly TV but I think much more than that.

Although I have social accounts, I don't use them, so am not aware how they 
all interact in one's lifestyle.  Doug uses his FB account quite a bit, I know. 
 And G+ gets a lot of use by the digerati.   I'm not sure how intensive use of 
these effect my Digital Ecology but I'm sure they would.

We don't heavily use a car GPS system either, although rented one in Italy last 
visit.  I *think* but not sure that they interact with at least a home computer 
to upgrade maps etc.  I know that there are phone GPS systems that are pretty 
sophisticated to, with add-on hardware/software *TomTom

Thinking about Pamela and others within Friam who write professionally, I 
presumed that would have quite an impact on the DE.  For example, publishers 
likely favor software that is not available on both mac/windows and might, for 
compatibility reasons, impact hardware/OS choice.  In general, work-home 
integration likely molds the DE.  Mathematicians use Mathematica and MatLab and 
R and ...  Scientists need lots of codes, most Fortran, and may need a pretty 
husky computer to run them.

So it does reach into the middle of your lifestyle and impacts the integration 
of all our digital critters.  I am interested in how others see it.

   -- Owen


FRIAM Applied Complexity Group listserv
Meets Fridays 9a-11:30 at cafe at St. John's College
to unsubscribe http://redfish.com/mailman/listinfo/friam_redfish.com


Re: [FRIAM] faith

2012-09-26 Thread John Kennison


There are some things we take so completely on faith that we have great trouble 
even realizing that we are making an assumption. For example, when I open my 
eyes, I take it on faith that I am seeing an actual physical universe, and not 
simply recording impulses that my eyes forwarded to my brain which then refined 
them, etc. 

But religious faith is not at all like that. Religious people often have to 
fight doubts --they often have to attend weekly meetings where they chant or 
sing or pray or perform rituals to keep their faith --they may hear sermons on 
the dangers of backsliding, etc.   



From: friam-boun...@redfish.com [friam-boun...@redfish.com] on behalf of 
Nicholas  Thompson [nickthomp...@earthlink.net]
Sent: Wednesday, September 26, 2012 12:29 AM
To: 'The Friday Morning Applied Complexity Coffee Group'
Subject: Re: [FRIAM] faith

If it is true that,

Russ believes that his old and broken down motorcycle can take him from A
to B, but he doesn't have faith that it will

Can it also be true that Russ doubt whether his ... motorcycle can take him
from A to B?   Is it the case that, on your understanding, doubt and belief
can exist in a person at the same time with respect to the same proposition?


Nick

-Original Message-
From: friam-boun...@redfish.com [mailto:friam-boun...@redfish.com] On Behalf
Of Sarbajit Roy
Sent: Tuesday, September 25, 2012 9:37 PM
To: The Friday Morning Applied Complexity Coffee Group
Subject: Re: [FRIAM] faith

It would take the inverse form

Faith is absolute acceptance whereas Belief is limited/conditional
acceptance.

So Russ may have belief in X without having faith in it.

eg.
Russ believes that his old and broken down motorcycle can take him from A
to B, but he doesn't have faith that it will

On 9/24/12, Nicholas  Thompson nickthomp...@earthlink.net wrote:
 Russ,

 I take your point, but still, I would have a hard time composing a
 sentence of the form,  Russ has faith in X but he doesn't believe in
 it.  Can you compose such a sentence for me?

 N



 From: friam-boun...@redfish.com [mailto:friam-boun...@redfish.com] On
 Behalf Of Russ Abbott
 Sent: Monday, September 24, 2012 12:42 AM
 To: The Friday Morning Applied Complexity Coffee Group
 Subject: Re: [FRIAM] faith



 Nick,



 As I understand your position the words faith and belief are synonyms.
 I
 would prefer a definition for faith that distinguishes it from belief.



 Tory,



 Thanks for  you comment on my posts. I'm glad you enjoy them.



 My definition of faith makes use of the notion of the everyday world.
 But I'm not saying that the everyday world is the same for everyone.
 Your everyday world may be different from mine. I'm just saying that
 believing that the world will continue to conform to your sense of
 what the everyday world is like is not faith; it's simple belief.



 Eric,



 I would take having faith in something in the colloquial sense as
 different from faith in a religious context, which is what I was
 focusing on.




 -- Russ



 On Sun, Sep 23, 2012 at 9:27 PM, Victoria Hughes
 victo...@toryhughes.com
 wrote:



 Russ wrote, in part-



 Faith, I would say (in fact I did earlier)





 is believing something that one wouldn't otherwise believe without faith.





 Believing that the everyday world is the everyday world





 doesn't seem to me to require faith.



 Russ, with all due respect for the enjoyment I get from your posts, I
 find this suspiciously tautological.



 Who are you to define for the rest of humanity (and other sentient
 life
 forms) what 'the everyday world' incorporates? Numerous 'for instance'
 cases
 can immediately be made here. All you can do is define what you
 believe for yourself. You cannot extrapolate what is defensible for
 others to believe, from your own beliefs.



 And this statement ' Faith is believing something that one wouldn't
 believe without faith'. Hm and hm again.



 Eagleman's new book Incognito
 http://www.amazon.com/Incognito-Secret-Lives-David-Eagleman/dp/030738
 9928/r
 ef=sr_1_1?s=booksie=UTF8qid=1348460523sr=1-1keywords=incognito+by+
 david+
 eagleman  offers fruitful information from recent neuroscience that
 eagleman may
 interest others on this list. His ultimate sections bring up hard
 questions about legal and ethical issues in the face of the myriad 'zombie
programs'
 that run most of our behaviour. This looks like - but is not as
 simplistic as - 'yet another pop science book.'



 A review David Eagleman's
 http://www.boston.com/bostonglobe/ideas/brainiac/2011/06/david_eaglem
 ans.ht
 ml Incognito - Brainiac



 Tory


 
 FRIAM Applied Complexity Group listserv Meets Fridays 9a-11:30 at cafe
 at St. John's College lectures, archives, unsubscribe, maps at
 http://www.friam.org






FRIAM Applied Complexity Group listserv
Meets Fridays 9a-11:30 at 

Re: [FRIAM] Unsolved Problems in Psychology

2012-05-19 Thread John Kennison


Yes, I agree with this defense of the concept of universal gravitation. It may 
seem strange to say that objects can instantaneously exert a force on a distant 
object but it enables us to explain and accurately predict motions. 

I have in mind a different type of criticism of Newton's laws of motion, that 
apparently wasn't made but could have been. The criticism would go as follows: 
The laws depend on the notion of force. Since forces are inferred by observing 
motions, why don't we do away with this extraneous idea and simply talk about 
motion. Moreover forces are not only pure fictions, they are wonderful fudge 
factors that explain away the many examples of how the laws of motion fail to 
describe the actual motion we see in the real world. For example, one of the 
laws says an object in motion tends to keep moving at the same speed and in the 
same direction. But this isn't true –a ball rolling on the ground tends to slow 
down and eventually stop. 

A disciple of Newton might reply, The ball only slows down because a force is 
acting on it, in fact the ground exerts a frictional force that accounts for 
the slowing down.  

To which the critic might say, Yeah and I suppose the frictional force is 
calculated as the precise amount needed to account for the discrepancy with the 
law about staying in motion. Similarly, an iron object which is at rest will 
suddenly start to move when a magnet is placed nearby, thus violating another 
'law' of motion. I suppose you will have to postulate a 'magnetic force' to 
account for this discrepancy. Mark my words, if we are ever to have a good 
theory of motion, we will have to do away with fictional concepts such as 
force. 

But of course the concept of force, along with a few provisions about 
calculating gravitational, frictional and magnetic forces, makes it possible to 
state an amazingly predictive theory about a wide range of motions. We are 
willing to accept the fictional notion of force because it leads to a theory 
that can be verified empirically. 

I remember a conversation, decades ago, with a psychologist who was attacking 
Freud for using mystical notions such as the ego, the id, libido etc. The 
psychologist felt his subject needed to eliminate mystical concepts and express 
everything in terms of concrete concepts such as foot-pounds, voltages and 
decibels. After the conversation ended, it occurred to me that the problem with 
Freud was not that he invented fictional notions but that the resulting theory 
did not have anything close to the predictive ability of Newton's theory. I 
wished I had said, You seem willing to accept the fictions of physics, but not 
to accept any fictions for psychology.  


From: friam-boun...@redfish.com [friam-boun...@redfish.com] On Behalf Of Bruce 
Sherwood [bruce.sherw...@gmail.com]
Sent: Friday, May 18, 2012 6:13 PM
To: russ.abb...@gmail.com; The Friday Morning Applied Complexity Coffee Group
Subject: Re: [FRIAM] Unsolved Problems in Psychology

Newton famously said about action at a distance, I frame no
hypotheses. I take this to mean something like the following:

I completely agree with you that I haven't explained gravity. Rather
I've shown that observations are consistent with the radical notion
that all matter attracts all other matter, here and in the heavens,
made quantitative by a one-over-r-squared force 'law'. On this basis I
have shown that the orbits of the planets and the behavior of the
tides and the fall of an apple, previously seen as completely
different phenomena, are 'explainable' within one single framework.

I propose that we provisionally abandon the search for an
'explanation' of gravity, which looks fruitless for now, and instead
concentrate on working out the consequences of the new framework.
Let's leave it as a task for future scientists to try to understand at
a deeper level than 'action-at-a-distance' what the real character of
gravity is. There has been altogether too much speculation, such as
maybe angels push the planets around. Let's get on with studying what
we can.

I think Newton doesn't get nearly enough credit for this radical
standpoint, which made it possible to go forward. And of course we
know that eventually Einstein found a deep 'explanation' for gravity
in terms of the effects that matter has on space itself. There are
hints in the current string theory community of even deeper insights
into the nature of gravity.

Bruce

On Fri, May 18, 2012 at 1:38 PM, Russ Abbott russ.abb...@gmail.com wrote:
 John, I like your gravity question. If this were Google+, I'd click its +1
 button.  My wife, who studies these things, says that one of the
 fiercest contemporary criticisms of Newton's theories was that they depended
 on a mysterious (magical?) action at a distance.

 -- Russ Abbott


FRIAM Applied Complexity Group listserv
Meets Fridays 9a-11:30 at cafe at St. John's 

Re: [FRIAM] Unsolved Problems in Psychology

2012-05-19 Thread John Kennison


Nick,

In college, I majored in Math and minored in Physics. My guess is that your 
points about how warped space is presented are basically right but obviously 
Bruce can respond much more authoritatively.  

On magical thinking, I think that before scientists can start testing out 
hypotheses, they need to set up a vocabulary in which to state these 
hypotheses. One approach to geometry uses Euclid's vocabulary of point, line, 
angle and distance. Euclidean geometry is a mathematical theory, with axioms 
and logical consequences, expressed in this vocabulary. To see if this is of 
any help in navigating the physical world, we have to say precisely what we 
mean when we talk about points, lines, angles and distances in the physical 
world. Once we do that, we can begin to test to see if the conclusions of 
Euclidean geometry are true in the physical world. 

In some sense, before we start talking abstractly about geometric points, it 
would be nice if we first stated precisely what concrete evidence will be taken 
as specifying, say, a point in the physical world. But it doesn't seem easy to 
do this until we have first done some substantial analysis about how points 
etc. would behave if they existed. During the period when we are still 
experimenting with a new vocabulary, it may seem like we are dealing in magic, 
but that vocabulary, suitably refined, may eventually revolutionize how we 
think about the concrete situations we are trying to understand.  

---John 


From: friam-boun...@redfish.com [friam-boun...@redfish.com] On Behalf Of 
Nicholas  Thompson [nickthomp...@earthlink.net]
Sent: Saturday, May 19, 2012 12:34 PM
To: 'The Friday Morning Applied Complexity Coffee Group'
Subject: Re: [FRIAM] Unsolved Problems in Psychology

John,  I haven't yet digested Bruce's comments above, nor entirely what you 
have written here, but I want to clarify one point.



When somebody speaks of space being warped one has in mind one of those 
diagrams where the Cartesian coordinates are bent, right?  In other words, we 
are using our pre-Einsteinian worldview as a frame of reference to describe the 
Einsteinian world.  But the Cartesian world has no reality, right?  It’s a 
figment.  Bent IS straight.  I suppose one could say that Cartesian space is 
the space that would be there if there were nothing in it, or if mega world and 
the micro-world were organized as the meta-world we humans live in is organized.



I don't know where this leaves us with the underlying question of the role, if 
any,  of magical thinking in science.  Is Psychology in trouble because it 
uses magical thinking, or is it in trouble because it uses bad magic? I want to 
think about these questions as a review these posts.  I will be in touch when I 
get back to Massachusetts.



Thanks, everybody.



Nick





-Original Message-
From: friam-boun...@redfish.com [mailto:friam-boun...@redfish.com] On Behalf Of 
John Kennison
Sent: Saturday, May 19, 2012 4:59 AM
To: The Friday Morning Applied Complexity Coffee Group
Subject: Re: [FRIAM] Unsolved Problems in Psychology







Yes, I agree with this defense of the concept of universal gravitation. It may 
seem strange to say that objects can instantaneously exert a force on a distant 
object but it enables us to explain and accurately predict motions.



I have in mind a different type of criticism of Newton's laws of motion, that 
apparently wasn't made but could have been. The criticism would go as follows: 
The laws depend on the notion of force. Since forces are inferred by observing 
motions, why don't we do away with this extraneous idea and simply talk about 
motion. Moreover forces are not only pure fictions, they are wonderful fudge 
factors that explain away the many examples of how the laws of motion fail to 
describe the actual motion we see in the real world. For example, one of the 
laws says an object in motion tends to keep moving at the same speed and in the 
same direction. But this isn't true –a ball rolling on the ground tends to slow 
down and eventually stop.



A disciple of Newton might reply, The ball only slows down because a force is 
acting on it, in fact the ground exerts a frictional force that accounts for 
the slowing down.



To which the critic might say, Yeah and I suppose the frictional force is 
calculated as the precise amount needed to account for the discrepancy with the 
law about staying in motion. Similarly, an iron object which is at rest will 
suddenly start to move when a magnet is placed nearby, thus violating another 
'law' of motion. I suppose you will have to postulate a 'magnetic force' to 
account for this discrepancy. Mark my words, if we are ever to have a good 
theory of motion, we will have to do away with fictional concepts such as 
force.



But of course the concept of force, along with a few provisions about 
calculating gravitational, frictional and magnetic forces, makes it possible

Re: [FRIAM] Unsolved Problems in Psychology

2012-05-18 Thread John Kennison


Eric,

A cannonball shot into the air eventually returns to Earth. In Newtonian 
physics, we say that the cannonball does so because the Earth exerts a force on 
the cannonball which pulls it back down. Would you say this is a magical 
explanation? Why or why not? 

Also, would you say this is an instance of a paradigm at work? 



From: friam-boun...@redfish.com [friam-boun...@redfish.com] On Behalf Of ERIC 
P. CHARLES [e...@psu.edu]
Sent: Friday, May 18, 2012 2:25 AM
To: Russ Abbott
Cc: The Friday Morning Applied Complexity Coffee Group
Subject: Re: [FRIAM] Unsolved Problems in Psychology

Russ,
I am about to get a bit defensive. I'm not sure why I feel the need to defend a 
discipline I am largely disenchanted with, but here it goes:

While I would NOT want to let generally accepted be a criterion for solved, 
I am a bit perturbed by your suspicion that psychology lacks generally accepted 
results.

Psychology has been an academic discipline for over a century, and likely has 
more professional members today than any other academic discipline, especially 
if you count people who do psychology-leaning neuroscience. There are several 
major conferences in psychology that have more than 10,000 attendees. There are 
over 1,000 peer reviewed academic journals in the field. There are at least 10 
major journals dedicated to literature reviews establishing results as 
generally accepted, and several have been operating for over 100 years. For a 
discipline without a dedicated category, psychologists have also garnered a 
pretty impressive number of Nobel Prizes. On what possible basis would you 
think there was not a MASSIVE body of generally agreed upon results?

We don't even have to get to the professional level for evidence: Any 
introductory psychology textbook is full of references to published results 
that are generally accepted. And a standard-size introductory psychology text 
is now around 800 pages long. There are between 12 and 20 standard mid-level 
courses in the field, each with a wide range of textbooks filled with generally 
accepted results.

On what possible basis would you suspect there are few generally accepted 
results, and what could you possibly mean by claiming that any any accepted 
results would probably be 'low level'?

While, as in any science, some percentage of the accepted results will later 
turn out to need revision (sometimes rejection, but more often notes regarding 
required circumstances), there is a lot that psychologists know. The big 
problem in psychology (IMHO) is the lack of a paradigm that effectively 
organizes the accepted results and shows where to seek results in the future.

Eric



On Thu, May 17, 2012 10:19 PM, Russ Abbott russ.abb...@gmail.com wrote:
Perhaps we can approach the question of which problems in psychology have been 
solved by asking which published results are generally accepted. I suspect 
there are quite a few--even if most of them are relatively low level.

-- Russ


On Thu, May 17, 2012 at 6:30 PM, ERIC P. CHARLES e...@psu.edu wrote:
Arlo, I agree completely about the process point.

I was a bit less certain when you said, something difficult about psychology 
is that much of the data has to be collected through someone else - those 
[people] involved in the study

I assume you would consider a person to be part of the physical world, 
treatable in most ways like any other type of object. Yes?  If so, how is your 
statement different than the following,

something difficult about chemistry is that much of the data has to be 
collected through something else - those chemicals involved in the study

Eric

On Thu, May 17, 2012 06:23 PM, Arlo Barnes arlo.bar...@gmail.com wrote:
It seems so far science and tech have been regarded as thing, or adjectives to 
describe 'problem' - whereas I consider them processes (and to a much lesser 
extent philosophies in the) and not necessarily even ones with discrete ends, 
but more a recursive approach - I see a phenomena, I make a 'magic' 
explanation, I collect data on it, and see if the magic matches the data. If 
not, I revise the explanation. If so, I see if it predicts more data. Wash, 
rinse, and repeat. Really we are making rules (that are not perfect and have 
exceptions, and are therefore not 'done') and making more rules that govern the 
exceptions (and those rules also have exceptions). So we have something 
asymptotically approaching whatever objective Truth/reality there is by way of 
infinite regression. Then if we are doing tech, we makes things that take 
advantage of this set of rules and therefore work most of the time.
I think something difficult about psychology is that much of the data has to be 
collected through someone else - those involved in the study.
-Arlo James Barnes.



FRIAM Applied Complexity Group listserv
Meets Fridays 9a-11:30 at cafe at St. John's College
lectures, 

Re: [FRIAM] Unsolved Problems in Psychology

2012-05-17 Thread John Kennison


The Cannonball trajectory problem seems to be solved, but maybe we need to take 
relativity or whatever into consideration for certain cannonballs. Or maybe 
cannonballs will start to behave differently next year (for example if basic 
physical constants can suddenly shift). But we can (I think) disprove the 
roadrunner theory of falling. The important thing about scientific theories is 
that we can imagine ways of disproving them. So what psychological theories 
have been disproven?




From: friam-boun...@redfish.com [friam-boun...@redfish.com] On Behalf Of 
Nicholas  Thompson [nickthomp...@earthlink.net]
Sent: Thursday, May 17, 2012 2:18 AM
To: c...@plektyx.com; 'The Friday Morning Applied Complexity Coffee Group'
Subject: Re: [FRIAM] Unsolved Problems in Psychology

Well, On Peirce’s account (yes I am still reading Peirce) Truth (or “solved”) 
is like “settled law”.  It could come undone any time, but usually doesn’t.   
(Actually, I have that wrong.  Truth is what wouldn’t come undone, but, of 
course, we never live to be sure that that’s what we got.

N

From: friam-boun...@redfish.com [mailto:friam-boun...@redfish.com] On Behalf Of 
Carl Tollander
Sent: Wednesday, May 16, 2012 10:16 PM
To: ERIC P. CHARLES
Cc: friam@redfish.com
Subject: Re: [FRIAM] Unsolved Problems in Psychology

Eric,

Re: 1) humming makes my sinuses happy, generally.

Re: 2) I quite agree, it's not so simple.  Yet, one has to start somewhere, and 
the 'magical thinking' pejoration is, by my lights, kinda simple on the face of 
it.   I don't agree, by any stretch,  that all 'bright minds' are necessarily 
scientists.  Science, as I understand it, is a continuous process of 
intensively figuring out what are the right questions to ask and wondering how 
to interpret such data as one can find or generate.  I do not see that it is 
legitimate, even in science terms, to cast the folks who sincerely tried to 
make sense of their experience as living in cartoons because they did not 
choose to live in the context of one's decades of training in whatever 
discipline.

Re: Is there anything you think is a solved scientific question or do you 
think the category is incoherent?  Yes, since I think science is about 
rigorously evolving questions, yep, the notion of solved scientific questions 
is indeed, at the very least, incoherent.  Which is not at all to imply one 
can't aim one's canon, but that's a different world of discourse.

C

On 5/16/12 9:45 PM, ERIC P. CHARLES wrote:
Well, to make two more general claims then:

1) I am not sure anyone is able to play the game in the order you suggest. Oh, 
some people can hum a few bars, but until you break out specific examples and 
dig into the details of them, it is just humming.

2) The line between a tech problem and a science problem cannot possibly be as 
simple as you suggest. By my read, at one point the trajectory of a cannon ball 
was a scientific question, there was a genuine question of how a cannon ball 
flew, and bright minds - people we would now call scientists - wrestled with 
the possibilities (a startlingly large part of the population still think 
falling works like the roadrunner cartoons). I can't see how you think it is a 
tech problem except in so much as it is a solved question, it is now 
something that it is fairly easy to do tech with it.

Is there anything you think is a solved scientific question or do you think 
the category is incoherent?

Eric



On Wed, May 16, 2012 11:15 PM, Carl Tollander 
c...@plektyx.commailto:c...@plektyx.com wrote:
Eric, so you've got a tech problem, not a science problem, and sure, the tech 
problem of trajectories wrt local gravitation can be solved.  How do I aim 
the cannon (or the canon) and better, how do I metabolize my error when my 
initial notion turns out to be a bit off.  Still, do we understand gravitation 
in the (apparently more general) context of quantum mechanics, well, no.  So 
there again is my worry about the notion of solved a problem, which seems, 
um, problematic.

As to your idea of the game, my text was in reply to Jochen and perhaps 
others who, perhaps, had weighed in on the idea of magical thinking as, 
somehow, a bad thing, rather than Nick's inner universe, specifically.

Carl

On 5/16/12 8:41 PM, ERIC P. CHARLES wrote:
Carl,
My guess is that Nick can't play the game to anyone's satisfaction in the order 
you proposed. He could go down that road, but it will digress endlessly and 
readers will become sad. The only way to have things stay on topic is for 
someone to propose things until they find one Nick thinks has been solved 
and only then will he be able to explain in any satisfactory detail what it 
means (to him) for that particular problem to be solved. If five things are 
found that he thinks are solved, presumably some sort of general rule will 
emerge.

Eric

P.S. To flip the question (and please rename the thread if you take this bait): 
As far 

Re: [FRIAM] [EXTERNAL] Re: Clarifying Induction Threads

2012-03-31 Thread John Kennison


Probability theory gives us an answer if we believe that the coin has heads on 
one side and tails on the other and further that the two sides come up with 
equal probability. These beliefs can be tested empirically, so induction is 
involved.

But there is another issue in the case of a coin that has produced a string of 
heads. As often happens, the simple probability formulas, that might appear in 
an elementary text, assume independence. That is, that whatever happens on the 
first few tosses has is independent of (has no effect on) the outcome of a 
subsequent toss. I have read that for automatic coin tossing machines, this 
seems not to be true. For example, assume the tossing mechanism gathers up the 
coin and tosses it again. The outcome might depend on the position of the coin 
after the first toss. There could be special positions P with the property that 
when the coin is in position P then the tossing mechanism returns it to 
position P (after the toss). This could lead to long strings of heads, more 
likely than what theory would predict, because independence (which the theory 
assumes) would not be true.  This is also the case with a person trying to 
simulate a series of Hs and Ts  that make look like the series of Heads and 
Tails of a fair (and independently functioning coin tossing operation)
If the person guesses a few heads, this influences the person's subsequent 
choices.

--John

From: friam-boun...@redfish.com [friam-boun...@redfish.com] On Behalf Of 
Nicholas  Thompson [nickthomp...@earthlink.net]
Sent: Friday, March 30, 2012 8:39 PM
To: 'The Friday Morning Applied Complexity Coffee Group'
Subject: Re: [FRIAM] [EXTERNAL] Re: Clarifying Induction Threads

Wait a minute, folks.  Aren't we talking about Deduction here.  Our theory that 
the coin should be fair comes not from our experience but from probability 
theory.  To relate coin flipping to induction, don't we have to talk about a 
coin of unknown fairness.  How many heads do we have to flip of such a coin 
before we are led to the conclusion that the coin has two heads.   Isn't that 
the analogue to the problem of inducition?

N

-Original Message-
From: friam-boun...@redfish.com [mailto:friam-boun...@redfish.com] On Behalf Of 
John Kennison
Sent: Friday, March 30, 2012 3:07 PM
To: russ.abb...@gmail.com; The Friday Morning Applied Complexity Coffee Group
Subject: Re: [FRIAM] [EXTERNAL] Re: Clarifying Induction Threads



I agree. People who think that a fair coin is due to come up tails after a 
string of heads are not so much anti-inductivist (or whatever term might be 
used) as naive in applying the rule that over the long run, the percentage 
difference between the the number of heads and number of tails rends towards 
zero.  If we replace a coin by a person trying to give a random sequence of H's 
and T's, the claim that the sequence is due for a T after a string of H's 
might be valid.

---John


From: friam-boun...@redfish.com [friam-boun...@redfish.com] On Behalf Of Russ 
Abbott [russ.abb...@gmail.com]
Sent: Friday, March 30, 2012 1:23 PM
To: The Friday Morning Applied Complexity Coffee Group
Subject: Re: [FRIAM] [EXTERNAL] Re: Clarifying Induction Threads

The naive strategy for predicting coin tosses is anti-reductionist in John 
Kennison's terms. There is even a rationale. We know that in the long run 
(given a fair coin) the number of heads will be approximately the same as the 
number of tails. Therefore, when one count grows larger than the other, predict 
that the other will occur to even things out.  Many people think that way.

-- Russ



On Fri, Mar 30, 2012 at 8:15 AM, Lakkaraju, Kiran 
klak...@sandia.govmailto:klak...@sandia.gov wrote:
T

From: Nicholas Thompson 
[mailto:nickthomp...@earthlink.netmailto:nickthomp...@earthlink.net]
Sent: Friday, March 30, 2012 08:26 AM
To: russ.abb...@gmail.com russ.abb...@gmail.com; 'The Friday Morning Applied 
Complexity Coffee Group' friam@redfish.commailto:friam@redfish.com
Subject: [EXTERNAL] Re: [FRIAM] Clarifying Induction Threads

Hi, Russ,

I will answer between the lines below.  Two cautions: I am rushing to Friam and 
these are Peirce’s views, not necessarily mine. :

From: friam-boun...@redfish.commailto:friam-boun...@redfish.com 
[mailto:friam-boun...@redfish.commailto:friam-boun...@redfish.com] On Behalf 
Of Russ Abbott
Sent: Thursday, March 29, 2012 9:55 PM
To: The Friday Morning Applied Complexity Coffee Group
Subject: Re: [FRIAM] Clarifying Induction Threads

Nick,

Would you explain further your notion that belief is that on which we act.
[NST ==]  Please be clear:  I am trying to channel Peirce, here, although, as 
you already know, I am, myself, a behaviorist.
 Does this imply for you that a lack of action implies a lack of belief? For 
example, I believe that the earth is round, but I don't act on that belief. 
Does that mean I do't really have that belief? How does

Re: [FRIAM] [EXTERNAL] Re: Clarifying Induction Threads

2012-03-30 Thread John Kennison


I agree. People who think that a fair coin is due to come up tails after a 
string of heads are not so much anti-inductivist (or whatever term might be 
used) as naive in applying the rule that over the long run, the percentage 
difference between the the number of heads and number of tails rends towards 
zero.  If we replace a coin by a person trying to give a random sequence of H's 
and T's, the claim that the sequence is due for a T after a string of H's 
might be valid.

---John


From: friam-boun...@redfish.com [friam-boun...@redfish.com] On Behalf Of Russ 
Abbott [russ.abb...@gmail.com]
Sent: Friday, March 30, 2012 1:23 PM
To: The Friday Morning Applied Complexity Coffee Group
Subject: Re: [FRIAM] [EXTERNAL] Re: Clarifying Induction Threads

The naive strategy for predicting coin tosses is anti-reductionist in John 
Kennison's terms. There is even a rationale. We know that in the long run 
(given a fair coin) the number of heads will be approximately the same as the 
number of tails. Therefore, when one count grows larger than the other, predict 
that the other will occur to even things out.  Many people think that way.

-- Russ



On Fri, Mar 30, 2012 at 8:15 AM, Lakkaraju, Kiran 
klak...@sandia.govmailto:klak...@sandia.gov wrote:
T

From: Nicholas Thompson 
[mailto:nickthomp...@earthlink.netmailto:nickthomp...@earthlink.net]
Sent: Friday, March 30, 2012 08:26 AM
To: russ.abb...@gmail.com russ.abb...@gmail.com; 'The Friday Morning Applied 
Complexity Coffee Group' friam@redfish.commailto:friam@redfish.com
Subject: [EXTERNAL] Re: [FRIAM] Clarifying Induction Threads

Hi, Russ,

I will answer between the lines below.  Two cautions: I am rushing to Friam and 
these are Peirce’s views, not necessarily mine. :

From: friam-boun...@redfish.commailto:friam-boun...@redfish.com 
[mailto:friam-boun...@redfish.commailto:friam-boun...@redfish.com] On Behalf 
Of Russ Abbott
Sent: Thursday, March 29, 2012 9:55 PM
To: The Friday Morning Applied Complexity Coffee Group
Subject: Re: [FRIAM] Clarifying Induction Threads

Nick,

Would you explain further your notion that belief is that on which we act.
[NST ==]  Please be clear:  I am trying to channel Peirce, here, although, as 
you already know, I am, myself, a behaviorist.
 Does this imply for you that a lack of action implies a lack of belief? For 
example, I believe that the earth is round, but I don't act on that belief. 
Does that mean I do't really have that belief? How does this work in your 
framework?
[NST ==]  In Peirce’s framework, exactly!  Although, come to think of it, I 
cannot think where he has worried about the problem of latent beliefs …. i.e., 
I would do X if Y, but Y hasn’t happened yet.  Given his emphasis on the long 
term in his definition of truth, I think he would probably take such 
dispositions as equivalent to the fact.

Also, you say that[NST ==]  PEIRCE SAYS! true propositions are those on which 
the human species as a community of inquiry will ultimately (in the infinite 
future) converge. That bothers me in a number of ways.

 *   This is such a remote definition that it hardly seems worth bothering with 
in the first place. There is no operational test to determine whether something 
is the truth under this definition.
[NST ==] YEP!
As you say, this is really just a definition, but it's not even a definition of 
one concept in terms of others. It simply labels the propositions on which ... 
 as the truth.  But giving a label to a set of things doesn't add anything 
new. It just adds a label. So why bother?
[NST ==] Well, I think there is a lot of sneaky meta-induction going in in 
Peirce.  Our communities of systematic inquiry have done well for us in the 
past and so I infer that they will do well for us in the future.
 And especially why other when the term the truth has some meaning to most 
people. Why say that we should forget about that meaning and use the term as a 
label for some set of things -- if that set even exists (see below).

 *   One doesn't know that it will ever refer to anything.  We won't converge 
on anything in the infinite future since there is no end to the infinite future 
and hence no convergence.
 *   If you are using converge as in mathematics as in the convergence of an 
infinite series (where there is convergence to infinity), then, as in 
mathematics, one may be able to determine at this moment what the series (or 
truth) will converge to. But that's probably not what you (or Peirce) have in 
mind. But if you reject that, then as in the previous point, there is no 
convergence since one never reaches infinity. So there is no such set.
[NST ==] Well, Peirce was a sophisticated mathematician and I am not, so far 
be it from me ….. .  But, I would say his view was something like communities 
of systematic inquiry are more likely to be closer to the truth at any point in 
than those who fix opinion by any other method.

 *   Some series don't ever converge. How 

Re: [FRIAM] Taking things on faith

2012-03-28 Thread John Kennison


Thanks for your response, Doug. Unitarianism used to consider itself a 
Christian religion, with the exception of its rejection of the trinity and of 
the concept that Jesus is God. The Christian flavoring now competes with 
humanist, Buddhist and pagan flavors. You can take your choice.  

In this country, the Unitarians merged with the Universalists who argue that a 
loving God wouldn't send anyone to Hell. This tends to focus the religion on 
the here and now rather than on a preparation for a successful afterlife. 

From: friam-boun...@redfish.com [friam-boun...@redfish.com] On Behalf Of 
Douglas Roberts [d...@parrot-farm.net]
Sent: Wednesday, March 28, 2012 7:45 AM
To: The Friday Morning Applied Complexity Coffee Group
Subject: Re: [FRIAM] Taking things on faith

I suppose if one felt a compelling need to subscribe to a Christian-flavored 
religion, one could do worse than the Unitarians.  At least the Unitarians 
don't require knowledge of a hierarchy of secret handshakes which are necessary 
to gain access to those decreasing-diameter inner circles of the club.  Unlike 
certain other, unnamed religions.

--Doug


On Wed, Mar 28, 2012 at 2:11 AM, John Kennison 
jkenni...@clarku.edumailto:jkenni...@clarku.edu wrote:
Sometimes religious leaders like to point out that intellectually, we need 
starting points, such as induction. So, faith in God, for example, is just one 
possible starting point. Other religious leaders say that faith is not an 
intellectual starting point, but an attitude that helps us be happy.
Consider a belief in the existence of the physical universe. It's 
philosophically respectable to not hold this belief, but even philosophers 
might need something like it in order to drive their cars effectively
   Another example of the difference, consider two people, A and B, who belong 
to the same fundamentalist church and both believe in God and the inerrancy of 
the bible. But A's faith means that she wakes up each morning feeling like she 
lives in the palm of God's hand just as surely as she feels that she is 
surrounded by world of physical objects. B's faith is intellectual. He feels 
that his church places nearly intolerable restrictions on his freedom, but it's 
best to go along with them to avoid spending eternity in hell instead of 
heaven. A's faith makes her happy just as B's faith makes him unhappy. (I 
didn't decide to make A female and B male, until I noticed how I was using the 
pronouns.)

One (of many) Unitarian-Universalist views of the problem can be found in the 
following. I don't know if I agree with every detail, but basically I like it:

http://clf.uua.org/quest/2010/10/morales.html


From: friam-boun...@redfish.commailto:friam-boun...@redfish.com 
[friam-boun...@redfish.commailto:friam-boun...@redfish.com] On Behalf Of 
Douglas Roberts [d...@parrot-farm.netmailto:d...@parrot-farm.net]
Sent: Tuesday, March 27, 2012 4:54 PM
To: The Friday Morning Applied Complexity Coffee Group
Subject: Re: [FRIAM] Just as a bye-the-way

Thanks, Nick, you describe an interesting way of establishing a life-view.

Not quite sure how to answer, except to say that if I have faith in anything, 
it is in evidence.  If I have accrued a sufficient pile of evidence that 
supports a conclusion about some observation, then I'll probably believe it.

If my collected evidence is such that the inescapable conclusion is that 
nothing is constant, then I suppose I'd eventually come around to believe that, 
so long as I had a constant framework from which to corroborate and verify the 
inconsistencies.  Otherwise, I'd continue to look for the missing pieces of the 
puzzle (a reference to the cosmological artifacts I sent you earlier).

As to religion:  for me it's a big No thank you to any cult mindthink that 
requires brainless acceptance of a supernatural homo-centric 
benevolent/malevolent boogyman. And that goes double for one particular cult 
whose belief system is predicated upon hieroglyph-inscribed disappearing 
golden tablets.  Oh, and I guess that goes triple for any cult that attempts to 
dictate what kind of skivies I must wear to become a member of the club.  I 
guess you could say that it would take a miracle to get me to assent to 
becoming a member of any of the existing flocks of theist-following sheep out 
there.

In retrospect, I suppose I do have faith in one other fairly immutable quality 
-- the accuracy of my bullshit detector.

--Doug


On Tue, Mar 27, 2012 at 11:47 AM, Nicholas Thompson 
nickthomp...@earthlink.netmailto:nickthomp...@earthlink.netmailto:nickthomp...@earthlink.netmailto:nickthomp...@earthlink.net
 wrote:
Dear Doug,

I am afraid that the black hole example is already too technologically dense 
for me, so I am going to punt on the project of luring you inside my walls and 
slaughtering you there, and just out-right tell you what I think
.
The argument began with my detecting

Re: [FRIAM] FW: See this?

2012-03-17 Thread John Kennison


Sarbajit,

Thanks for explaining the four points. Combining religion and science is tricky 
--both are searches for truth but religion has the additional burden, I think, 
of being a search for meaning too. My father was a math professor and in an act 
of rebellion, I thought I might major in physics instead of math. But I 
gravitated back to math and became a math professor like my dad. I was more 
successful in rebelling against my Baptist upbringing and am now a 
Unitarian-Universalist.

John

From: friam-boun...@redfish.com [friam-boun...@redfish.com] On Behalf Of 
Sarbajit Roy [sroy...@gmail.com]
Sent: Saturday, March 17, 2012 2:06 PM
To: The Friday Morning Applied Complexity Coffee Group
Subject: Re: [FRIAM] FW: See this?

Of course you are correct.

If the Mother is X1+X2, and the Father is X3+Y, I seem to recall
vaguely that the Mother's X contribution is essentially a string of
snippets from  X1 and X2, whereas the Father contributes either a pure
X3 or a pure Y to the Child.

If my recollection is correct, then this leads us to the 4th point
Godhood of Father

Sarbajit

On 3/17/12, Nicholas  Thompson nickthomp...@earthlink.net wrote:
 Thanks, Sarbajit,

 One quibble:

 a child is the genetic sum of its parents

 If we are talking genetic tokens (as opposed to types), a child has half the
 genes of each of its parents.

 N


 -Original Message-
 From: friam-boun...@redfish.com [mailto:friam-boun...@redfish.com] On Behalf
 Of Sarbajit Roy
 Sent: Friday, March 16, 2012 9:33 PM
 To: The Friday Morning Applied Complexity Coffee Group
 Subject: Re: [FRIAM] FW: See this?

 John,

 wrt statement #2

 IF our ancestors are contained within us AND live (on) in us, THEN all
 the information we have is in our ancestors too. {Life as an information /
 communication problem}

 Of course we can be more than the sum of our parents. The information is
 already out there in the wild/cloud, we are just downloading it onto our
 genetic hard drives at an increasingly faster biological rate.

 To clarify with an example.

 In the early 1980's I coded boot sector computer virii. These code strings
 would infect by attaching themselves to theend of a copy
 of another executable program (which may have already been infected by code
 strings by some other hacker - and not only at the end but perhaps also
 inserted in the middle). The actual application software (say
 pacman.exe) would continue to run until the competing information strings
 being injected / infected clashed and caused it to die.

 Similarly, a child is the genetic sum of its parents (and through them the
 ancestors)  and information strings (via culture / television / parent et.al
 ) which attach itself to the child's memory (memes).

 Sorry, if I'm somewhat vague/unclear - buts its not easy reconciling
 religion and science.

 Sarbajit

 On 3/17/12, John Kennison jkenni...@clarku.edu wrote:

 Sarbajit,
 Thanks for the explanation. I was thinking of genes as hereditary units
 but
 I guess they can also refer to any chemical strings of a certain type.
 How
 about statement (2)? Can't we be more than the sum of our ancestors?
 --John
 
 From: friam-boun...@redfish.com [friam-boun...@redfish.com] On Behalf
 Of Sarbajit Roy [sroy...@gmail.com]
 Sent: Friday, March 16, 2012 2:22 PM
 To: The Friday Morning Applied Complexity Coffee Group
 Subject: Re: [FRIAM] FW: See this?

 Lets take those points 1 by 1

 1) Information is transmitted genetically.

 a) Instead of information being transmitted as am electronic series
 (string) of 0s and 1s (ie. base 2 encoding), its transmitted as a
 chemical series (string) of base 4 proteins, both series being
 readable.

 b) The statement does not imply that information cannot be transmitted
 by books or converstaions or culture or upbringing etc.

 c) The 19th century reference is probaby with reference to experiments
 by Sir Jagdish Chandra Bose, who did some work on what would be termed
 nowadays as memory RNA (involving plants and not planaria soup).

 d) Data such as blue eyes are transmitted (imperfectly) genetically
 onto copies using GCTA, just as I suppose a colour photocopier does
 using CMYK.

 Sarbajit

 On 3/16/12, John Kennison jkenni...@clarku.edu wrote:


 Yes, sometimes scientific theories resemble religions and vice-versa
 and sometimes the debate on how genes evolve looks a bit like a
 battle between competing religions.

 I would disagree with principles (1) and (2): As for (1) I sometimes
 find that knowledge is transmitted via books or conversations or even
 lectures but none of these transmissions seem to be genetic. As for
 (2) we are not the sum of our ancestors because we are affected by
 our upbringing, our culture, our education etc. (I don't see how
 statement (2) could have been proven in the nineteenth century.)

 As for (3) and (4), I'm not certain what they mean. Can someone
 explain them to me?

 --John

Re: [FRIAM] FW: See this?

2012-03-16 Thread John Kennison
 evolutionary history as a way to understand human behavior.





 -Original Message-
 From: friam-boun...@redfish.com [mailto:friam-boun...@redfish.com] On Behalf
 Of John Kennison
 Sent: Tuesday, March 13, 2012 6:51 AM
 To: The Friday Morning Applied Complexity Coffee Group
 Subject: Re: [FRIAM] FW: See this?





 Hi Nick,



 I understand that you are irked by the phrase genetic greed but I am not
 clear about why this phrase irks you. Here are several possible reasons:

 (1)  Genes are not capable of being greedy.

 (2)  Genetic greed suggests that evolution is largely a competition
 between genes thus overlooking the competition  between groups.

 (3)  Genetic greed overlooks that genes often compete by inducing
 cooperative attitudes rather than greedy ones.

 (4)  You disagree with the statement that, evolution does not
 operate to benefit the group.

 (5)  You disagree with Hamilton's equation.

 (6)  You think that sociobiology sucks.

 Am I on the right track with any of these reasons?

 --John

 

 From:  mailto:friam-boun...@redfish.com friam-boun...@redfish.com
 [friam-boun...@redfish.com] On Behalf Of Nicholas  Thompson
 [nickthomp...@earthlink.net]

 Sent: Sunday, March 11, 2012 6:08 PM

 To: 'The Friday Morning Applied Complexity Coffee Group'

 Subject: Re: [FRIAM] FW: See this?



 Dear Frank



 I am in a rain engulfed open plan, bay-side,  house with 5 other adults and
 two kids, and many competitors for the one copy of the new Yorker, and for
 the space to rethink what I wrote.  So it may be some time before I can get
 you a proper response.  In the meantime, here is an improper one.



 My explicit beef was with the interviewer, not with Wilson.  It is certainly
 news to Wilson that, having believed something dumb for decades, he now
 comes, in old age, to the obvious truth.  But why is it news to us?!   The
 news, it seems to me, that there were a few people who stood up to the
 deluge of Reagen-biology that saturated the field, and it is to THOSE
 people, not Wilson, that we should look for insight.



 I am not sure there IS redemption for an academic who has killed off many
 good ideas (and presumably graduate students) to make a towering academic
 career, and then sees the truth in his dotage.  At least, he has to do more
 than just change he mind.  He has to make restitution:  hasto pay back his
 royalties and recompense damages  to those whom he has  injured.  And
 probably all the other items in the 12 step list, as well.



 Worse than the belated discovery of the truth, is the belated discovery of
 foolishness.   Perhaps the most dramatic instance of this was Donald
 Griffin, who after a career of tough minded neurophys, woke one day as a
 mentalist.



 Oh was that ugly.



 Nick



 From:  mailto:friam-boun...@redfish.com friam-boun...@redfish.com
 mailto:[mailto:friam-boun...@redfish.com]
 [mailto:friam-boun...@redfish.com] On Behalf Of Frank Wimberly

 Sent: Sunday, March 11, 2012 1:58 PM

 To: 'The Friday Morning Applied Complexity Coffee Group'

 Subject: Re: [FRIAM] FW: See this?



 But, Nick, later in the article it says, .even as Wilson campaigned for
 sociobiology, he began to grow dismayed with the scientific framework that
 made it possible.  'I noticed that the foundations of inclusive fitness were
 crumbling,'  Wilson says.  'The reasoning that had convinced me it was
 correct no longer held.'  For instance, after pursuing Hamilton's
 haplodipoidy hypothesis, scientists discovered that many of the most
 cooperative insect species, such as termites and  ambrosia beetles, weren't
 actually haplodiploid.  Furthermore, tens of thousands of species  that did
 manifest haplodiploidy never evolved eusociality-although these insects were
 closely related, they didn't share food or serve the queen.[Wilson]
 concluded that inclusive fitness was no longer a tenable concept.



 Didn't he redeem himself by your lights?



 Frank



 Frank C. Wimberly

 140 Calle Ojo Feliz

 Santa Fe, NM 87505



  mailto:wimber...@gmail.com%3cmailto:wimber...@gmail.com
 wimber...@gmail.commailto:wimber...@gmail.com
 mailto:wimbe...@cal.berkeley.edu%3cmailto:wimbe...@cal.berkeley.edu
 wimbe...@cal.berkeley.edumailto:wimbe...@cal.berkeley.edu

 Phone:  (505) 995-8715  Cell:  (505) 670-9918



 From:  mailto:friam-boun...@redfish.com%3cmailto:friam-boun...@redfish.com
 friam-boun...@redfish.commailto:friam-boun...@redfish.com
 mailto:[mailto:friam-boun...@redfish.com]
 [mailto:friam-boun...@redfish.com]
 mailto:[mailto:friam-boun...@redfish.com]
 mailto:[mailto:friam-boun...@redfish.com] On Behalf Of Nicholas Thompson

 Sent: Sunday, March 11, 2012 11:10 AM

 To: 'The Friday Morning Applied Complexity Coffee Group'

 Subject: Re: [FRIAM] FW: See this?



 Owen, etc.,



 Even after having been carefully instructed by the young concerning how to
 access my new yorker subscription on the web, the best

Re: [FRIAM] FW: See this?

2012-03-16 Thread John Kennison

Sarbajit,
Thanks for the explanation. I was thinking of genes as hereditary units but I 
guess they can also refer to any chemical strings of a certain type.   How 
about statement (2)? Can't we be more than the sum of our ancestors?
--John

From: friam-boun...@redfish.com [friam-boun...@redfish.com] On Behalf Of 
Sarbajit Roy [sroy...@gmail.com]
Sent: Friday, March 16, 2012 2:22 PM
To: The Friday Morning Applied Complexity Coffee Group
Subject: Re: [FRIAM] FW: See this?

Lets take those points 1 by 1

1) Information is transmitted genetically.

a) Instead of information being transmitted as am electronic series
(string) of 0s and 1s (ie. base 2 encoding), its transmitted as a
chemical series (string) of base 4 proteins, both series being
readable.

b) The statement does not imply that information cannot be transmitted
by books or converstaions or culture or upbringing etc.

c) The 19th century reference is probaby with reference to experiments
by Sir Jagdish Chandra Bose, who did some work on what would be termed
nowadays as memory RNA (involving plants and not planaria soup).

d) Data such as blue eyes are transmitted (imperfectly) genetically
onto copies using GCTA, just as I suppose a colour photocopier does
using CMYK.

Sarbajit

On 3/16/12, John Kennison jkenni...@clarku.edu wrote:


 Yes, sometimes scientific theories resemble religions and vice-versa and
 sometimes the debate on how genes evolve looks a bit like a battle between
 competing religions.

 I would disagree with principles (1) and (2): As for (1) I sometimes find
 that knowledge is transmitted via books or conversations or even lectures
 but none of these transmissions seem to be genetic. As for (2) we are not
 the sum of our ancestors because we are affected by our upbringing, our
 culture, our education etc. (I don't see how  statement (2) could have been
 proven in the nineteenth century.)

 As for (3) and (4), I'm not certain what they mean. Can someone explain them
 to me?

 --John
 
 From: friam-boun...@redfish.com [friam-boun...@redfish.com] On Behalf Of
 Sarbajit Roy [sroy...@gmail.com]
 Sent: Friday, March 16, 2012 1:09 AM
 To: The Friday Morning Applied Complexity Coffee Group
 Subject: Re: [FRIAM] FW: See this?

 W.r.t to your pointwise comments to John's points.
 This to me seems a clear case of reinventing the wheel.
 It also seems that the inventors do not know that the wheel has been
 invented.

 Referring to at least 5,000 years of evolved human history
 http://brahmo.org/brahmoism-genetics-memetics.html
 There is at least 1 religion (yes religion and not science) which
 holds as follows:

 # 1) Information / knowledge is transmitted genetically (this was
 experimentaly proveable in 19th century and is trivial to prove today)
 # 2) That we are the sum of our ancestors
 # 3) That we contain all our ancestors in our genes and our bodies and
 within us
 # 4) Godhood of father.

 What is curious is that this belief (or variations) seems to span
 many leading cultures separated by time and distance, and is used as a
 device to propagate an idea or belief .

 I apologise for not being able to state the proposition in the formal
 manner/practice of Judeo-Christian Western civilisation

 Sarbajit

 On 3/16/12, Nicholas  Thompson nickthomp...@earthlink.net wrote:
 Hi, everybody,

 Am I the only person that the FRIAM server mucks with the head of?



 Anyway, the following was sent in response to John Kennison's interesting
 set of questions concerning my gripes about the E. O. Wilson interview.
 Yet, John never got it and it does not, so far as I can see, appear in the
 FRIAM archive.



 So, here it is again, in case anyone else missed it.



 From: Nicholas Thompson [mailto:nickthomp...@earthlink.net]

 Sent: Tuesday, March 13, 2012 10:23 AM

 To: 'The Friday Morning Applied Complexity Coffee Group'

 Subject: RE: [FRIAM] FW: See this?



 Thanks for writing, John.



 You missed the most important objection.  Genes are not the object of
 greed.
 They are not analogous to coins, in reverse.   With a nickel, it makes a
 difference whether it came from your pocket or mine.  With genes, it only
 makes a difference which coin is in the pocket, not who put it there.
 Genes
 are all about type, not token.



 Comments on your specific points below:



 JK: I understand that you are irked by the phrase genetic greed but I am
 not clear about why this phrase irks you. Here are several possible
 reasons:



 (1)  Genes are not capable of being greedy.



 [NST ==] Greed is a behavior pattern.  An individual genes just makes a
 protein or tells another gene when to make a protein.  Gene's can't vary
 their behavior in telic ways.



 JK:(2)  Genetic greed suggests that evolution is largely a
 competition between genes thus overlooking the competition  between
 groups.



 [NST ==]  Well, as I suggested above, you are missing Wilson

Re: [FRIAM] [sfx: Discuss] Fwd: FlowingData - Best statistics question ever

2011-10-29 Thread John Kennison

Self-referential statements can lead to paradoxes, so one could say the 
question is not well-formed because it is self-referential.

 If, as mentioned, choice (C) were 0%, and options (A),(B),(D) were unchanged, 
then the question leads to a paradox. 

If choice (D) were 50%, and options (A), (B), (C) were unchanged, then both 25% 
and 50% would be consistent answers --so (A), (B) and (D) would all be 
defensible (but, obviously, they cannot all be correct).

As it stands the answer appears to be 0% since every choice leads to a 
contradiction, but I would prefer the answer that the question is not 
well-formed. 

From: friam-boun...@redfish.com [friam-boun...@redfish.com] On Behalf Of Carl 
Tollander [c...@plektyx.com]
Sent: Saturday, October 29, 2011 12:08 PM
To: The Friday Morning Applied Complexity Coffee Group
Subject: Re: [FRIAM] [sfx: Discuss] Fwd: FlowingData - Best statistics question 
ever

Imagine it's not multiple choice...

On 10/29/11 9:44 AM, Owen Densmore wrote:
Oops fat fingered earlier email.  I think this, as Tyler sez, is tricky because 
of the double 25.  You have a 50% chance of 25, but only 25% of the other two.  
Like the Monty Hall, I'd like to hear a pro reason through to the answer.

On Sat, Oct 29, 2011 at 9:39 AM, Owen Densmore 
o...@backspaces.netmailto:o...@backspaces.net wrote:


On Sat, Oct 29, 2011 at 9:12 AM, Tyler White 
tylerwhitedes...@gmail.commailto:tylerwhitedes...@gmail.com wrote:
The solution depends on how you consider the answers...  you can say that there 
are four unique answers (A, B, C, D) or you could say there are only 3 answers 
(25%, 50%, 60%).  It's a trick question!  Hahahah

Tyler White¹
http://TylerWhiteDesign.com
http://twitter.com/Uberousful




FRIAM Applied Complexity Group listserv
Meets Fridays 9a-11:30 at cafe at St. John's College
lectures, archives, unsubscribe, maps at http://www.friam.org



FRIAM Applied Complexity Group listserv
Meets Fridays 9a-11:30 at cafe at St. John's College
lectures, archives, unsubscribe, maps at http://www.friam.org


Re: [FRIAM] Math education

2010-12-07 Thread John Kennison


I approached this talk on Math Education as a skeptic -I have always thought 
that the idea of letting the computer do all the work sounds great but is 
flawed. Of course, I don't like the idea of presenting math in the schools as 
mainly rules of calculation, but I feared that using calculators wouldn't be 
much better. If a student is asked to find out how much it costs in all to buy 
a hammer for $23 and a pair of pliers for $17 and if he hits the multiply 
button instead of the add button will he realize that $391 is simply 
impossible? Most people with a traditional math education will realize this 
immediately (I hope) but if a person taught that numbers are nothing more than 
things that come out of a calculator might not see any problem with an answer 
of $391. The traditional approach does provide some intuition about numbers. 
Even worse, the student who pushes the wrong button and gets marked wrong, 
might see math as a boring subject where you have to be so very careful about 
pushing the right buttons. Not unlike the student who only learns meaningless 
rules of calculation who is bored by the need to be so very careful about using 
the rules precisely.

I was somewhat relieved when I actually listened to the talk. He actually said 
that mental arithmetic could be useful and he outlined a bold approach to math 
that I would applaud. My fear remains that if his program were adopted, the 
ambitious parts of it, which I like, would only be given lip service, while the 
message would get through to the schools that math is the same except without 
the drudgery.



On 11/28/10 2:06 AM, Pieter Steenekamp piet...@randcontrols.co.za wrote:



I found the TED talk on math education at 
http://www.ted.com/talks/conrad_wolfram_teaching_kids_real_math_with_computers.html
 very interesting.

In summary, this guy says that our math education is wrong. He defines math 
broadly as the the process of (1) translating a problem to a mathematical form; 
(2) deciding what result is required mathematically; (3) doing the computation; 
and (4) interpreting the result. His point is that math education focuses on 
doing the computation by hand whilst that could be done very easily by 
computer. He reckons math education should focus on the points 1,2  4 and let 
the computer do the computation.



FRIAM Applied Complexity Group listserv
Meets Fridays 9a-11:30 at cafe at St. John's College
lectures, archives, unsubscribe, maps at http://www.friam.org

Re: [FRIAM] Peer review

2010-12-04 Thread John Kennison


But would referees need some incentive to do a good job of reviewing a paper? 
If we only go by economic motivation, and if our pay goes down the longer we 
take, why not do a rush job? If reviewers are paid, shouldn't their work be 
evaluated? Perhaps it is. Editors make note of who does a good review --but we 
would need formal feedback to the reviewers. Who would review the reviewers' 
work?  

From: friam-boun...@redfish.com [friam-boun...@redfish.com] On Behalf Of 
lrudo...@meganet.net [lrudo...@meganet.net]
Sent: Saturday, December 04, 2010 8:07 PM
To: 'The Friday Morning Applied Complexity Coffee Group'
Subject: Re: [FRIAM] Peer review

I am told that in economics these days, some journals
do pay referees (which I presume means peer reviewers)
an honorarium that diminishes by some set amount every
day from the receipt of the paper (not dipping below $0,
though; that *would* get my attention).

This might be an Academic Urban Legend, however.
And I don't really like to *talk* to economists...
it always makes me feel poor, nasty, brutish, and
short-tempered.

 Russell,

 Money for doing peer reviews!?  Oh, gosh.  If the world were thus!

 Nick

 -Original Message-
 From: friam-boun...@redfish.com [mailto:friam-boun...@redfish.com] On Behalf
 Of Russell Standish
 Sent: Saturday, December 04, 2010 3:31 PM
 To: The Friday Morning Applied Complexity Coffee Group
 Subject: [FRIAM] Peer review

 On Fri, Dec 03, 2010 at 10:18:26AM -0800, glen e. p. ropella wrote:
 
  On a tangent, however, I found this article interesting:
 
  Citizens Against Peer Review
  http://blogs.discovermagazine.com/intersection/2010/12/03/citizens-aga
  inst-peer-review/
 
 
  But it does bring up the point that we humans do as little work as we
  can get away with.  We're lazy.  We won't dig into any subject unless
  we must, for whatever reason.  The reviewers will dig in deeper than
  the lay person (mostly) because it's their job/profession to do so.
  Oh sure, they may have chosen that job/profession based on some
  inherent energy or curiosity about the domain; but in the end, they
  have groceries to buy on the way home, yards to rake, burnt out light
  bulbs to change, etc.  So, they really do have to commit to work like
 this.
 

 I weas fine with this, until I got to this bit. No scientist will do peer
 review for the sake of paying bills. In fact it seems to be the fashion not
 to do any work for peer reviewing, and just make snap judgements, as it
 takes you away from the 'real science' (ie writing research grant proposals
 to lure the grad students). They'll do it because they're fundamentally
 interested in science, and want to give back to the scientific community by
 returning the courtesy some other reviewer has given them. But career
 scientists don't, so the peer review process is often just a waste of time,
 or sometimes even positively catty. Sorry for the snarky comments :(.

 --

 
 Prof Russell Standish  Phone 0425 253119 (mobile)
 Mathematics
 UNSW SYDNEY 2052   hpco...@hpcoders.com.au
 Australiahttp://www.hpcoders.com.au
 

 
 FRIAM Applied Complexity Group listserv
 Meets Fridays 9a-11:30 at cafe at St. John's College lectures, archives,
 unsubscribe, maps at http://www.friam.org


 
 FRIAM Applied Complexity Group listserv
 Meets Fridays 9a-11:30 at cafe at St. John's College
 lectures, archives, unsubscribe, maps at http://www.friam.org




FRIAM Applied Complexity Group listserv
Meets Fridays 9a-11:30 at cafe at St. John's College
lectures, archives, unsubscribe, maps at http://www.friam.org


FRIAM Applied Complexity Group listserv
Meets Fridays 9a-11:30 at cafe at St. John's College
lectures, archives, unsubscribe, maps at http://www.friam.org


Re: [FRIAM] Real-world genetic algorithm example... help!

2010-07-10 Thread John Kennison
Selecting for productive coops rather than productive hens might reject highly 
productive, highly aggressive hens in favor of somewhat less productive, 
considerably less aggressive hens who would leave their coop-mates in peace 
(and therefore able to produce more eggs). Such hens need not have anything 
like a “concept” of loyalty to the coop  --we could redistribute these hens to 
different coops and without affecting coop productivity. But after a while, we 
might find we are selecting for highly productive, potentially highly 
aggressive hens who are strongly inhibited against bothering a hen they grew up 
with. Then if we redistributed the hens among different coops, coop 
productivity would decrease.

Are there applications to genetic algorithms? It shows you have to be careful 
about dividing the task to be done into subtasks. You don’t want to overlook an 
algorithm for doing one subtask that provides useful byproducts for another 
subtask. Instead of selecting for each subtask separately, you might select for 
teams of algorithms that do the whole task.


From: friam-boun...@redfish.com [friam-boun...@redfish.com] On Behalf Of Russ 
Abbott [russ.abb...@gmail.com]
Sent: Saturday, July 10, 2010 2:50 AM
To: The Friday Morning Applied Complexity Coffee Group
Subject: Re: [FRIAM] Real-world genetic algorithm example... help!

It's not a good example as an illustration of GA because (1) the selection 
mechanism to move from one generation to the next is essentially select the 
best and shake it up. At best you might call that elitism plus mutation. But it 
is not representative of GA. (2) it has no explicit representation of the 
genome (3) there are no explicit genetic operators applied to one or more 
parents to produce children.

The issue of whether there is mutation points out that there is no coop genome 
that is being evolved. Since there is no coop genome, it's hard to say that 
there is or is not mutation. It certainly isn't a good illustration of mutation 
for a textbook.

You might make the case that the coop genome is the collection of the chicken 
genomes and that the offspring coop genome is generated from the parent coop 
genome by breeding the chickens. I guess you could call that mutation of the 
coop genome.So the mutation operator on the parent coop genome is to breed the 
chickens to get a new coop genome. But I think that's about as far as you could 
push it.

If I were forced to describe this in GA terms, I would say that the coop genome 
is the sequence, in some arbitrary order, of chicken genomes. To get an 
offspring, take a coop genome and treat the segments that correspond to 
individual chickens as separate genomes, mate them to get offspring, and then 
concatenate the genomes of the resulting offspring to get a new coop genome. 
I've never heard of a genetic operator like that, but I guess that doesn't mean 
you couldn't claim it as a genetic operator.

The bottom line for me though is that the experiment is great biology, but it's 
a pretty limited and confusing example of a GA.


-- Russ Abbott
__

  Professor, Computer Science
  California State University, Los Angeles

  cell:  310-621-3805
  blog: http://russabbott.blogspot.com/
  vita:  http://sites.google.com/site/russabbott/
__



On Fri, Jul 9, 2010 at 11:12 PM, Ted Carmichael 
teds...@gmail.commailto:teds...@gmail.com wrote:
Ha!  I knew someone would complain about that.

First of all, Eric is correct: the main point of the story - beyond a nice, 
illustrative example of how a GA works - is the need to properly define a 
fitness function.  In the case of individual chickens, the fitness function was 
ill-defined and didn't work very well.  In particular, this section points out 
that it is not necessary to know why a good solution is good.  Why doesn't have 
to come into it ... the fitness function simply ensures that the best solution, 
no matter what the reasons are for being the best, can emerge from this process.

In regards to Russ' complaints, I'm not sure I can agree that no 
crossover/mutation occurred.  I haven't read the original paper yet, just the 
Huff Post treatment, so I didn't realize that the chicken clusters weren't 
mixed.  That is, I just assumed that more than one cluster was selected among 
the best, and that they collectively produced the subsequent generations.

However, consider the case of mutation.  Russ says there is no mutation within 
the population elements - the clusters of chickens.  But functionally, there 
actually is mutation.  This becomes obvious when we remember that a 
second-generation chicken coop is different from the first-generation coop.  
The genes were all there, but some of them weren't expressed ... that is, they 
simply combined together in a different way to produce a different coop.  It 
doesn't matter that the kids have all the genes of the parents 

[FRIAM] Virtual-world genetic algorithm example... help!

2010-07-10 Thread John Kennison

I am reminded of two conflicting reports I got from two friends about an 
attempt to evolve a sorting program. One friend reported that it was 
discouraging. The evolved programs never were reliable and they took all kinds 
of time and had many superfluous features. The only way to actually get an 
algorithm that worked was to have a sorting method in mind then give the 
program more survival credit the more it mimicked the program in mind. 
 Another friend reported that the attempt was a phenomenal success. 
A program evolved which sorted lists perfectly and efficiently and which was 
unlike any known sorting algorithm, In fact, no on could figure out what the 
program was doing; the only reason they felt it most be theoretically correct 
was that it sorted a huge number of lists perfectly every time.
Can any of you tell me which friend is giving a more accurate 
account? (It is possible that the accounts refer to different experiments and 
are both accurate. The pessimistic account was told to me about 10 years ago, 
the other account recently.)



FRIAM Applied Complexity Group listserv
Meets Fridays 9a-11:30 at cafe at St. John's College
lectures, archives, unsubscribe, maps at http://www.friam.org


Re: [FRIAM] Virtual-world genetic algorithm example... help!

2010-07-10 Thread John Kennison


Thanks Russ. depending on the primitives chosen, this could be more in line 
with the pessimistic account. Putting in the swapping primitive seems like 
aiming for the simple sort which keeps on swapping until it can't be done 
anymore.

Do you know of any evolutionary process which produced a highly efficient and 
previously unknown sorting algorithm?

---John 

From: friam-boun...@redfish.com [friam-boun...@redfish.com] On Behalf Of Russ 
Abbott [russ.abb...@gmail.com]
Sent: Saturday, July 10, 2010 8:32 PM
To: The Friday Morning Applied Complexity Coffee Group
Subject: Re: [FRIAM] Virtual-world genetic algorithm example... help!

I've had both experiences. The successful version had a couple of advantages. 
It had more useful primitives and a more useful fitness function. I don't 
remember the details, but a primitive that says swap adjacent cells if one is 
less that the other helps a lot!  A fitness function that counts the number of 
elements out of place is much less useful than one that measures the extent to 
which the result is ordered, e.g., how many elements are on the correct side of 
their neighbors.

The bottom line is that there has to be a path from the initial primitives to 
the goal in which each step has increasing fitness. If you've got that an 
evolutionary process should get there. If not, it probably won't.


-- Russ



On Sat, Jul 10, 2010 at 1:22 PM, John Kennison 
jkenni...@clarku.edumailto:jkenni...@clarku.edu wrote:

I am reminded of two conflicting reports I got from two friends about an 
attempt to evolve a sorting program. One friend reported that it was 
discouraging. The evolved programs never were reliable and they took all kinds 
of time and had many superfluous features. The only way to actually get an 
algorithm that worked was to have a sorting method in mind then give the 
program more survival credit the more it mimicked the program in mind.
Another friend reported that the attempt was a phenomenal success. 
A program evolved which sorted lists perfectly and efficiently and which was 
unlike any known sorting algorithm, In fact, no on could figure out what the 
program was doing; the only reason they felt it most be theoretically correct 
was that it sorted a huge number of lists perfectly every time.
   Can any of you tell me which friend is giving a more accurate 
account? (It is possible that the accounts refer to different experiments and 
are both accurate. The pessimistic account was told to me about 10 years ago, 
the other account recently.)



FRIAM Applied Complexity Group listserv
Meets Fridays 9a-11:30 at cafe at St. John's College
lectures, archives, unsubscribe, maps at http://www.friam.org



FRIAM Applied Complexity Group listserv
Meets Fridays 9a-11:30 at cafe at St. John's College
lectures, archives, unsubscribe, maps at http://www.friam.org


Re: [FRIAM] It's competition time!

2010-07-09 Thread John Kennison


I bet those brains would be good on toast, like calf brains.


From: friam-boun...@redfish.com [friam-boun...@redfish.com] On Behalf Of 
Douglas Roberts [d...@parrot-farm.net]
Sent: Thursday, July 08, 2010 10:51 PM
To: The Friday Morning Applied Complexity Coffee Group
Subject: Re: [FRIAM] It's competition time!

Mmmm.  Brains.  Big, bursting brains.  Yummy.

On Thu, Jul 8, 2010 at 8:44 PM, Robert Holmes 
rob...@holmesacosta.commailto:rob...@holmesacosta.com wrote:
$5000 for IEEE data-mining contest on predicting traffic jams: 
http://tunedit.org/challenge/IEEE-ICDM-2010

C'mon guys, SF Complex and Friam are *bursting* with brains. Surely we're a 
shoe-in for this. Go Team Santa Fe! Yay!

-- R



FRIAM Applied Complexity Group listserv
Meets Fridays 9a-11:30 at cafe at St. John's College
lectures, archives, unsubscribe, maps at http://www.friam.org


Re: [FRIAM] Real-world genetic algorithm example... help!

2010-07-09 Thread John Kennison


I believe it is referred to in the book Unto Others by David Sloan Wilson and 
Eliot Sober.



From: friam-boun...@redfish.com [friam-boun...@redfish.com] On Behalf Of Ted 
Carmichael [teds...@gmail.com]
Sent: Friday, July 09, 2010 5:34 AM
To: The Friday Morning Applied Complexity Coffee Group
Subject: [FRIAM] Real-world genetic algorithm example... help!

Dear all,

I'm trying to find reference to a story I read some time ago (a few years, 
perhaps?), and I'm hoping that either: a) I heard it from someone on this list, 
or b) someone on this list heard it, too.

Anyway, it was a really cool example of a real-world genetic algorithm, having 
to do with chickens.  Traditionally, the best egg-producing chickens were 
allowed to produce the offspring for future generations.  However, these new 
chickens rarely lived up to their potential.  It was thought that maybe there 
were unknown things going on in the clusters of chickens, which represent the 
actual environment that these chickens are kept in.  And that the high 
producers, when gathered together in these groups, somehow failed to produce as 
many eggs as expected.

So researchers decided to apply the fitness function to groups of chickens, 
rather than individuals.  This would perhaps account for social traits that are 
generally unknown, but may affect how many eggs were laid.  In fact, the 
researchers didn't care what those traits are, only that - whatever they may be 
- they are preserved in future generations in a way that increased production.

And the experiment worked.  Groups of chickens that produced the most eggs were 
preserved, and subsequent generations were much more productive than with the 
traditional methods.

Anyway, that's the story.  If anyone can provide a link, I would be very 
grateful.  (As I recall, it wasn't a technical paper, but rather a story in a 
more accessible venue.  Perhaps the NY Times article, or something similar?)

Thanks!

-Ted


FRIAM Applied Complexity Group listserv
Meets Fridays 9a-11:30 at cafe at St. John's College
lectures, archives, unsubscribe, maps at http://www.friam.org


Re: [FRIAM] Palenque, Chichen Itza and Katyn

2010-05-02 Thread John Kennison


Just a note --I've never liked that old saw about the black cow 
--mathematicians are always working with generalizations. Scientists are the 
ones who claim to be working strictly with observations.

From: friam-boun...@redfish.com [friam-boun...@redfish.com] On Behalf Of 
sarbajit roy [sroy...@gmail.com]
Sent: Friday, April 30, 2010 1:15 PM
To: The Friday Morning Applied Complexity Coffee Group
Subject: Re: [FRIAM] Palenque, Chichen Itza and Katyn

Hi

Where would one place say the Taj Mahal in the context of the Mughal
(alternatively Indo-Islamic) civilisation's ascendancy? Or the Eiffel
Tower in the context of French (alternatively Gallic) civilisation?

Mr Sabloff's observation reminds me of the old saw, A priest, a
scientist and a mathematician are travellng on a train through France
and spot a black cow in the distance. Priest: Behold God hath colored
all cows black. Scientist: You're wrong, only some cows are black.
Mathematician: Somewhere in a field in France there exists at least
one cow at least one of side of which is black..

On 4/29/10, Pamela McCorduck pam...@well.com wrote:


 Grandiosity of civilizations is easily observed but that same
 grandiosity
 applies to Buildings architecture as well as death circuses.
 The Human need for Grand Gestures may be at the root of civilization.

 Jerry Sabloff, the president of the Santa Fe Institute, whose
 specialty is the archaeology (and thus the life) of everyday Mayan
 civilization, gave a little talk in late December to a small group
 where he mentioned in passing that the great architectural monuments
 of a civilization are nearly always erected early in that
 civilization's ascendancy--the Egyptian pyramids, the Mayan ziggurats,
 etc.


FRIAM Applied Complexity Group listserv
Meets Fridays 9a-11:30 at cafe at St. John's College
lectures, archives, unsubscribe, maps at http://www.friam.org


FRIAM Applied Complexity Group listserv
Meets Fridays 9a-11:30 at cafe at St. John's College
lectures, archives, unsubscribe, maps at http://www.friam.org


Re: [FRIAM] Schroedinger's What is Life?

2010-04-27 Thread John Kennison

Suppose there are C cups of water in the ocean and each cup has M molecules of 
water. Then there are CM molecules of water in the ocean. One cupful, or M 
molecules, are marked, so M out of MC, or 1 out C, molecules in the ocean are 
marked. In a cup of water taken at random (i.e. from a well-stirred ocean) 
there would still be about 1 out of C marked molecules so about M/C  molecules 
per cup. 

In the case at hand, M/C is 1000 so each random cup has about 1000 marked 
molecules. If we examined a large number of such cups they would have a mean of 
1000 marked molecules wit a standard deviation of the square root of 1000 
(between 31 and 32). Their distribution would be (for all practical purposes) 
normal. a cup with no marked molecules would be 31 standard deviations from the 
mean and such events essentially don't happen. Even a cup with only 100 marked 
molecules would be (almost) 28 standard deviations from the mean. So virtually 
all cups of water drawn at random will have at least 100 molecules   

From: friam-boun...@redfish.com [friam-boun...@redfish.com] On Behalf Of 
Nicholas Thompson [nickthomp...@earthlink.net]
Sent: Monday, April 26, 2010 11:50 PM
To: friam@redfish.com
Subject: [FRIAM] Schroedinger's What is Life?

All,

I am working my way through this book, and, rather than write one huge email 
that nobody reads, I thought I would write some short ones that somebody might 
read.

It's a splendid little book, very cleanly and economically written.  S. is not 
beset with jargonophilia.  The basic idea of the book (correct if wrong, 
please) is that living systems are orderly systems  that fight off disorder 
with order.   Although  written many years before the double helix, he is 
struck by the fact that the elemental particles of genetic inheritance are so 
very small that their absense of vulnerability to quantum processes is next to 
miraculous.

Right now I just have questions, so I will start with a question.

S. writes, channelling Lord Kelvin:

 Suppose that you could mark the molecules in a glass of water; then pour the 
contents of the glass into the ocean and stir the latter thoroughly so as to 
distrubute the marked molecules uniformly through out the seven seqs;  if then 
you took a glass of water anywhere out ot the ocean, you wound find in it about 
a hundred of your marked molecules.

I am sorry this HAS to be wrong.  However many molecules there are in a glass, 
there are a gazillion glasses of water in the ocean, and isnt the probability 
of coming up with any part of any one of them, vanishingly small.

Ok, work it out, thompson:  There are, apparently, 8x 10^21 cups of water in 
the ocean.  and 8 x 10^24 molecules in each cup.  Which means to this former 
english major that there are a thousand times as many molecules of water in the 
glass as there are glasses of water in the ocean  in the ocean.  So, my chance 
of drawing any one of the hundred marked molecules by chance is one in a 
thousand, right?


Nicholas S. Thompson
Emeritus Professor of Psychology and Ethology,
Clark University (nthomp...@clarku.edumailto:nthomp...@clarku.edu)
http://home.earthlink.net/~nickthompson/naturaldesigns/
http://www.cusf.org [City University of Santa Fe]






FRIAM Applied Complexity Group listserv
Meets Fridays 9a-11:30 at cafe at St. John's College
lectures, archives, unsubscribe, maps at http://www.friam.org


Re: [FRIAM] Why are there theorems?

2010-04-25 Thread John Kennison

Russ,

The natural numbers can be described by listing a few axioms for the notion of 
successor (or the next whole number after this one or the operation of 
adding one) so, in some sense it is a very simple system. Yet all of 
mathematics can, in some sense be coded into statements bout the natural 
numbers. Propositions can  e given Godel numbers and methods of deduction 
reduced to simple arithmetic operations. So there are, in some sense, as many 
theorems about the natural numbers as there are in all of math.

Your question reminded me of the article The unreasonable effectiveness of 
mathematics --which I only know by title. I have never read it, but I have 
referred to it. Maybe it's time for me to look at it --and the Lorenz article 
too.

--John



From: friam-boun...@redfish.com [friam-boun...@redfish.com] On Behalf Of Russ 
Abbott [russ.abb...@gmail.com]
Sent: Sunday, April 25, 2010 5:45 PM
To: The Friday Morning Applied Complexity Coffee Group
Subject: Re: [FRIAM] Why are there theorems?

Too many interesting comments to follow up. But to Lee, Friam probably doesn't 
forward attachments. I didn't get the article with your earlier message either. 
 There is an entry in the Stanford Encyclopedia of Philosophy on Evolutionary 
Epistemologyhttp://plato.stanford.edu/entries/epistemology-evolutionary/. It 
seems from first glance that it makes sense.  We -- and all animals -- evolve  
epistemological capabilities that improve survival. At that level it seems 
almost tautological.

Secondly, your answer to my

question about finding a characterization of the domains to which 
mathematical thinking is successfully applied, namely, that they are (or at 
least necessarily include) the domains for which effective [i.e., 
mathematical] thinking promotes species survival in the external world in 
which we have evolved.

seems to be contradicted by your own good ideas. Does knowledge about the 
domains to which they apply promote species survival? (They certainly promote 
individual survival as a successful mathematician, but that's another matter.)

Does knowledge generated by any so-called pure science promote species 
survival? Only by chance, it seems. Besides why should improved species 
survival be related to the possibility of interesting theorems?  The importance 
to us of a domain is certainly a function of its role in species survival. But 
why does that suggest that the domain is likely to give rise to sophisticated 
mathematics? I don't see the connection.


-- Russ



On Sun, Apr 25, 2010 at 2:07 PM, Eric Smith 
desm...@santafe.edumailto:desm...@santafe.edu wrote:
(expressions of ignorance to follow:)

I wonder in all this whether there is anything interesting to be said
by looking at the relation of syntax to semantics in mathematics,
perhaps not in the sense of applying language, but rather in the
sense of recognizing that mathematics shares syntactic elements with
other constructs, which are more primitive than either, and have to do
with applying formal descriptions to models of onesself.

To be less random and cryptic (with luck):

1. We perform repetitive operations all the time, so our actions
  embody the inductive aspect of the natural numbers in some vague
  sense.  But the natural numbers as a formal construct come into
  existence when we represent addition-by-one as a syntactic
  operation.  (Here showing my ignorance of what Conway, Knuth, and
  other number theorists do to show how real all these formalisms
  are.)  The point was, one is never supposed to ask what comes
  after Z in the alphabet, while the transition to realizing that
  one must ask what comes after 26, sometime between three and four
  years of age, is the human transition to understanding
  arithmetic, which chimps and monkeys never make, even though they
  share some of the quantity-sense that is part of the semantic
  dimension of arithmetic.

2. So now we have the natural numbers as syntactic as well as semantic
  constructs.  Why isn't that all, or why isn't every consequence of
  it immediately available to us?

2a. [Back to behavior] We break collections into groups all the time,
  and we compare groups for equivalence.  Again, operationally, our
  actions embody (en-corp-orate) multiplication and division.  When
  the natural numbers have been created, they present an opportunity
  for us to do that to them, too.  I think of that opportunity as a
  semantically created thing.  Once numbers exist, we can do to them
  the same things we do to other objects, because they exist in a
  representation that allows us to think of them as objects.

2b. But grouping and comparing groups of numbers may not yet be
  multiplication and division.  Those become parts of arithmetic when
  they are assigned a syntactic representation, so that operations
  are well-defined without reference to their semantic antecedents,
  if I understand the goal of Russell and Whitehead, with 

Re: [FRIAM] invitation + introduction

2010-04-11 Thread John Kennison

Thanks very much for the references. A basic reference for much of what is in 
my papers would be 

Stone Spaces
Peter Johnstone
Cambridge University Press, Cambridge, 1982

The toposes I work with are all categories of sheaves over locales. (See my 
2006 paper for some details, Johnstone (above) for more. 

For general topos theory, there are:

Michael Barr  Charles Wells
Toposes, Triples and Theories
Springer-Verlag, New York, 1985  (Also available on Mike Barr's web page (I 
believe)

Also
Topos Theory
Peter Johnstone
Academic Press, London, 1977 (Out of print)

I'll hunt up other references to Toposes.

--John


From: friam-boun...@redfish.com [friam-boun...@redfish.com] On Behalf Of Leigh 
Fanning [le...@versiera.net]
Sent: Sunday, April 11, 2010 1:07 AM
To: The Friday Morning Applied Complexity Coffee Group
Subject: Re: [FRIAM] invitation + introduction

On 09 Apr 2010 at 04:47 PM, John Kennison related

 Leigh,

 Is there a more complete title for the Dynamical System Theory in Biology 
 volume from 1970? Is it a journal? or a series? It does sound interesting. 
 Thanks.


This is a book.

Dynamical System Theory in Biology
Volume I:  Stability Theory and Its Applications
Wiley-Interscience, 1970

There is a second volume:

Foundations in Mathematical Biology
Volume II:  Cellular Systems
Academic Press, 1972

The second volume was edited by Rosen, and he contributed three chapters.  
Other contributors include
Michael Arbib, James Beck, and Aldo Rescigno.

These are out of print.  Alabris might have a copy, or a library.

Leigh


FRIAM Applied Complexity Group listserv
Meets Fridays 9a-11:30 at cafe at St. John's College
lectures, archives, unsubscribe, maps at http://www.friam.org


FRIAM Applied Complexity Group listserv
Meets Fridays 9a-11:30 at cafe at St. John's College
lectures, archives, unsubscribe, maps at http://www.friam.org


Re: [FRIAM] invitation + introduction

2010-04-10 Thread John Kennison


Thanks, Grant and Owen, for the votes of confidence. Concerning complex 
adaptive systems, I would have to define a CAS in such a way that it can be 
interpreted in any topos --then see if we can analyze CAS's by working in a 
topos.

Currently I am working on finding cycles. The idea is that we have a system 
which can be in different states. Let S be the set of all states that the 
system can be in. Let t:S to S be a transition function so that if the 
system is now in state x, then, in the next time unit, it will be in state 
t(x). I then look for cycles (such as t(a) = b, t(b) = c, t(c) = a, so that 
t^3(a)=t(t(t(a)))=a --or, more generally, states x for which t^n(x)=x for some 
n, where t^n(x) = t(t(t( t(x))  iterating t n times. Then I can map the 
system in the best possible way into a topos where it becomes cyclic, meaning 
that for every x there is some n with t^n(x)=x. So n would be a whole number in 
the topos, but whole numbers can jump around and be 3 in some places and 5 in 
other places, etc.

Just exploring this set-up has occupied me since 2001, and I have published 3 
papers on it in the TAC (a web-based journal).

I'll say more and put it in a PDF file, so I  can arrows and exponents and keep 
tabbing and spacing the way I intended it.

---John  




From: friam-boun...@redfish.com [friam-boun...@redfish.com] On Behalf Of Grant 
Holland [grant.holland...@gmail.com]
Sent: Friday, April 09, 2010 5:29 PM
To: The Friday Morning Applied Complexity Coffee Group
Subject: Re: [FRIAM] invitation + introduction

John,

I love such clarity - as expressed in your explanation of category theory. My 
reaction is Oh, so THAT's what category theory is! Thanks for taking the time 
to explain.

Grant

John Kennison wrote:

Owen
Thanks for asking the question. In my answer, below, I describe the technical 
terminology impressionistically. If you want more precision, the Wikipedia 
articles are usually pretty good at giving precise definitions, along with some 
sense of the underlying ideas.

Category theory claims to be a formalization of how mathematics actually works. 
For example, consider the following mathematical structures, which have been 
defined in the 19th and 20th centuries:
Groups =   “sets with a notion of multiplication”
Rings   =“sets with notions of both multiplication 
and addition”
Linear Spaces =  “sets in which vector operations can be defined”
Topological Spaces = “sets with a notion of limit”
Each structure has a corresponding notion of a structure-preserving function:
   Group homomorphism = “function f for which f(xy) = f(x)f(y)”
   Ring homomorphism = “function f for which f(xy)=f(x)f(y) and 
f(x+y)=f(x)+f(y)”
   Linear map = “function preserving operations such as scalar mult: 
f(kv)=kf(v)”
   Continuous function = “function f for which f(Lim x_n) = Lim(f(x_n)”

A category consists of a class of objects, together with a notion of 
“homomorphism” or “map” or “morphism” between these objects. The main operation 
in a category is that morphisms compose (given a morphism from X to Y and 
another from Y to Z, there is then a composite morphism from X to Z).
Examples of catgeories:
 Objects = Groups;Morphisms = Group 
Homomorphisms
 Objects = Rings;   Morphisms = 
Ring Homomorphisms
Objects = Linear spaces; Morphisms = Linear maps
Objects = Top’l spaces;   Morphisms = Cont. 
functions
Objects = Sets;  Morphisms = 
Functions
(The above examples are respectively called the categories of groups, of rings, 
of linear spaces, of top’l spaces, and of sets.)

The claimed advantages of using categories are:
(1) The important and natural questions that mathematicians ask are 
categorical in nature –that is they depend not on operations such as group 
multiplication, but strictly on how the morphisms compose. (that is, the 
objects are like black boxes, we don't see the limits or multiplication inside 
the box, we only see arrows, representing morphisms going from one box to 
another.)
(2) Looking at a subject from a category-theoretic point of view sheds 
light on what is really happening and suggests new research areas.
(3) Proving a theorem about an arbitrary category can have applications to 
all of the traditional categories mentioned above.
(4) As would be expected, there are suitable mappings between categories, 
called functors, which enable us to compare and relate different parts of 
mathematics.
I work in topos theory which ambitiously proposes to study where logic comes 
from. We start by noting that many ideas in logic are closely tied to the 
category of sets.
For example the sentence “x  3” is true for some values of x and not 
for others

Re: [FRIAM] invitation + introduction

2010-04-10 Thread John Kennison


Grant,

My papers are found at http://www.tac.mta.ca/tac/ in Vol 22 [2009] No 14 and 
Vol 16 [2006] No 17 
and Vol 10 [2002] No 15. (There are also some other papers I wrote with Mike 
Barr and Bob Raphael and some on DE's but these are not on dynamical systems.)

I'm working on the PDF paper.

--John

From: friam-boun...@redfish.com [friam-boun...@redfish.com] On Behalf Of Grant 
Holland [grant.holland...@gmail.com]
Sent: Saturday, April 10, 2010 10:46 AM
To: The Friday Morning Applied Complexity Coffee Group
Subject: Re: [FRIAM] invitation + introduction

John,

Sounds very pertinent, and applicable to my current research - which I
call Organic Complex Systems.  Looking forward to your PDF. Pls send
links to, or copies of, your 3 pubs if you will.

Thanks,
Grant

John Kennison wrote:
 Thanks, Grant and Owen, for the votes of confidence. Concerning complex 
 adaptive systems, I would have to define a CAS in such a way that it can be 
 interpreted in any topos --then see if we can analyze CAS's by working in a 
 topos.

 Currently I am working on finding cycles. The idea is that we have a system 
 which can be in different states. Let S be the set of all states that the 
 system can be in. Let t:S to S be a transition function so that if the 
 system is now in state x, then, in the next time unit, it will be in state 
 t(x). I then look for cycles (such as t(a) = b, t(b) = c, t(c) = a, so that 
 t^3(a)=t(t(t(a)))=a --or, more generally, states x for which t^n(x)=x for 
 some n, where t^n(x) = t(t(t( t(x))  iterating t n times. Then I can 
 map the system in the best possible way into a topos where it becomes 
 cyclic, meaning that for every x there is some n with t^n(x)=x. So n would be 
 a whole number in the topos, but whole numbers can jump around and be 3 in 
 some places and 5 in other places, etc.

 Just exploring this set-up has occupied me since 2001, and I have published 3 
 papers on it in the TAC (a web-based journal).

 I'll say more and put it in a PDF file, so I  can arrows and exponents and 
 keep tabbing and spacing the way I intended it.

 ---John



 
 From: friam-boun...@redfish.com [friam-boun...@redfish.com] On Behalf Of 
 Grant Holland [grant.holland...@gmail.com]
 Sent: Friday, April 09, 2010 5:29 PM
 To: The Friday Morning Applied Complexity Coffee Group
 Subject: Re: [FRIAM] invitation + introduction

 John,

 I love such clarity - as expressed in your explanation of category theory. My 
 reaction is Oh, so THAT's what category theory is! Thanks for taking the 
 time to explain.

 Grant

 John Kennison wrote:

 Owen
 Thanks for asking the question. In my answer, below, I describe the technical 
 terminology impressionistically. If you want more precision, the Wikipedia 
 articles are usually pretty good at giving precise definitions, along with 
 some sense of the underlying ideas.

 Category theory claims to be a formalization of how mathematics actually 
 works. For example, consider the following mathematical structures, which 
 have been defined in the 19th and 20th centuries:
 Groups =   “sets with a notion of multiplication”
 Rings   =“sets with notions of both 
 multiplication and addition”
 Linear Spaces =  “sets in which vector operations can be defined”
 Topological Spaces = “sets with a notion of limit”
 Each structure has a corresponding notion of a structure-preserving function:
Group homomorphism = “function f for which f(xy) = f(x)f(y)”
Ring homomorphism = “function f for which f(xy)=f(x)f(y) and 
 f(x+y)=f(x)+f(y)”
Linear map = “function preserving operations such as scalar mult: 
 f(kv)=kf(v)”
Continuous function = “function f for which f(Lim x_n) = Lim(f(x_n)”

 A category consists of a class of objects, together with a notion of 
 “homomorphism” or “map” or “morphism” between these objects. The main 
 operation in a category is that morphisms compose (given a morphism from X to 
 Y and another from Y to Z, there is then a composite morphism from X to Z).
 Examples of catgeories:
  Objects = Groups;Morphisms = 
 Group Homomorphisms
  Objects = Rings;   Morphisms = 
 Ring Homomorphisms
 Objects = Linear spaces; Morphisms = Linear 
 maps
 Objects = Top’l spaces;   Morphisms = Cont. 
 functions
 Objects = Sets;  Morphisms = 
 Functions
 (The above examples are respectively called the categories of groups, of 
 rings, of linear spaces, of top’l spaces, and of sets.)

 The claimed advantages of using categories are:
 (1) The important and natural questions that mathematicians ask are 
 categorical in nature –that is they depend not on operations such as group 
 multiplication, but strictly on how

Re: [FRIAM] invitation + introduction

2010-04-09 Thread John Kennison
To: The Friday Morning Applied Complexity Coffee Group
Subject: Re: [FRIAM] invitation + introduction

On Apr 7, 2010, at 12:10 PM, John Kennison jkenni...@clarku.edu wrote:

 Hi Leigh,

 snip
 Nick introduced me to Rosen’s “Life Itself” and I have skimmed some articles 
 by Rosen.  I am both fascinated and disappointed by Rosen’s work. Fascinated 
 by what Rosen says about the need to develop radically different kinds of 
 models to deal with biological phenomena and disappointed by Rosen’s 
 heavy-handed stabs at developing such models. And yet still stimulated 
 because I have enough ego to believe that with my mathematical and 
 category-theoretic background, I might succeed where Rosen failed.

Category theory has been mentioned several times, especially in the early days 
of friam. Could you help us out and discuss how it could be applied here? CT 
certainly looks fascinating but thus far I've failed to grasp it.  I'd love a 
concrete example (like how to address Rosen's world) of it's use, and possibly 
a good introduction (book, article).

 Owen


I am an iPad, resistance is futile!

FRIAM Applied Complexity Group listserv
Meets Fridays 9a-11:30 at cafe at St. John's College
lectures, archives, unsubscribe, maps at http://www.friam.org


FRIAM Applied Complexity Group listserv
Meets Fridays 9a-11:30 at cafe at St. John's College
lectures, archives, unsubscribe, maps at http://www.friam.org


Re: [FRIAM] invitation + introduction

2010-04-09 Thread John Kennison

Leigh,

Is there a more complete title for the Dynamical System Theory in Biology 
volume from 1970? Is it a journal? or a series? It does sound interesting. 
Thanks.

From: friam-boun...@redfish.com [friam-boun...@redfish.com] On Behalf Of Leigh 
Fanning [le...@versiera.net]
Sent: Friday, April 09, 2010 1:35 AM
To: The Friday Morning Applied Complexity Coffee Group
Subject: Re: [FRIAM] invitation + introduction

Thank you for the welcome, and thank you also to any of you who attended
Melanie Mitchell's talk or any of the other talks today.

I don't have this particular book, however your examples are found in other
Rosen books such as the Dynamical System Theory in Biology volume from 1970.
I'm surmising that the Life Itself book must be a summary of previous works.

I cannot argue that there isn't a decidedly physics-based bent in construction
of various points.  A section on scaling in a different volume uses set
theoretic notation as a starting point which seems strange.  But, as in
picking stocks, the hindsight view is superior.  Rosen's companion volume
on Cellular Systems was published from the Center for Theoretical
Biology, in the earlier 70s this must have been among the first such
institutions formed.  In contrast, nearly 20 years later, Manfred Schroeder's
1991 Fractals, Chaos, Power Laws book gives an excellent and thorough treatment
of scaling and similarity.  And you probably have your different favorite
as well.

Having said that, I'm somewhat neutral because I haven't absorbed enough to 
make more
than observational rather than critical statements.  The initial things I have
read were intriguing, in particular one of the online essays details the idea
of anticipatory systems.

Leigh

On 07 Apr 2010 at 02:10 PM, John Kennison related


 Hi Leigh,

 I guess I?m a Friam lurker too. I?m a friend of Nick Thompson and a retired 
 math professo. I like to read the Friam posts but I comment only 
 occasionally. I?m currently working on dynamical systems and using category 
 theory to break a system down into its cyclic components.

 Nick introduced me to Rosen?s ?Life Itself? and I have skimmed some articles 
 by Rosen.  I am both fascinated and disappointed by Rosen?s work. Fascinated 
 by what Rosen says about the need to develop radically different kinds of 
 models to deal with biological phenomena and disappointed by Rosen?s 
 heavy-handed stabs at developing such models. And yet still stimulated 
 because I have enough ego to believe that with my mathematical and 
 category-theoretic background, I might succeed where Rosen failed.

 For example, in ?Life Itself? Rosen starts by talking about ?Newtonian 
 science? and the need to go beyond it, but then continues with a 
 misunderstanding of Taylor?s theorem which, thankfully, is never really used 
 in the rest of the book. Similarly, in some of his writings, Rosen talks 
 about the insolubility of the three-body problem, about Godel?s theorem, 
 about category theory, but never gets close to using any of this stuff.

 Rosen?s definition of ?component of a system? and his method of dealing with 
 ?non-recursiveness? are not just mathematically imprecise, they seem 
 completely heavy-handed and insensitive to what the situation demands. In 
 chapter 6, he gives a decomposition of a mathematical system into parts, but 
 he claims the decomposition is unique and even gives a ?proof? of this fact. 
 The proof is bogus and it?s easy to find counter-examples to what he claimed 
 he proved. (Rosen is aware of the problem since he casually notes that there 
 are some exceptions to the theorem but these are not important for he wants 
 to do. )
 Perhaps one real difference is that Rosen is a scientist, an ?inductivist? 
 who generalizes from experiments and doesn?t worry if there are exceptions. I 
 am a mathematician, a ?deductivist? who can?t tolerate exceptions. But even 
 an inductivist needs some mathematical skills and mathematical sensitivities, 
 particularly when tackling such an ambitious project as life itself.

 I?d be interested in hearing about your experience with reading Rosen.

 Welcome to Friam, from one lurker to another.

 
 From: friam-boun...@redfish.com [friam-boun...@redfish.com] On Behalf Of 
 Leigh Fanning [le...@versiera.net]
 Sent: Tuesday, April 06, 2010 9:19 PM
 To: The Friday Morning Applied Complexity Coffee Group
 Subject: [FRIAM] invitation + introduction

 First, the invitation:
 On Thusday, the University of New Mexico Computer Science department will
 hold it's annual student conference highlighting active research within
 the department.  Dr. Melanie Mitchell will give the keynote address at
 11:00 am.

 The conference is open with no admission fees, however we are not able to
 provide you with lunch.  Proceedings hardcopy can be ordered for $10,
 and will be available for free via download from the website shortly.
 Details

Re: [FRIAM] Health care [was Sources of Innovation]

2010-02-16 Thread John Kennison


Thanks Owen and Eric for giving me a simple answer to my simple question. 
Thanks also to Doug for the reassuring private email he sent me.  I do have a 
question for Doug: I don't see how the facts that you list pertain to my 
question of how uninsured people are treated when they suddenly need medical 
help. The facts that lots of people get poor health care, or even that the 
question is naive don't really give me the information I asked for. 




From: friam-boun...@redfish.com [friam-boun...@redfish.com] On Behalf Of Owen 
Densmore [o...@backspaces.net]
Sent: Monday, February 15, 2010 5:36 PM
To: The Friday Morning Applied Complexity Coffee Group
Subject: Re: [FRIAM] Health care [was Sources of Innovation]

On Feb 15, 2010, at 11:15 AM, John Kennison wrote:
 I was thinking about what Eric said and realized that I don't know
 what happens if an uninsured person suddenly needs massive medical
 treatment to avoid death or crippling consequences. If he can't pay,
 do we just let him suffer the consequences? At this point, I just
 want to find out what the current practice is. The question of what
 we should do is another matter.

I believe all emergency rooms are required to provide care, whether
the person is insured or not.  I've been in the emergency room every
year with family members needing care and they always are clear that
payment and/or insurance is not mandatory.

Also, NM, and most states, provide insurance for those who cannot
afford it.  I forget the name it goes under, but my son Gil is insured
with it.

-- Owen



FRIAM Applied Complexity Group listserv
Meets Fridays 9a-11:30 at cafe at St. John's College
lectures, archives, unsubscribe, maps at http://www.friam.org


FRIAM Applied Complexity Group listserv
Meets Fridays 9a-11:30 at cafe at St. John's College
lectures, archives, unsubscribe, maps at http://www.friam.org


Re: [FRIAM] Health Bill Summary

2009-12-24 Thread John Kennison

There is much I don't like about the two healthcare bills, and I fear that the 
compromise package will not work well (Compromises that make good sense 
politically  often fail to make sense economically --as in the cartoon of two 
congressmen making a deal: Okay, then it's agreed, I'll vote for your 
appropriation bill and you'll support my tax cut.)

But working through states might be a good idea. Maybe one state will find an 
effective way to manage the insurers and other states will copy.


From: friam-boun...@redfish.com [friam-boun...@redfish.com] On Behalf Of Owen 
Densmore [o...@backspaces.net]
Sent: Thursday, December 24, 2009 12:26 PM
To: The Friday Morning Applied Complexity Coffee Group
Subject: [FRIAM] Health Bill Summary

Given that the just-passed health bill is huge, I went looking for a
summary.  This appears reasonably complete, at least in the aims of
the bill:
   http://energycommerce.house.gov/Press_111/20090714/hr3200_summary.pdf

Any other good summary appreciated!

One question: does anyone understand why health insurance is state-
based?  I.e. if you move to a new state, you have to get new insurance
based on that state's insurance providers, and not all providers sell
in particular states.

 -- Owen




FRIAM Applied Complexity Group listserv
Meets Fridays 9a-11:30 at cafe at St. John's College
lectures, archives, unsubscribe, maps at http://www.friam.org


FRIAM Applied Complexity Group listserv
Meets Fridays 9a-11:30 at cafe at St. John's College
lectures, archives, unsubscribe, maps at http://www.friam.org


Re: [FRIAM] A little Proof, Dr Thurston! It aint Elementary!

2009-12-18 Thread John Kennison
As noted below, Thurston’s interesting paper states that: 

“Mathematics AS WE PRACTICE IT, is much more formally complete and precise than 
other sciences, but it is much less formally complete and precise for its 
content than computer programs.”

If it were required that mathematical proofs be written in full formality, then 
some of them would be unreadable. Verifying a typical published proof might be 
similar to proving the correctness of a large computer program. Even worse, the 
basic ideas of the proof would no longer be visible. As Thurston points out, 
the basic ideas are what mathematicians actually work with in practice.  

Consider the theory of integrating Cos^n (kx) (for n a positive integer) as 
taught in a calculus class. Briefly, if n is odd, we can integrate this 
function by rewriting all but one factor of Cos(kx) in terms of Sin(kx) (using 
Cos^2(kx)=1-Sin^2(kx)) then making the substitution U = Sin(kx). If n is even, 
we can use a trigonometric identity to reduce the problem to integrating 
smaller powers of Cos(kx). [The identity is Cos^2(kx)=(1+Cos(2kx))/2.] Thus, we 
can reduce the problem to finding integrals of smaller and smaller even powers 
of Cos (kx) until we get to the second or first power, Cos^2(kx) or Cos(kx),  
both of which can readily be integrated. 

The above argument would be clear to mathematicians, and extremely good 
calculus students, and would enable them to compute certain integrals. Calculus 
students who are good, but not extremely good, would generally need to see a 
few examples before the process becomes clear. Weaker students would need to 
see more examples –maybe a lot more. Theoretically, the argument in the above 
paragraph would be an acceptable proof that, for example, the integral of 
Cos^n(kx) can always be carried out and has a certain form.  Although the 
argument is informal and sketchy, it does give the main ideas, which is enough 
for mathematicians to convince themselves that the method is correct.  A 
completely formal version of the argument would be like constructing a program 
to do the integration, then verifying that the program works. The main ideas 
would be lost. 
 
--John


From: friam-boun...@redfish.com [friam-boun...@redfish.com] On Behalf Of 
Nicholas Thompson [nickthomp...@earthlink.net]
Sent: Tuesday, December 15, 2009 4:10 PM
To: friam@redfish.com
Subject: Re: [FRIAM] A little Proof, Dr Thurston! It aint Elementary!

Robert,

thanks for the additional quotations.

However, you made a slip of the fingers when you keyed in one of the passages.  
To head off needless controversy, I key it in correctly below.  The capitalized 
word is where the slipup occured.

Mathematics as we practice it is much MORE formally complete and precise than 
other sciences, but it is much less formally complete and precise for its 
content than computer programs.
Easy slip to make because of the structure of the sentence.

n


Nicholas S. Thompson
Emeritus Professor of Psychology and Ethology,
Clark University (nthomp...@clarku.edumailto:nthomp...@clarku.edu)
http://home.earthlink.net/~nickthompson/naturaldesigns/
http://www.cusf.org [City University of Santa Fe]




- Original Message -
From: Marcus G. Danielsmailto:mar...@snoutfarm.com
To: friam@redfish.commailto:friam@redfish.com
Sent: 12/15/2009 1:09:23 PM
Subject: Re: [FRIAM] A little Proof, Dr Thurston! It aint Elementary!

On 12/15/09 12:27 PM, Robert J. Cordingley wrote:
I think this is him:
http://en.wikipedia.org/wiki/William_Thurston
The essay that Russ mentioned only mentions programming in passing..  He 
doesn't say anything about it relative to `intellectual challenge', but he does 
talk a lot about what is the deep value of his enterprise.   The message is in 
some sense that the rigor is not the end, it's the means.  Some quotes:

When one considers how hard it is to write a computer program even approaching 
the intellectual scope of a good mathematical paper, and how much greater time 
and effort have been put into it to make it almost formally correct, it is 
preposterous to claim that mathematics as we practice it is anywhere near 
formally correct.

Mathematics as we practice it is much less formally complete and precise than 
other scienc es, but it is much less formally complete and precise for its 
content than computer programs.  The difference has to do not just with the 
amount of effort:  the kind of effort is qualitatively different.  In large 
computer programs, a tremendous proportion of effort must be spent on myriad 
compatibility issues:  making sure that all of the definitions are consistent, 
developing good data structures that have useful but not cumbersome 
generality, deciding on the right generality for functions, etc.  The 
proportion of effort spent on the working part of a large program, as 
distinguished from the bookkeeping part, is surprisingly small.  Because of the 
compatibility issues that almost 

Re: [FRIAM] emergence

2009-09-07 Thread John Kennison

Isn't it possible that an emergent phenomenon might be mysterious to an 
observer who didn't know how it was implemented? For example, how might 
lodestones(?) (I mean magnetized rocks) appear to someone who observed them 
before the theory of magnetism had been formulated?

From: friam-boun...@redfish.com [friam-boun...@redfish.com] On Behalf Of 
Nicholas Thompson [nickthomp...@earthlink.net]
Sent: Sunday, September 06, 2009 7:20 PM
To: friam@redfish.com
Subject: Re: [FRIAM] emergence

OK.  On the question of what Bedau believes, I leave the field in a rout!  
However, I want to look at Bedau's own article in the book, where he seems 
mostly to treat emergence quite casually, before I decide whether I want to try 
to reinfiltrate the field in the night.

But you do realize, Russ, to your shame, that we agree on one important point.  
Whatever Bedau might believe, you and I believe that emergence is ubiquitous 
and non-mysterious.



Nicholas S. Thompson
Emeritus Professor of Psychology and Ethology,
Clark University (nthomp...@clarku.edumailto:nthomp...@clarku.edu)
http://home.earthlink.net/~nickthompson/naturaldesigns/




- Original Message -
From: Russ Abbottmailto:russ.abb...@gmail.com
To: nickthomp...@earthlink.netmailto:nickthomp...@earthlink.net;The Friday 
Morning Applied Complexity Coffee Groupmailto:friam@redfish.com
Sent: 9/6/2009 3:00:12 PM
Subject: Re: [FRIAM] emergence

Come on Nick. Later on in the Introduction they write the following.

When we finally understand what emergence truly is, we might see that many of 
the examples are only apparent cases of emergence. Indeed, one of the hotly 
contested issues is whether there are any genuine examples of emergence.

Here's how the Introduction finishes.

The study of emergence is still in its infancy and currently is in a state of 
considerable flux, so a large number of important questions still lack clear 
answers. Surveying those questions is one of the best ways to comprehend the 
nature and scope of the contemporary philosophical and scientific debate about 
emergence. Grouped together here are some of the interconnected questions about 
emergence that are particularly pressing,

1. How should emergence be defined? ... We should not presume that only one 
type of emergence exists and needs definition. Instead, different kinds of 
emergence may exist, so different that they fall under no unified account. ... 
Given the high level of uncertainty about how to properly characterize what 
emergence is, it should be no surprise that many other fundamental questions 
remain unanswered.

2. What ontological categories of entities can be emergent: properties, 
substances, processes,phenomena, patterns, laws, or something else? ...

3. What is the scope of actual emergent phenomena? ...

4. Is emergence an objective feature of the world, or is it merely in the eye 
of the beholder? ...

5. Should emergence be viewed as static and synchronic, or as dynamic and 
diachronic, or are both possible? ...

6. Does emergence imply or require the existence of new levels of phenomena? ...

7. In what ways are e mergent phenomena autonomous from their emergent bases? 
... Another important question about the autonomy of emergent phenomena is 
whether that autonomy is merely epistemological or whether it has ontological 
consequences. An extreme version of the merely epistemological interpretation 
of emergence holds that emergence is simply a sign of our ignorance. One final 
issue about the autonomy of emergent phenomena concerns whether emergence 
necessarily involves novel causal powers, especially powers that produce 
??downward causation,?? in which emergent phenomena have novel effects on their 
own emergence base. One of the questions in this context is what kind of 
downward causation is involved, for the coherence of downward causation is 
debatable.

Emergence ... is simultaneously palpable and confusing ... New advances in 
contemporary philosophy and science ... now are converging to enable new 
progress on these questions ...
This book?s chapters illuminate these que stions from many perspectives to help 
readers
with framing their own answers.

If this isn't an attempt to grapple with an apparently mysterious phenomenon 
what do you think it is? Or do you suppose they are simply compiling a 
collection of philosophical papers for the sake of history?  If that were the 
case, I would think they would make the philosophical landscape of emergence 
sound a lot more settled.  Or perhaps they simply believe that they can make 
some money selling books -- and writing the introduction as if the topic of 
energence were so unsettled was just a way to intice people to buy it.

-- Russ


On Sun, Sep 6, 2009 at 1:13 PM, Nicholas Thompson 
nickthomp...@earthlink.netmailto:nickthomp...@earthlink.net wrote:
seems would seem to be the operative word.  He is the editor of the book and 
he has to represent the 

Re: [FRIAM] manifold in mathematics

2009-08-06 Thread John Kennison
Lee, 

You do seem to be on the list. If everyone apologized for nearly content-free 
posts, the number of nearly content-free posts would be greatly multiplied.

The word manifold is certainly not being used in its mathematical sense when 
Epping Forest is called a manifold. I think it means a collective noun there, 
so that all this means the collection of all this, not the individual 
components of all this. Still, I would be interested in what you have to say.
 
--John


From: lrudo...@black.clarku.edu [lrudo...@black.clarku.edu]
Sent: Wednesday, August 05, 2009 12:49 PM
To: friam@redfish.com; nickthomp...@earthlink.net
Cc: John Kennison
Subject: Re: [FRIAM] manifold in mathematics

On 5 Aug 2009 at 10:37, Nicholas Thompson wrote:

 There is a topologist on the list (at least one)

Before attempting a substantive reply to this post, I am
going to try posting to the list to see if I *am* indeed on
the list.  John, do you know for sure that you are on the
list?  If not, you might try the same...

(Apologies to all if, in fact, I am on the list, and
everyone gets this nearly content-free post.)


FRIAM Applied Complexity Group listserv
Meets Fridays 9a-11:30 at cafe at St. John's College
lectures, archives, unsubscribe, maps at http://www.friam.org


Re: [FRIAM] A great story about forgetting what's real

2009-06-22 Thread John Kennison


Yes, that's a terrific story. I liked the phrase hard-earned TARP money

From: friam-boun...@redfish.com [friam-boun...@redfish.com] On Behalf Of Russ 
Abbott [russ.abb...@gmail.com]
Sent: Sunday, June 21, 2009 9:55 PM
To: The Friday Morning Applied Complexity Coffee Group
Subject: [FRIAM] A great story about forgetting what's real

I just came across this amusing story that illustrates what can happen when one 
takes a metaphor for reality.

-- Russ


FRIAM Applied Complexity Group listserv
Meets Fridays 9a-11:30 at cafe at St. John's College
lectures, archives, unsubscribe, maps at http://www.friam.org


[FRIAM] Theories of behavior

2009-06-20 Thread John Kennison

Russ,

I agree that the question of what it means to say a person acts as of deeply 
hurt is not easy to answer. For example, some people would become 
non-communicative while others would communicate aggressively, so how do I come 
up with a third-person description of acting as if deeply hurt?. I would have 
to rely on an elaborate theory that would determine, from a person's history of 
past behavior and a large sample of current behavior that the person is, for 
example, depressed.  (I'm not certain if current psychological theory is stated 
that way.)

This suggests a question for Nick
Suppose that in analyzing behavior, we found that the rules (or probabilistic 
rules) governing behavior were greatly simplified if we assumed that the person 
could be in various states that resembled what we usually think of as 
psychological states, such as depression, fear, elation, etc.  Would this count 
as evidence for inner states?
---John

From: Russ Abbott [russ.abb...@gmail.com]
Sent: Friday, June 19, 2009 10:57 AM
To: John Kennison
Cc: The Friday Morning Applied Complexity Coffee Group; 
nickthomp...@earthlink.net; e...@psu.edu
Subject: Re: [FRIAM] Nick and dishonest behavior

As I wrote to Nick directly, I think Nick is gracious and kind and a man of 
great integrity.

But this doesn't make sense to me: We don't have to believe in inner minds to 
say that a person accused of dishonesty behaves as if deeply hurt. What could 
it possibly mean to say that a person is deeply hurt if there is no such thing 
as first person experience?  And if there is no such thing as being deeply hurt 
in a first person way, what could it possibly mean to say that someone is 
behaving as if deeply hurt?

This suggests that it is very dangerous to claim that there is no first person 
experience and that observable behavior is all there is. It would encourage 
treating people as objects because that's exactly the position it takes. An 
attitude of this sort would seem to discard millennia of progress in our 
understanding and acceptance of what ethical human-to-human interaction 
consists of.

-- Russ




FRIAM Applied Complexity Group listserv
Meets Fridays 9a-11:30 at cafe at St. John's College
lectures, archives, unsubscribe, maps at http://www.friam.org


Re: [FRIAM] Nick and dishonest behavior

2009-06-19 Thread John Kennison


Nick and I are on opposite sides of the consciousness debate. I think there is 
an inner mind and that I experience it. Nick rejects statements not made from 
the third person perspective. Perhaps the debate suffers from a feeling that if 
we take Nick's third person view, we are not allowed to use metaphorical 
statements that suggest an inner mind. But clearly we can say The computer had 
an illusion or a breakdown etc. to describe behavior. (e.g. The behavior was 
as we imagined it would be if the computer had a inner mind which suffered a 
breakdown.) Moreover, not only can these metaphorical statements about behavior 
be defined rigorously, but we can formulate and test rules about how they are 
related. We don't have to believe in inner minds to say that a person accused 
of dishonesty behaves as if deeply hurt. That is why we should not casually 
make such accusations nor assume they will be without negative consequences 
even if there is no inner mind.




From: friam-boun...@redfish.com [friam-boun...@redfish.com] On Behalf Of Russ 
Abbott [russ.abb...@gmail.com]
Sent: Thursday, June 18, 2009 11:07 PM
To: nickthomp...@earthlink.net
Cc: friam@redfish.com; e...@psu.edu
Subject: Re: [FRIAM] Nick and dishonest behavior

Nick wrote:

To call a man dishonest (my word, I admit, but you have embraced it) is very 
harsh in my world, and seems (to me) to require a level of certainty about 
another person's motives that I just don't know how you could come by from your 
limited experience with me.  ...

You are insisting on the correctness of your view of my mind based on 
inferences from my behavior.

Yes, I'm doing exactly that, judging you on the basis of your behavior -- in 
this conversation. (The past 40 years aren't relevant to that.) Your position 
in this discussion seems to be that your behavior is all there is. So why are 
you objecting that I'm doing it?

Furthermore, your objection seems to be that I don't know what your motives 
are.  I'm not sure what you mean by motives in this case. I'm not assuming any 
particular motive. In fact I'm confused about what your motives might be and 
why you are acting so dishonestly. Yet you are acting dishonestly.

To review: a good example of your dishonest behavior was your answer to my 
question about nausea. Your provided a very nice first person description of 
what it means to feel nauseous.

If you say that you are feeling nauseous i will understand that your world 
seems like it is churning around but that your visual cues do not confirm 
(i.e., you are dizzy) and that your stomach feels the way it does when on 
previous occasions you have thrown up.

Note your use of the first person words seems and feels. But  then you refused 
to answer whether that description would ever apply to a robot. Instead you 
offered a 3rd person description of what it looks like to feel nauseous and 
said that of course a robot could fit that description. I call that dishonest.  
You know what a first person description means because you used it yourself. 
But then you refused to answer the question whether such a first person 
description could apply to a robot. Furthermore, you refused to acknowledge 
that this is what you were doing. I see that as dishonest. But I don't know 
what your motives for acting this way might be.

Besides, why are you so concerned about my characterizing your behavior as 
dishonest? Why is that a very harsh term? It's simply a description of your 
behavior.

Are you upset because you are taking my use of the term dishonest to apply more 
broadly than to your behavior? In the second passage of yours quoted above, you 
talked about my view of your mind. Are you unhappy that I seem to be implying 
that your mind is dishonest? I thought your position was that there is no mind 
for me to have a view of. I thought your position was that behavior was all 
that mattered. It should not matter to you what my view of your mind is if it 
doesn't mean anything to talk about minds.


-- Russ


FRIAM Applied Complexity Group listserv
Meets Fridays 9a-11:30 at cafe at St. John's College
lectures, archives, unsubscribe, maps at http://www.friam.org


Re: [FRIAM] The FRIAM mind (was: 'Do robots dream of electricillusions? or Bladerunner, theRealist's Cut')

2009-06-19 Thread John Kennison

I'd like to switch from robots to towns. Sometimes we loosely talk of a town's 
spirit. We might say that the spirit is dampened when the town's little league 
team loses to the team from an adjoining town. Or that the town is suffering 
from a political illusion, etc. But we can say these things yet maintain that 
there really isn't a town spirit or an inner mind of the town. I don't think 
we have the scientific knowledge to come up with much scientific evidence for 
or against the position that inner minds exist for people, or for towns for 
that matter.   So far, the one reason I believe there are inner minds is purely 
subjective. Yes, I realize this is conceding something, maybe quite a lot, to 
Nick, but I don't see how my sense of awareness can be an illusion.


 

From: friam-boun...@redfish.com [friam-boun...@redfish.com] On Behalf Of 
Nicholas Thompson [nickthomp...@earthlink.net]
Sent: Friday, June 19, 2009 11:17 AM
To: Friam@redfish.com
Subject: Re: [FRIAM] The FRIAM mind (was: 'Do robots dream of   
electricillusions? or Bladerunner, theRealist's Cut')

Jochen,

Nice.  I discovered when I moved to a small town in New England 40 years
ago, I found  that there were lots of theys but no we's.  So, people
would get together and discuss the bad behavior of they who lived up
town, but when you got up town, you never could find any we that
corresponded to that they.

So, what if your consciousness is just like that.  I look across the table
and I see one body, and I assume that there is a mind to correspond to
that unitary body.  But lo, it isnt true.  Over there, on that side of the
table, there is no I that corresponds to that he, just a bunch of
different systems struggling to be in control of the body and to look
responsible to the people accross the table.  Given that a body can get
chucked in the loony bin for not being unitary, it is not surprizing that
the conflicting systems that control it would be under some constraint to
try to appear organized to the outside world.

Isnt this very close to Dennett's view in CONSCIOUSNESS EXPLAINED?

Nick



Nicholas S. Thompson
Emeritus Professor of Psychology and Ethology,
Clark University (nthomp...@clarku.edu)
http://home.earthlink.net/~nickthompson/naturaldesigns/




 [Original Message]
 From: Jochen Fromm jfr...@t-online.de
 To: The Friday Morning Applied Complexity Coffee Group friam@redfish.com
 Date: 6/19/2009 4:35:43 AM
 Subject: [FRIAM] The FRIAM mind (was: 'Do robots dream of
electricillusions? or Bladerunner, theRealist's Cut')

 You are talking about the list as if it is a single
 entity (let us see how 'the list' responds), although
 it is composed of several independent individuals:
 Russ, Stephen, Glen, Douglas, to name a few.
 Can we think of the mind as a similar kind of
 list or group, which is composed of several agents?
 Is the society of mind metaphor from Minsky
 helpful to explain behavior?

 If the FRIAM list discusses itself, would
 this be a form of self-consciousness for
 the FRIAM mind?

 -J.

 - Original Message -
 From: Nicholas Thompson
 To: friam@redfish.com
 Cc: Sent: Thursday, June 18, 2009 11:32 PM
 Subject: Re: [FRIAM] Do robots dream of electric illusions? or
 Bladerunner,theRealist's Cut

 [...]
 It will be interesting to see how the list responds.

 Nick

 Nicholas S. Thompson
 Emeritus Professor of Psychology and Ethology,
 Clark University (nthomp...@clarku.edu)
 http://home.earthlink.net/~nickthompson/naturaldesigns/


 
 FRIAM Applied Complexity Group listserv
 Meets Fridays 9a-11:30 at cafe at St. John's College
 lectures, archives, unsubscribe, maps at http://www.friam.org




FRIAM Applied Complexity Group listserv
Meets Fridays 9a-11:30 at cafe at St. John's College
lectures, archives, unsubscribe, maps at http://www.friam.org


FRIAM Applied Complexity Group listserv
Meets Fridays 9a-11:30 at cafe at St. John's College
lectures, archives, unsubscribe, maps at http://www.friam.org


Re: [FRIAM] The ghost in the machine (was 'quick question')

2009-06-15 Thread John Kennison


Nick,

I am puzzled about all statements being metaphors. To me one feature about 
metaphors is that we must tolerate inconsistencies. 
For example,  I can say (metaphorically) that my friend is a political tiger; 
that all tigers have long tails but my friend does not have a long tail. If I 
weren't using metaphors the last two statements would contradict the first. So 
it would be very difficult to conclude that a specimen is not a tiger, when 
speaking metaphorically. But you claim that I, for example, suffer from an 
illusion of being conscious. I presume this means that you have concluded that 
I believe I am conscious but that I am not conscious. From what perspective do 
you draw these conclusions? What do you observe from that perspective that 
leads you to those conclusions? Particularly that I am not conscious, seeing 
that you would have difficulty even concluding that my friend is not a tiger. 

--John




From: Nicholas Thompson [nickthomp...@earthlink.net]
Sent: Monday, June 15, 2009 1:32 AM
To: John Kennison; friam@redfish.com
Subject: RE: [FRIAM] The ghost in the machine (was 'quick question')

John,

John,

All good questions.

I dont think I make a distinction between precise statements and metaphors.
I think I think it's metaphors all the way down

I also I think I think there is no such thing as a first person perspective
... not really.  Specting seeing the world from a position ... is what
every creature does.  One of the events that I can spect, is a creature
specting its world, and one of the creatures that I can spect, in this way
, is myself.  Not my inner processes or my mind, but me, an actor in the
world.

Like all observers, I am situated, and since I am the only person who is
around me all the time, I am situated in a particularly unique way with
respect to myself.  My situation may sight me or blind me, depending on the
kind of information that is required to make an accurate prediction about
what I will do.

Those are my best answers.

Nick



Nicholas S. Thompson
Emeritus Professor of Psychology and Ethology,
Clark University (nthomp...@clarku.edu)
http://home.earthlink.net/~nickthompson/naturaldesigns/




 [Original Message]
 From: John Kennison jkenni...@clarku.edu
 To: nickthomp...@earthlink.net nickthomp...@earthlink.net; The
FridayMorning Applied Complexity Coffee Group friam@redfish.com
 Date: 6/14/2009 9:35:12 PM
 Subject: RE: [FRIAM] The ghost in the machine (was 'quick question')



 Nick,

 I'm not sure if I am correctly representing your position about the
third-person point of view, but I would agree that if we want to construct
a scientific theory of consciousness, it must be based on a third person
approach. But it seems possible that there  are some facts about 'the world
as it really is' that are not now accessible to science. If this is so, the
impressions we receive from the first-person point of view may offer us the
best insights we can get, given the current state of scientific knowledge.
So why must we rigorously ignore such impressions?

 I agree with your point that our language about consciousness is not very
consistent. Trying to use precise language about our minds may be as
difficult as creating a scientific theory of our 'inner lives'. Maybe when
discussing this area, we can only use language metaphorically and hope that
the person we are communicating with can make sense of it. What about your
statements that 'consciousness is an illusion' or a 'huge language game' .
Are these metaphors or precise statements?

 --John
 
 From: friam-boun...@redfish.com [friam-boun...@redfish.com] On Behalf Of
Nicholas Thompson [nickthomp...@earthlink.net]
 Sent: Sunday, June 14, 2009 1:24 PM
 To: friam@redfish.com
 Subject: Re: [FRIAM] The ghost in the machine (was 'quick question')

 Dear Jochen,

 What I am about to say will seem crazy and I certainly don't expect to
 convince you.  At max, I might get you to try out the world from this
 rather strange point of view, and see why somebody might explore it.

 My basic position is that consciousness is an illusion.  I am not talking
 user-illusion here or even  epiphenomenalism.  What I am saying is much
 stronger and more obnoxious than either of those positions.   The best
 metaphor I can think of is the sun rising.  We all talk as if the sun
 rises, but it doesn't, or at best, the statement, the sun rises, relates
 only in a vague way to the actual state of affairs.  Our belief that the
 sun rises  get's its force not from the facts but from the enormous
 authority of language, and other social arrangements. Consciousness is a
 huge language game, which we violate on pain of being called crazy.

 So what do I have to offer instead?  Well, nothing, actually.  I confess
to
 being as caught in the illusion as anybody else.   All I can say is that
 the way we talk about consciousness verges seems not to make a lot of
 sense

Re: [FRIAM] The ghost in the machine (was 'quick question')

2009-06-14 Thread John Kennison


Nick,

I'm not sure if I am correctly representing your position about the 
third-person point of view, but I would agree that if we want to construct a 
scientific theory of consciousness, it must be based on a third person 
approach. But it seems possible that there  are some facts about 'the world as 
it really is' that are not now accessible to science. If this is so, the 
impressions we receive from the first-person point of view may offer us the 
best insights we can get, given the current state of scientific knowledge. So 
why must we rigorously ignore such impressions?

I agree with your point that our language about consciousness is not very 
consistent. Trying to use precise language about our minds may be as difficult 
as creating a scientific theory of our 'inner lives'. Maybe when discussing 
this area, we can only use language metaphorically and hope that the person we 
are communicating with can make sense of it. What about your statements that 
'consciousness is an illusion' or a 'huge language game' . Are these metaphors 
or precise statements?

--John

From: friam-boun...@redfish.com [friam-boun...@redfish.com] On Behalf Of 
Nicholas Thompson [nickthomp...@earthlink.net]
Sent: Sunday, June 14, 2009 1:24 PM
To: friam@redfish.com
Subject: Re: [FRIAM] The ghost in the machine (was 'quick question')

Dear Jochen,

What I am about to say will seem crazy and I certainly don't expect to
convince you.  At max, I might get you to try out the world from this
rather strange point of view, and see why somebody might explore it.

My basic position is that consciousness is an illusion.  I am not talking
user-illusion here or even  epiphenomenalism.  What I am saying is much
stronger and more obnoxious than either of those positions.   The best
metaphor I can think of is the sun rising.  We all talk as if the sun
rises, but it doesn't, or at best, the statement, the sun rises, relates
only in a vague way to the actual state of affairs.  Our belief that the
sun rises  get's its force not from the facts but from the enormous
authority of language, and other social arrangements. Consciousness is a
huge language game, which we violate on pain of being called crazy.

So what do I have to offer instead?  Well, nothing, actually.  I confess to
being as caught in the illusion as anybody else.   All I can say is that
the way we talk about consciousness verges seems not to make a lot of
sense, much of the time.

For instance, not only do we talk as if the conscious-actor can act on his
body, or through his body, on the world;  we also talk as if the
conscious-actor can act on his own mind, e.g.,   make it up like a
rumpled bed.  In these intra-mental transactions, who is the agent and who
the receiver of the action?  Only in talking about consciousness do we
allow the agent to act upon itself in such a profligate way.

An other oddity is our curious ambivalence concerning   third-person point
of view.  There are four billion people in the world, right?  When you and
I speak of any of those people, we take a third-person point of view.
Early in the conversation, we will make a decision, depending on our
metaphysics, concerning whether another person's consciousness is something
we have access to, or not.  Some will take the position that we never
REALLY can know what is in another person's mind.  We could, of course, ask
the agent, but the agent need not tell us the truth.  So we are stuck
because [scientific] knowledge of another's mind is beyond our reach.  For
such people, a scientific conversation concerning the true thoughts,
feelings, intentions, etc., of another person is not possible.

But what of people who don't hold to the primacy of the first person view.
With such people we can have a conversation about the true intentions of
another person, confident that we can get to the truth of the matter.  Was
OJ Simpson a murderer?  Don't ASK him;  look at the evidence.  Our legal
system is based on the notion that the intentions of an agent are something
that a jury of peers can assess.  In such circumstances, we are convinced
that we can invade the so called privacy of the mind.

But even people who grant their own powers to see the true intentions of
others, still grant themselves primacy in the determination of their own
behavior.  To that extent, we indulge ourselves in a dualism in which we
hold one theory that works for ourselves and another theory that works for
the other 4 billion people on earth.  And it is the personal  theory that
holds the most sway when called upon to talk about the relationship between
the brain and consciousness.

Ok, so having confessed to all of that, please allow me to comment on your
letter below.  I will use CAPS, because it is a quick way to distinguish my
text from yours.  Owen will accuse me of SHOUTING, which I promise I am
not.  I am speaking in a teensy weensy voice.

All the best,

Nick

Nicholas S. Thompson
Emeritus 

Re: [FRIAM] Robots forming human-like societies - electronic evolution? // Current

2009-05-31 Thread John Kennison


Nick,

It seems to me that a negotiating model assumes that the robots have an agreed 
upon method of communication, which includes transmitting of offers. Also, it 
is assumed that we have a model of each robot's position, which might be 
simplified to something like 'the robot moves to the place where another robot 
suggested there was food'. As I read this article, the robots have lights and 
light-sensors, an ability to physically move and to switch lights off and on, 
and a program which determines how it does these things. So methods of 
communication must evolve and decisions such as 'move to where another robot 
indicated food' must be expressed as specific physical motions.

Two questions arise: Does the resulting evolution of the physical robots 
reflect anything that would be suggested by examining models of negotiating 
strategies. The answer seems to be yes, very much so. The other question is 
whether all of the behavior exhibited by robots is predictable by such models. 
Or is it the case that the physical set up has possibilities that we would 
almost certainly overlook no matter how we tried to define some negotiating 
entities. This still seems to be open.

---John  
  

From: friam-boun...@redfish.com [friam-boun...@redfish.com] On Behalf Of 
Nicholas Thompson [nickthomp...@earthlink.net]
Sent: Sunday, May 31, 2009 2:53 AM
To: friam@redfish.com
Cc: e...@psu.edu
Subject: [FRIAM] Robots forming human-like societies - electronic evolution?
// Current

Dear All,

I guess my naive question here is, Were the robots, as such, necessary.  Is 
there anything you can do with robots that you can't do with netlogo  well, 
except have the scratch your back, or something.  Anything of THEORETICAL 
significance?

http://current.com/items/90119924_robots-forming-human-like-societies-electronic-evolution.htm


Nicholas S. Thompson
Emeritus Professor of Psychology and Ethology,
Clark University (nthomp...@clarku.edumailto:nthomp...@clarku.edu)
http://home.earthlink.net/~nickthompson/naturaldesigns/






FRIAM Applied Complexity Group listserv
Meets Fridays 9a-11:30 at cafe at St. John's College
lectures, archives, unsubscribe, maps at http://www.friam.org


Re: [FRIAM] emergence, again

2009-04-29 Thread John Kennison

As a practicing mathematician, my understanding is that it is permissible to 
define anything by a property if and only if you can prove there exists a 
unique thing with that property.
For example, you cannot define sqrt(49) as an integer whose square is 49 
since there are two such integers. Nor could you define sqrt(-1) as the real 
number whose square is -1 as there is no such real number (and you can't define 
sqrt(-1) as the complex number whose square is -1 as there are two such complex 
numbers, i and -i.)
So circular definitions (where A is defined in terms of B and B in terms of A) 
are permissible if and only if you can show there is a unique pair (A,B) with 
the given relation.


On 4/29/09 9:21 AM, glen e. p. ropella g...@agent-based-modeling.com wrote:

Thus spake Nicholas Thompson circa 04/28/2009 08:33 PM:
 let a, b, and c
 constitute macro-entity E and let the behavior of E. be controled by the
 properties and intereactions of a, b and c.  Now, let one of the behaviors
 of E to control the behavior of a, b, or c.  Is there a problem here?

There's no problem with it.  It's called an impredicative definition,
which basically means the application of a universal quantifier (e.g.
for all) over a set as a part of the definition of the members of that
set.  (IIRC, of course... ;-)

Here's a quote from Barwise and Moss' Vicious Circles that may address
the problem you've heard philosophers talk about:

In certain circles, it has been thought that there is a conflict
between circular phenomena, on the one hand, and mathematical rigor, on
the other.  This belief rests on two assumptions.  One is that anything
mathematically rigorous must be reducible to set theory.  The other
assumption is that the only coherent conception of set precludes
circularity.  As a result of these two assumptions, it is not uncommon
to hear circular analyses of philosophical, linguistic, or computational
phenomena attacked on the grounds that they conflict with one of the
basic axioms of mathematics.  But both assumptions are mistaken and the
attack is groundless.

--
glen e. p. ropella, 971-222-9095, http://agent-based-modeling.com



FRIAM Applied Complexity Group listserv
Meets Fridays 9a-11:30 at cafe at St. John's College
lectures, archives, unsubscribe, maps at http://www.friam.org



FRIAM Applied Complexity Group listserv
Meets Fridays 9a-11:30 at cafe at St. John's College
lectures, archives, unsubscribe, maps at http://www.friam.org

Re: [FRIAM] The unreasonable Effectiveness of ABMs in Complex Systems

2009-04-28 Thread John Kennison
Thanks Robert for your reply

I want to move on to the question of where math is effective. Previously, I 
wondered about the existence of domains where short logical implications were 
reliable but long chains of logical implications may start to be ineffective. 
In a sense this is true of any chaotic system, such as weather. We can now 
predict weather fairly well for the short term but not for the long term 
because we cannot measure the initial conditions to the required degree of 
precision (as even arbitrarily small changes now can cause big changes in 
future states). It is posible that weather is mathematically determined, say 
perfectly described by some chaotic system and yet math itself would be only of 
limited use in predicting weather?

Perhaps Physics has (so far, mainly) only analyzed non-chaotic phenomena.

This raises the question of whether some other mathematical system, say one not 
involving numbers, could tell us somethging useful about chaotic phenomena. 
Maybe the use of ABMs would work, as suggested by Jochen.

From: friam-boun...@redfish.com [friam-boun...@redfish.com] On Behalf Of Jochen 
Fromm [jfr...@t-online.de]
Sent: Tuesday, April 28, 2009 2:27 PM
To: The Friday Morning Applied Complexity Coffee Group
Subject: [FRIAM] The unreasonable Effectiveness of ABMs in Complex Systems

If physics is so successfully described by mathematics
because the physical world is mathematical, and nearly
isomorphic to a mathematical structure, then maybe
complex systems are so successfully described by ABMs
because their are isomorphic to them, too. Complex systems,
especially social ones, are agent-oriented.
What do you think ?

-J.


FRIAM Applied Complexity Group listserv
Meets Fridays 9a-11:30 at cafe at St. John's College
lectures, archives, unsubscribe, maps at http://www.friam.org


FRIAM Applied Complexity Group listserv
Meets Fridays 9a-11:30 at cafe at St. John's College
lectures, archives, unsubscribe, maps at http://www.friam.org


  1   2   >